Medical PG Orthopaedics

MEDICAL PG ORTHOPAEDICS

1. Damage to the radial nerve in the spinal groove spares which muscle
(A) Lateral head of triceps
(B) Long head of triceps
(C) Medial head of triceps
(D) Anconeus


Answer: (B)

2. Meralgia paraesthetic is due to the involvement of the
(A) Ulnar nerve
(B) Median nerve
(C) Lateral cutaneous nerve of thigh
(D) Lateral peroneal nerve of thigh

Answer: (C)

3. Cubitus varus is commonly caused by
(A) Supracondylar fracture of humerus
(B) Fracture of the lateral epicondyle
(C) Chronic osteromyelitis
(D) Acute epiphysitis

Answer: (A)

4. Management of Smith’s fracture is
(A) Open reduction and fixation
(B) Plaster cast with forearm in pronation
(C) Closed reduction with below-elbow cast
(D) Above-elbow cast with forearm in supination

Answer: (D)

5. Ortolani’s test is done for
(A) Congenital dislocation of hip
(B) Traumatic dislocation of hip
(C) Rheumatoid arthritis
(D) Tubuerculous arthritis

Answer: (A)

6. Treatment of choice for one week old fracture neck femur at 65 years age is
(A) Hemi-replacement arthroplasty
(B) Closed reduction and internal fixation by cannulated cancellous screws
(C) Closed reduction and internal fixation by Austin more pins
(D) Total hip replacement

Answer: (A)

7. All the following are benign tumors except
(A) Chondroma
(B) Chordoma
(C) Teratoma
(D) Meningioma

Answer: (B)

8. Solitary bone cyst is most common in the
(A) Upper end of humerus
(B) Lower end of humerus
(C) Lower end of humerus
(D) Lower end of femur

Answer: (A)

9. Erosion of bone is seen with all of the following except
(A) Gout
(B) SLE
(C) Psoriasis
(D) Rheumatoid arthritis

Answer: (B)

10. Which of the following is true regarding a phantom limb
(A) Occurs in leprosy
(B) Follows amputation
(C) Follows a psychiatric illness
(D) After filariasis

Answer: (B)

11. The commonest complication of Colle’s fracture is
(A) Malunion
(B) Non-union
(C) Sudeck’s osteodystrophy
(D) Stiffness of fingers

Answer: (D)

12. Injury to the Ulnar nerve at the wrist causes paralysis of
(A) Apposition of the thumb
(B) Abduction of the carpo-metacarpal joint of the thumb
(C) Adduction of the thumb
(D) Flexion of the MCP joint of the middle finger

Answer: (C)

13. All the following are causes of a painful limb except
(A) Slipped femoral epiphysis
(B) TB of the hip
(C) Perthes disease
(D) Infantile Coxa Vara

Answer: (D)

14. Which joint is commonly involved in Osterochondritis Dissecans
(A) Ankle joint
(B) Knee joint
(C) Wrist joint
(D) Elbow joint

Answer: (B)

15. Painless effusion in joints Congential Syphilis are called
(A) Clutton’s joint
(B) Charcot’s joint
(C) Barton’s joint
(D) Chronic osteromyelitis

Answer: (A)

16. In rheumatoid arthritis, pathology starts in the
(A) Articular cartilage
(B) Capsule
(C) Synovium
(D) Muscles

Answer: (C)

17. Treatment of choice for Paget’s disease of the bone is
(A) Vitamin D
(B) Immobilization of the limb
(C) Surgical treatment
(D) Calcitonin

Answer: (D)

18. Commonest site of TB spine is
(A) Dorsolumbar
(B) Lumbar
(C) Lumbosacral
(D) Cervical

Answer: (A)

19. All the following are true about musculocutaneous nerve injury at shoulder except
(A) Loss of flexion at shoulder
(B) Loss of flexion of forearm
(C) Loss of supination of forearm
(D) Loss of sensation on radial side of forearm

Answer: (A)

20. The nerve involved in anterior dislocation of the shoulder is
(A) Radial nerve
(B) Axillary nerve
(C) Ulnar nerve
(D) Musculocutaneous nerve

Answer: (B)

21. The most common type of Supracondylar fracture is
(A) Flexion type
(B) Extension type
(C) Abduction type
(D) Adduction type

Answer: (B)

22. Complications of Colles’ fracture all except
(A) Malunion
(B) Non union
(C) Sudeck’s dystrophy
(D) Ruputre of extensor policis longus

Answer: (B)

23. The most common site of fracture neck of femur that causes avascular necrosis is
(A) Sub-capital
(B) Intertrochanteric
(C) Trans-cervical
(D) Basal

Answer: (A)

24. Treatment of choice in fracture neck of femur in a 40-year old male presenting after 2 days is
(A) Hemiarthroplasty
(B) Closed reduction and Internal fixation by cancellous screws
(C) Closed reduction and Internal fixation by Austin Moore pins
(D) Plaster and rest

Answer: (B)

25. Action of intramedullary ‘K’ nail is
(A) Two-point fixation
(B) Three-point fixation
(C) Compression
(D) Weight concentration

Answer: (B)

26. Which type of injury causes more damage to the semi-lunar cartilage in the Knee
(A) Flexion and extension at the ankle
(B) Rotation on a flexed knee
(C) Rotation on an extended knee
(D) Squatting position

Answer: (B)

27. Which of the following is not a benign bone tumor
(A) Osteoid osteoma
(B) Chondroma
(C) Enchondroma
(D) Chordoma

Answer: (B)

28. Most common nerve to be damaged in dislocation of shoulder is :
(A) Axillary nerve
(B) Radial nerve
(C) Median nerve
(D) Musculocutaneous nerve

Answer: (A)

29. Most common joint to undergo recurrent dislocation is :
(A) Shoulder joint
(B) Patella
(C) Knee joint
(D) Hip joint

Answer: (A)

30. Most common complication of colle’s fracture is :
(A) Stiffness of fingers
(B) Sudeck’s dystrophy
(C) Nonunion
(D) Tendon rupture

Answer: (A)

31. “Tennis elbow’, is characterized by :
(A) Tenderness over the medial epicondyle
(B) Tendinitis of common extensor origin
(C) Tendinitis of common flexor origin
(D) Painful flexion and extension

Answer: (B)

32. Open reduction & internal fixation is done for all of the following #s except :
(A) Patella #
(B) Olecranon #
(C) Von barton’s #
(D) # Lateral condyle of humerus

Answer: (C)

33. All of the following factors fascilitate non union except
(A) Haematoma formation
(B) Periosteal injuries
(C) Absence of nerve supply
(D) Chronic infection

Answer: (A)

34. Most common site of scaphoid fracture is :
(A) Waist
(B) Proximal fragment
(C) Distal fragment
(D) Tilting of the lunate

Answer: (A)

35. First symptom in tuberculous cord compression is :
(A) Sensory change
(B) Decrease tendon reflex
(C) Spasticity
(D) Lower limb weakness

Answer: (C)

36. Most common charcot’s joints involved in diabetes mellitus are those of:
(A) Shoulder
(B) Ankle
(C) Knee
(D) Foot

Answer: (D)

37. Characteristic crystals in pseudogout are :
(A) Calcium pyrophosphate
(B) Sodium monourate
(C) Potassium urate
(D) Sodium pyrophasphate

Answer: (A)

38. Complications of Colle’s fracture include all of the following except :
(A) Malunion
(B) Non Union
(C) Sudeck’s osteodystrophy
(D) Rupture of EPL tendon

Answer: (B)

39. Distal interphalangeal joint is not involved in :
(A) Rheumatoid arthritis
(B) Psoriatic arthritis
(C) Multicentric histeocytosis
(D) Neurophatic arthropathy

Answer: (A)

40. Ankle sprain is due to :
(A) Rupture of anterior talo-fibular ligament
(B) Rupture of posterior talo-fibular ligament
(C) Rupture of deltoid ligament
(D) Rupture of Calcaneo-fibular ligament

Answer: (A)

41. Most common complication of intertrochanteric fracture femur is :
(A) Malunion
(B) Nonunion
(C) Osteoarthritis
(D) Nerve injury

Answer: (A)

42. Commonest site for tuberculous spondylitis is :
(A) T12/L1
(B) C6-7
(C) S1-2
(D) None

Answer: (A)

43. All of the following tumor are benign tumour except :
(A) Chondroma
(B) Chordoma
(C) Osteochondroma
(D) Enchondroma

Answer: (B)

44. Which of the following is persistant biochemical marker of rickets :
(A) S.Ca⁺⁺
(B) S. Alkphosphate
(C) S. Acid phosphate
(D) S. phosphate

Answer: (B)

45. Biochemical marker in early gouty arthritis is :
(A) S. uric acid
(B) Increased urinary uric crystal
(C) Synovial fluid urate crystal
(D) Alkaline phosphate

Answer: (C)

46. Most common site for pseudotumour like growth in haemophilic arthropathy is :
(A) Quadriceps femoris
(B) Hamstring muscle
(C) Gastrocnemius
(D) Iliopsoas

Answer: (D)

47. Pseudoarthrosis may be seen in all of the following conditions except :
(A) Fracture
(B) Idiopathic
(C) Neurofibromatosis
(D) Osteomyelitis

Answer: (D)

48. All of the following are associated with chronic osteomyelitis except :
(A) Sequestrum
(B) Amyloidosis
(C) Myositis-ossificans
(D) Metastatic abscess

Answer: (C)

49. Most common site of Actinomycosis amongst the following is :
(A) Tibia
(B) Rib
(C) Mandible
(D) Femur

Answer: (C)

50. Defective mineralization of proximal zone of cartilar and epiphysis is a feature of :
(A) Rickerts
(B) Scurvy
(C) Osteomalacia
(D) Syphilis

Answer: (A)

51. Most common site of avascular necrosis amongst following is :
(A) Medial candyle of femur #
(B) Talus #
(C) Olecranon #
(D) Head of the radius #

Answer: (B)

52. Commonest site of fracture leading to fat-embolism are :
(A) Tibia #
(B) Femur #
(C) Humerus #
(D) Ulna #

Answer: (B)

53. Jefferson’s # is :
(A) Cl
(B) C₂
(C) C₂C₁
(D) C₂C₃

Answer: (C)

54. Osteoarthritis is associated with all of the following except :
(A) Decrease joint space
(B) Subchondral sclerosis
(C) Osteophyte formation
(D) Ca⁺⁺ deposition in joint space

Answer: (D)

55. According to a newer hypothesis Ewings sarcoma arises from :
(A) Epiphysis
(B) Diaphysis
(C) Medullary cavity
(D) Cortex

Answer: (C)

56. Most common nerve involvement in volkmann’s ischaemic contracture is :
(A) Median nerve
(B) Radial nerve
(C) Ulner nerve
(D) Posterior interosseous nerve

Answer: (A)

57. In posterior dislocation of hip, there is:
(A) Flexion, abduction, int. rotation
(B) Flexion, abduction, ext. rotation
(C) Flexion, adduction, int. rotation
(D) Flexion, adduction, ext. rotation

Answer: (A)

58. Bunion is:
(A) Exostosis
(B) Adventitious bursitis lateral to 1st metatarsal
(C) Base of 1st metatarsal
(D) Head of 1st metatarsal

Answer: (B)

59. Paronychia is defined as infection of:
(A) Pulp space
(B) Web
(C) Subcuticular space
(D) Folds of finger nail

Answer: (D)

60. Clergymen’s knee is due to involvement of:
(A) Prepatellar bursa
(B) Intrapatellar bursa
(C) Supraptellar bursa
(D) Infrapatellar bursa

Answer: (D)

61. Crutch palsy in axilla, nerve most often involved is
(A) Radial
(B) Ulnar
(C) Musculocutaneous
(D) Axillary

Answer: (A)

62. Most common site of osteoblastoma is:
(A) Epiphysis
(B) Metaphysis
(C) Diaphysis
(D) B or C

Answer: (C)

63. Anterior dislocation of shoulder is most commonly complicated by:
(A) Recurrent dislocation
(B) Circumflex nerve injury
(C) Axillary nerve injury
(D) Axillary artery injury

Answer: (C)

64. Syme’s amputation involves joint:
(A) Ankle
(B) Mid tarsal
(C) Tarsometatarsal
(D) All of the above

Answer: (A)

65. Bony ankylosis is caused by:
(A) Pyogenic arthritis
(B) TB
(C) Osteoarthritis
(D) Rheumatoid arthritis

Answer: (A)

66. In talipes equnovarus the foot is in position of:
(A) Equinus, eversion, abduction
(B) Equinus, inversion, abduction
(C) Equinus, inversion, adduction
(D) Equinus, eversion, abduction

Answer: (C)

67. Inversion and eversion occurs at which joint
(A) Subtalar/midtarsal
(B) Metatarsophalangeal
(C) Calcaneocuboid only
(D) Tibiofibular

Answer: (A)

68. Most common muscle involved in lower leg in polio is:
(A) Ant. tibialis
(B) Post. tibialis
(C) Extensor pollicis longus
(D) Flexor pollicis longus

Answer: (A)

69. In cervical rib following are seen except
(A) Cervical rib palpable
(B) Ischaemic pain of muscles
(C) Atrophy of muscles
(D) Radial pulse not palpable

Answer: (A)

70. Most common complication of supracondylar #:
(A) Malunion with gun stock deformity
(B) Volkman’s ischaemicc contracture
(C) Blood vessel injury
(D) Genu valgum

Answer: (A)

71. Complication of acetabular fracture is:
(A) Avascular necrosis of head of femur
(B) Sciatic N injury
(C) Secondary osteoarthritis of hip joint
(D) Fixed deformity of hip joint

Answer: (C)

72. Least correction in remodeling of bone in children is seen in:
(A) Fracture shaft of femur
(B) Fracture shaft of humerus
(C) Fracture above lateral condyle
(D) Subtrochanteric fracture

Answer: (B)

73. Galeazzi fracture is:
(A) Lower 1/3rd # radius with radioulnar dislocation
(B) Upper 1/3rd # ulna with inf. radioulnar dislocation
(C) Lower 1/3rd # ulna without radioulnar dislocation
(D) Upper 1/3rd # ulna with sup. radioulnar dislocation

Answer: (C)

74. Von rosen splint is used in:
(A) CDH
(B) CTEV
(C) Fracture shaft of femur
(D) Fracture tibia

Answer: (A)

75. Burlow’s sign is seen in:
(A) CDH
(B) Fracture neck femur
(C) Perthes disease
(D) Polio

Answer: (A)

76. Psoas spasm is seen in following except:
(A) TB hip
(B) Ac. appednicitis
(C) TB spine
(D) Mesentric iliac lymphadenitis

Answer: (A)

77. Most common organism causing osteomyelitis in drug abusers:
(A) Staph. aureus
(B) Klebsiella
(C) E.coli
(D) Pseudomonas

Answer: (A)

78. Rheumatoid arthritis most commonly causes:
(A) Pericarditis
(B) Endocarditis
(C) Myocarditis
(D) Pancarditis

Answer: (A)

79. Enthesopathy is characteristic of:
(A) OA
(B) TB arthritis
(C) RA
(D) Ankylosing spondylitis

Answer: (D)

80. True about giant cell tumour is
(A) Never recurs after excision
(B) Periarticular epiphyseal
(C) Malignant potential is not predictable
(D) All of the above

Answer: (B)

81. Vitamin required for collagen synthesis is:
(A) Vitamin C
(B) Vitamin E
(C) Vitamin A
(D) Vitamin D

Answer: (A)

82. Trigger finger occurs in :
(A) Rheumatoid arthritis
(B) Trauma
(C) Osteoarthritis
(D) Contracture

Answer: (A)

83. Not attached to pisiform bone :
(A) Flexor retinaculum
(B) Extensor carpi ulnris
(C) Flexor carpi ulnaris
(D) Pisohamate ligament

Answer: (B)

84. Definitive diagnosis for blow out # of orbit is by :
(A) CSF rhinorrhoea
(B) Loss of vision
(C) Tear drop sign
(D) Fracture of sphenoid sinus

Answer: (C)

85. Albert Schonberg disease is also called as :
(A) Osteoporosis
(B) Osteodystrophy
(C) Osteopetrosis
(D) Osteitis punctata

Answer: (C)

86. Caffey’s disease is :
(A) Chronic osteomyelitis is children
(B) Osteomyelitis of jaw in children
(C) Infantile cortical hyperostosis
(D) Renal osteodystrophy

Answer: (C)

87. Chaissagne tubercle is related to :
(A) Cervical vertebra
(B) Digastric tendon
(C) Carotid sinus
(D) Stellate ganglion

Answer: (A)

88. X-ray appearance of malignancy are following except :
(A) Sclerotic margins
(B) Soft tissue mass
(C) Periosteal elelvation
(D) Cortical destruction

Answer: (B)

89. Dupuytren’s contracture is seen in :
(A) Supracondylar # humerus
(B) Colle’s fracture
(C) Thickening of palmar fascia
(D) Radial nerve palsy

Answer: (C)

90. Increase in Limb length is not found in :
(A) Neurofibromatosis
(B) Salter harris #
(C) Haemophilia
(D) Rh. arthritis of juvenile type

Answer: (B)

91. Fasciectomy is indicated when intracompartmental pressure rises above mm Hg :
(A) 10
(B) 60
(C) 45
(D) 100

Answer: (C)

92. Difficult to diagnose salter harris type by X-ray :
(A) V
(B) I
(C) IV
(D) III

Answer: (A)

93. Median nerve injury at wrist causes :
(A) Loss of opposition of thumb
(B) Claw hand
(C) Saturday night palsy
(D) Policemans tip deformity

Answer: (A)

94. About sequestrum all are true except :
(A) Dead piece of bone
(B) Lighter than live bone
(C) Heavier than live bone and trabeculated
(D) Surround an infected nidus

Answer: (C)

95. Backahe, radiating to groin, along lateral part of thigh, front of let and dorsum of foot is mostly due to disc prolapsed at :
(A) L₄ L₅
(B) S₁- S₂
(C) L₅, S₁
(D) L₂ L₃

Answer: (A)

96. Not sensitive to pain :
(A) Synovial membrane
(B) Atricular cartilage
(C) Muscle
(D) Periosteum

Answer: (B)

97. Osteogenesis imperfect is defect in :
(A) Calcification
(B) Bone
(C) Collagen
(D) Cartilage

Answer: (C)

98. Crystals found in pseudogout are of :
(A) Sodium urate
(B) Pyrophosphate
(C) Potassium urate
(D) Oxalate

Answer: (B)

99. Commonest cervical vertebral fracture of :
(A) C₃
(B) C₄
(C) C₂
(D) C₅

Answer: (D)

100. Fatigue # does not occur in :
(A) Metatarsal
(B) Metacarpal
(C) Calcaneum
(D) Tibia

Answer: (B)

101. X-ray in atlanto-axial dislocation should be taken in which position :
(A) Flexion
(B) Extension
(C) Neutral
(D) Open mouth view

Answer: (D)

102. Crystals in synovial fluid are seen in :
(A) Pseudogout
(B) Diabetes mellitus
(C) Syphilis
(D) Osteoarthritis

Answer: (A)

103. Pain along hip, back of thigh and loss of sensation along the lateral b order of foot and decreased ankle jerk, site of lesion is :
(A) L₄ -5
(B) L₅-S₁
(C) L₃
(D) L₁-T₁₂

Answer: (B)

104. Stress # is most often seen in :
(A) 2nd/3rd metalarsal bone
(B) Talus
(C) Calcaneum
(D) Metacarpals

Answer: (A)

105. Trigger finger is caused by:
(A) OA
(B) Rheumatoid arthritis
(C) Tenosynovitis
(D) Injury to tendons

Answer: (B)

106. Treatment for CTEV is started at:
(A) 6 months
(B) Immediately after birth
(C) 1 year
(D) At puberty

Answer: (B)

107. Commonest dislocation of shoulder is:
(A) Subcoracoid
(B) Subacromial
(C) Supraacromial
(D) Posterior

Answer: (A)

108. Gun stock deformity is seen in fracture of:
(A) Lateral condyle
(B) Humerus, lower end
(C) Radius, lower end
(D) Ulna, upper end

Answer: (B)

109. Onion peel appearance on X-ray is seen in:
(A) Giant cell tumour
(B) Ewing’s sarcoma
(C) Osteosarcoma
(D) Fibrosarcoma

Answer: (B)

110. Carpal tunnel syndrome, all are seen except :
(A) Loss of sensation over middle finger
(B) Opponens pollicis atrophy
(C) Flexor pollicis Brevis atrophy
(D) Adductor policies atrophy

Answer: (B)

111. Von-Rosen’s splint is used in :
(A) CDH
(B) CTEV
(C) # shaft of femur
(D) Scoliosis

Answer: (A)

112. Blue sclera and multiple # are seen in :
(A) Osteoporosis
(B) Osteopetrosis
(C) Osteogenesis imperfecta
(D) Osteosclerosis

Answer: (C)

113. Trident hand is seen in:
(A) Mucopolysaccharidosis
(B) Achondroplasia
(C) Diaphyseal achlasia
(D) Chondrodysplasia

Answer: (B)

114. In young man cause of backache and asymmetrical lower limb weakness :
(A) Seronegative spondyloarthropathy
(B) RA
(C) OA
(D) Gout

Answer: (A)

115. Weakest part of mandible is :
(A) Subcondylar part
(B) Coronoid process
(C) Angle
(D) Neck of condyle

Answer: (D)

116. Blount’s disease :
(A) Tibia vara
(B) Renal osteodystrophy
(C) Genu valgum
(D) Genu vara

Answer: (D)

117. Pulled elbow is :
(A) Subluxation of radius head
(B) Subluxation of wrist
(C) Subluxation of humerus
(D) Fracture ulna

Answer: (A)

118. Barlow’s test is used in :
(A) CDH
(B) Coxa vara
(C) Bow legs
(D) Knock knee

Answer: (A)

119. The normal angle of the acetabulum is :
(A) 5-7
(B) 30-32
(C) 35-37
(D) 50-52

Answer: (B)

120. Tom-Smith arthritis spreads to the hip joint because :
(A) Metaphysis is inside the joint
(B) Epiphysis is absent
(C) Periosteum lacks a cambium layer
(D) All of the above

Answer: (A)

121. The most radiosensitive part of the bone is :
(A) Epiphysis
(B) Osteoblastic layer
(C) Growing cartilage cells
(D) Fibroblasts

Answer: (A)

122. Froments sign is used to test :
(A) Adductor pollicis
(B) Flexor pollicis longus
(C) Flexor pollicis brevis
(D) Flexor digitorum profundus

Answer: (A)

123. The classical position adopted in posterior dislocation of the hip is :
(A) Flexion, adduction and internal rotation
(B) Flexion, abduction and external rotation
(C) Flexion, abduction and internal rotation
(D) Extension, adduction and internal rotation

Answer: (A)

124. Patella is at a higher level in :
(A) Recurrent dislocation
(B) Nail-patella syndrome
(C) Rheumatoid arthritis
(D) Plica syndrome

Answer: (A)

125. Progressive stiffening of a joint is seen in :
(A) Peri-arthritis of shoulder
(B) Osteochondritis
(C) Gout
(D) Ankylosis

Answer: (A)

126. Which among the following should not be given by acute gout :
(A) Acetyl salicylic acid
(B) Indomethacin
(C) Piroxicam
(D) Allopurinol

Answer: (D)

127. In a child with congential hip dislocation, the legs are abducted with maintained traction till a click is heard. This test is known is :
(A) Barlow’s test
(B) Ortolani’s lest
(C) Telescopy
(D) Trendelenburg’s sign

Answer: (B)

128. Spina ventosa is caused by :
(A) Tuberculosis
(B) Leprosy
(C) Metastasis
(D) Spine deformity

Answer: (A)

129. The muscle most commonly affected in polio is :
(A) Tensor fasica lata
(B) Tibialis anterior
(C) Tibialis posterior
(D) Quadriceps

Answer: (D)

130. The etiology of tom-smith arthritis is :
(A) Pyogenic
(B) Rheumatoid
(C) Fungal
(D) Syphilis

Answer: (A)

131. Trendelenburg sign is seen in palsy of :
(A) Superior gluteal nerve
(B) IIiohypogastric nerve
(C) Obturator nerve
(D) None of the above

Answer: (A)

132. True regarding mandibular fracture is :
(A) Some infection is harmless
(B) Aim is to get good occlusion
(C) Open reduction is better
(D) Always do wiring

Answer: (B)

133. Ring sign is seen in :
(A) Osteosarcoma
(B) Osteoclastoma
(C) Scurvy
(D) Barlow’s disease

Answer: (C)

134. Equinus deformity in poliomyelitis is due to weakness of :
(A) Dorsiflexors
(B) Plantar flexors
(C) Evertors
(D) Invertors

Answer: (A)

135. True about dupuytren’s contracture is :
(A) More common in females
(B) Palmar nodule is the earliest sign
(C) M.C. in orientals
(D) Contracture of dermal tissue

Answer: (B)

136. Bony ankylosis of hip is seen in :
(A) Pyogenic arthritis
(B) Tuberculosis
(C) Osteoarthritis
(D) Fracture neck femur

Answer: (A)

137. MC cause of genu valgum is :
(A) Trauma
(B) Rickets
(C) Polio
(D) Scurvy

Answer: (A)

138. Trophic ulcer is seen in A/E:
(A) Polio
(B) CNSTB
(C) Spinal dysraphism
(D) Leprosy

Answer: (A)

139. Osteomalacia is characterized by:
(A) Compression vertebral
(B) Cod fish vertebra
(C) Looser’s zone
(D) Bamboo spine

Answer: (C)

140. Treatment of fracture patella in 24 year old young male is :
(A) Patellectomy if undisplaced #
(B) No treatment required
(C) Internal fixation if communited fracture
(D) POP in full extension

Answer: (D)

141. Pointing finger is seen in :
(A) Ulnar N palsy
(B) Median N palsy
(C) Ulnar and median N palsy
(D) Radial N pasly

Answer: (B)

142. All are common with elbow dislocation except :
(A) Myositis ossificans progressive
(B) Median N palsy
(C) Brachial artery injury
(D) Volkmann’s contracture

Answer: (A)

143. Club foot in a 2 year old child is best treated by:
(A) Soft tissue release
(B) Tripple arthodesis
(C) Dennis brown splint
(D) Manipulation by mother and cast

Answer: (A)

144. Non union in long bones is treated by:
(A) Intramedullary nailing
(B) POP cast
(C) Open reduction and internal fixation with bones grafting
(D) Conservation Tt

Answer: (C)

145. Onion peel appearance is seen in :
(A) Osteosarcoma
(B) Ewings sarcoma
(C) Osteoclastoma
(D) Osteochondroma

Answer: (B)

146. Commonest tumour of mandible is :
(A) Ameloblastoma
(B) Osteosarcoma
(C) Squamous cell CA
(D) All

Answer: (A)

147. Soap bubble appearance is the radiological feature of :
(A) Osteoid osteoma
(B) Osteosarcoma
(C) Osteoclastoma
(D) Chrodoma

Answer: (C)

148. True about Osteoclast is all except :
(A) Derived from monocytes
(B) Stimulated by PTH
(C) Phagocytosis of foreign bodies
(D) Resorption of bone

Answer: (C)

149. Albers Schonberg disease is :
(A) Osteopetrosis
(B) Osteoporosis
(C) Osteochondritis
(D) Osteomalacia

Answer: (A)

150. In traumatic myositis ossificans, following are true except :
(A) Hydroxyapatite deposition
(B) Common in elbow injury
(C) Periosteal hematoma & leakage
(D) Injury of tendon over muscle

Answer: (D)

151. Commonest site of march # is :
(A) Involves shaft of 2 & 3rd metatarsals
(B) Avulsion # of 5th metatarsals
(C) Calcaneus involved
(D) Olecranon involved

Answer: (A)

152. Blount’s disease is :
(A) Genu valgus
(B) Tibia vera
(C) Flat foot
(D) Genu recurvatum

Answer: (B)

153. Pathological # is seen in following except :
(A) Radiation
(B) Anaemia
(C) Osteoporosis
(D) Osteomalacia

Answer: (B)

154. True about TB spine is all except :
(A) Early paraplegia is good prognosis
(B) Insidious onset paraplegia is good prognosis
(C) Dorsolumbar spine is commonest site
(D) It is commonest site for TB of bone in the body

Answer: (B)

155. Most common soft tissue tumour in a child :
(A) Rhabdomyosarcoma
(B) Histiocytoma
(C) Fibrosarcoma
(D) Liposarcoma

Answer: (A)

156. How to differentiate gout with pseudogout :
(A) Large joint involvement
(B) Birefringent (Particles) crystals
(C) Serum uric acid normal
(D) Associated with hyperparathyroidism

Answer: (B)

157. All are relevant in compartment syndrome except :
(A) Fasciotomy
(B) Splitting of tight pop cast
(C) Reexploration
(D) Exercise

Answer: (D)

158. Shephared crooke’s deformity is seen is :
(A) Achondroplasia
(B) Gaucher’s disease
(C) Hypothyroidism
(D) Fibrous dysplasia

Answer: (D)

159. Giant cell tumor is treated with :
(A) Resection of tumor part
(B) Curetage & bone graft
(C) Radiotherapy
(D) Melphalan

Answer: (B)

160. Which is/are not true tumor :
(A) Fibrosarcoma
(B) Osteochondroma
(C) Osteoclastoma
(D) Chondrosarcoma

Answer: (B)

161. Iliac horn’ is seen in :
(A) Achondroplasia
(B) Muco-polysaccharodosis
(C) Nail patell syndrome
(D) Tuberous sclerosis

Answer: (C)

162. Components of Gardner’s synd. a/e :
(A) Intestinal polyps
(B) Osteomas
(C) Sebaceous cyst
(D) Neuromas

Answer: (D)

163. Treatment of # with intraarticular extension is/are A/E :
(A) Excision of the joint
(B) Arthrodesis
(C) Traction
(D) Intramedullary nailing

Answer: (D)

164. Mallet finger is due to avulsion of extensor tendon of :
(A) Proximal phalanx
(B) Middle phalanx
(C) Distal phalanx
(D) None of these

Answer: (C)

165. Osgood Schlatters disease affects :
(A) Upper tibia
(B) Lower tibia
(C) Distal femur
(D) Proximal femur

Answer: (A)

166. CTEV is caused by all except :
(A) Neurological disorder
(B) Idiopathic
(C) Spina bifida
(D) Cubitus varus

Answer: (D)

167. Osteogenic sarcoma arise from :
(A) Epiphysis
(B) Metaphysis
(C) Growth plate
(D) Epiphyseal cortex

Answer: (B)

168. In traumatic injury of the hip :
(A) Anterior dislocation is common
(B) Posterior dislocation is common
(C) Avascular fracture is common
(D) Visceral injury usually associated with fracture femur

Answer: (B)

169. Absent lateral 1/3rd of clavicle is seen in :
(A) Hyperparathyroidism
(B) Turner’s syndrome
(C) Fibrous
(D) Cleidocranial dysostosis

Answer: (D)

170. Tubercular bursitis involve :
(A) Prepatellar bursa
(B) Pretibial bursa
(C) Olecranon bursa
(D) Subdeltoid bursa

Answer: (A)

171. True about Osteoclast is all except:
(A) Derived from monocytes
(B) Stimulated by PTH
(C) Phagocytosis of foreign bodies
(D) Resorption of bone

Answer: (B, C)

172. True about Osteoclast is all except:
(A) Derived from monocytes
(B) Stimulated by PTH
(C) Phagocytosis of foreign bodies
(D) Resorption of bone

Answer: (C, B)

173. Major mineral of the bone is:
(A) Calcium chloride
(B) Hydroxyapatite
(C) Calcium oxide
(D) Calcium carbonate

Answer: (B)

174. One of the features given below is essential in the diagnosis of fracture of a bone
(A) Deformity
(B) A crepitus
(C) A partial or complete loss of continuity of the bone
(D) None of the above

Answer: (C)

175. The one most consistent sign of fresh fracture is
(A) Crepitus
(B) Bony tenderness
(C) Deformity
(D) Abnormal mobility
(E) Shortening of bone

Answer: (A)

176. Pathognomonic sign of traumatic fracture is
(A) Redness
(B) Swelling
(C) Crepitus
(D) Tenderness

Answer: (C)

177. Direct impact on the bone will produce a:
(A) Transvers fracture
(B) Oblique fracture
(C) Spiral fracture
(D) Communited fracture

Answer: (A)

178. Pathologic fracture can occur in all except:
(A) Metabolic bone disease
(B) Osteosarcoma
(C) Osteochondroma
(D) Bone cyst
(E) Fluorosis

Answer: (C)

179. Pathological # is seen in following except:
(A) Radiation
(B) Anaemia
(C) Osteoporosis
(D) Osteomalacia

Answer: (B)

180. The commnonest cause of pathological fracture is generalized affection is:
(A) Carcinoma
(B) Osteoporosis
(C) Cyst
(D) All of the above

Answer: (B)

181. The most common cause of pathological fracture is
(A) Delayed union
(B) Mal union
(C) Non union
(D) Secondary deposits
(E) Any of thee

Answer: (D)

182. The treatment of choice in pathological fractures is
(A) Internal fixation
(B) Plaster of Paris casts
(C) Skin traction
(D) External skeletal fixation

Answer: (A)

183. Mirel’s criteria is developed for the evaluation of
(A) Risk of fatigue fracture
(B) Severity of osteoporosis
(C) Risk of pathological fracture
(D) Severity of neurological defecit

Answer: (C)

184. An army recruit, smoker and 6 months into training started complaining of pain at postero medial aspect of both legs. There was acute point tenderness and the pain was aggravated on physical activity. the most likely diagnosis is
(A) Bearger’s disease
(B) Gout
(C) Lumbar canal stenosis
(D) Stress fracture

Answer: (D)

185. Commonest site f march # is:
(A) Involves shaft of 2% 3rd metatarsals
(B) Avulsion # of 5th metatarsals
(C) Calcaneus involved
(D) Olecranon involved

Answer: (A)

186. Stress fracture is treated by:
(A) Rest
(B) Cast immobilization
(C) Closed reduction
(D) Internal fixation

Answer: (B)

187. What is March fracture?
(A) Fracture of 2nd metatarsal
(B) Fracture of 4th metatarsal
(C) Fracture of cuboids
(D) Fracture of tibia

Answer: (A)

188. The usual site of stress fracture includes
(A) Tibia
(B) First metacarpal bone
(C) Second metacarpal bone
(D) Second metatarsal bone

Answer: (D)

189. Stress fracture of involves:
(A) Metatarsals
(B) Metacarpals
(C) Tibia
(D) Calcanium

Answer: (B)

190. Fatigue fractures (Stress fractures) are most commonly seen in:
(A) Metatarsals
(B) Tibia
(C) Fibula
(D) Neck of femur

Answer: (A)

191. All of the following factors facilitate non-union except:
(A) Haematoma formation
(B) Periosteal injuries
(C) Absence of nerve supply
(D) Chronic infection

Answer: (A)

192. Last step in fracture healing is:
(A) Haematoma
(B) Callus formation
(C) Remodeling
(D) Consolidation

Answer: (C)

193. Initial stage of clinical union of bone is equivalent to
(A) Callus formation
(B) Woven bone
(C) Haematoma formation
(D) Calcification only
(E) None of the above

Answer: (B)

194. The time necessary for healing of fracture depends on the following factors:
(A) Age of the patient
(B) Location of the fracture
(C) Type of the fracture
(D) Degree of damage to soft tissues
(E) All of the above

Answer: (E)

195. The most important factor in fracture healing is:
(A) Good alignment
(B) Organization of blood clot
(C) Accurate reduction and 100% apposition of fractured fragments
(D) Immobilization
(E) Adequate calcium intake

Answer: (D)

196. The most common cause of non union is
(A) Infection
(B) Inadequate immobilization
(C) Ischaemia
(D) Soft tissue interposition

Answer: (B)

197. Delayed union of fracture of a bone following a surgical treatment may be due to
(A) Infection
(B) Inadequate circulation
(C) Inadequate mobilization
(D) Soft tissue interposition

Answer: (D)

198. Bone apposition is best in
(A) Osteoblastic activity at the area of stress
(B) Endochondral ossification
(C) Subperiosteal cambian layer
(D) Osteobalstic activity in howship’s lancunae

Answer: (D)

199. Regarding bone remodeling, all are true EXCEPT:
(A) Osteoclastic activity at the compression site
(B) Osteoclastic activity at the tension site
(C) Osteoclastic activity and osteoblastic activity are both need for bone remodeling in cortical and cancellous bones.
(D) Osteoblasts transforms into osteocytes

Answer: (A)

200. Which is not a marker of new bone formation?
(A) Alkaline phosphatase
(B) Osteocalcin
(C) Urine hydroxyproline
(D) Pro collagen

Answer: (C)

201. Bone resorption markers are:
(A) Serum propeptide of type I procollagen
(B) Osteocalcin
(C) Urine total free deoxypyridinoline
(D) Free glutamic acid cross linkage.

Answer: (C)

202. Bone formation markers:
(A) Procollagen-I
(B) Alkaline phosphatase
(C) Hydroxy appetite
(D) Calcitonin
(E) TRAP

Answer: (A, B)

203. Marker for bone formation is:
(A) Tartrate resistant acid phosphate
(B) Osteocalcin
(C) Urinary calcium
(D) Serum nucleotidase

Answer: (B)

204. Indicators of bone formation includes all of following except-
(A) Osteocalcin
(B) Alkaline phosphatase
(C) Hydroxyprolin
(D) Bone scan

Answer: (C)

205. For growth of bone, which term is most appropriate:
(A) Enlargement
(B) Apposition
(C) Hyper plasia
(D) Hypertrophy

Answer: (B)

206. Rate of mineralization of newly formed osteoid can be estimated by the following:
(A) Von Kossa staining for calcium
(B) Alzarin red stain
(C) Labeled tetracycline
(D) Immunoflurescence Calcein Stain

Answer: (C)

207. Callus induction is hampered in:
(A) Hypoxemia
(B) Fractured fragments of bone
(C) Micromovements
(D) Muscle interposition in between fractured fragments
(E) Early mobilization

Answer: (A, B, D)

208. Healing of # of bone is affected by:
(A) Micromovements
(B) Muscle interposition
(C) Hypoxia
(D) Bone fragments

Answer: (ALL)

209. Factors that promotes callus formation:
(A) Micro movements between the fracture fragments
(B) Appropriate approximation of the fragments
(C) Muscle interposed in between the fracture fragment
(D) Early initiation of mobilization
(E) Ischemia

Answer: (A, B)

210. Non – union is a complication of :
(A) Scaphoid #
(B) Colle’s #
(C) Inter – trochanteric # of hip
(D) Supra condylar # of the humerus

Answer: (A)

211. In some old fractures, cartilaginous tissue forms over the fractured bone ends with a cavity in between containing clear fluid. This condition is called as:
(A) Delayed union
(B) Slow union
(C) Non union
(D) Pseudoarthrosis

Answer: (D)

212. Pseudoarthrosis may be seen in all of the following conditions except:
(A) Fracture
(B) Idiopathic
(C) Neurofirbomatosis
(D) Osteomyelitis

Answer: (D)

213. The correct order of priorities in the initial management of head injury is
(A) Airway, Breathing, Circulation treatment of extra cranial injuries
(B) Treatment of extra cranial injuries Airway, Breathing, Circulation
(C) Circulation airway, Breathing treatment of extra cranial injuries
(D) Airway, circulation, breathing treatment of extra cranial injuries

Answer: (A)

214. In an injury with multiple fractures, most important is
(A) Airway maintenance
(B) Blood transfusion
(C) Intravenous fluids
(D) Open reduction of fractures

Answer: (A)

215. Patients comes with fracture femur in an acute accident, the first things to do is
(A) Secure airway and treat the shock
(B) Splinting
(C) Physical examination
(D) X-rays

Answer: (A)

216. Consider the following sign(s)
(1) Increasing pallor
(2) Restlessness
(3) Air hunger
(4) Water-hammer pulse
Haemorrhagic shock due to acute blood loss includes
(A) 1 & 4
(B) 1& 2
(C) 1, 2 & 3
(D) 2, 3 & 4

Answer: (C)

217. Polytrauma patient with open bleeding wound of right thigh should be managed first of all by
(A) Tourniquet application
(B) Tight bandage application
(C) Airway maintenance
(D) Blood transfusion

Answer: (C)

218. Severely injured patient with spinal fracture and unconsciousness first thing to be done is
(A) GCS scoring
(B) Spinal stabilization by cervical collar
(C) Mannitol drip to decrease ICT
(D) Airway maintenance

Answer: (D)

219. Which of the following is not a component of the crust syndrome.
(A) Myohemoglobinuria
(B) Massive crushing of muscles
(C) Acute tubular necrosis
(D) Bleeding diathesis

Answer: (D)

220. Crush syndrome is managed by
(A) 20% Dextrose
(B) Hydrocortisone
(C) Maintaining high urine output
(D) Acidification of urine

Answer: (C)

221. Compound fracture is
(A) Fracture with artery involvement
(B) Fracture with nerve involvement
(C) Fracture with muscle involvement
(D) Fracture with skin involvement

Answer: (D)

222. A patient presents with compound fracture of Tibia with 1 cm opening in skin. Which grade it belongs?
(A) Grade I
(B) Grade II
(C) Grade IIIA
(D) Grade IIIB

Answer: (A)

223. Tibial Fracture with > 1 cm wound, slight comminution and moderate crushing is
(A) Grade I
(B) Grade II
(C) Grade IIIA
(D) Grade IIIB

Answer: (B)

224. Following are principles in the treatment of compound fractures except
(A) Wound debridement
(B) Immediate wound closure
(C) Tendon repair
(D) Aggressive antibiotic therapy

Answer: (B)

225. Immediate treatment of compound fracture of tibia is
(A) Intravenous antibiotics
(B) Thorough debridement
(C) Internal fixation of fracture
(D) Amputation of limb

Answer: (B)

226. Open fracture is treated by
(A) Tourniquet
(B) Internal fixation
(C) Debridement
(D) External fixation

Answer: (C)

227. Internal splints (fixation devices) are used in all except
(A) Compound fractures
(B) Multiple fractures
(C) Fractures in elderly patients
(D) Fracture neck of femur

Answer: (A)

228. A compound fracture is initially treated by antibiotics, wound toilet and
(A) Skin over
(B) External splintage
(C) Prosthesis
(D) Internal fixation

Answer: (B)

229. Which of the following is the most appropriate hospital treatment of a patient with compound fracture?
(A) Under anesthesia, thorough scrubbing and cleaning of the area getting the fracture end inside, suturing the wound and applying continuous skeletal traction with adequate antibiotic cover
(B) Cleaning and suturing the wound, applying plaster Spica under traction on a Harly’s table and administering antibiotics round the clock
(C) Scrubbing and cleaning the area, resecting the protruding one inch of the bone, suturing the wound, bringing the fractured ends into alignment and applying plaster Spica with continuous antibiotic cover
(D) Thorough cleaning of the area, extending the wound, bringing the fragments into alignment under vision, fixing them with intra-medullary nail and giving antibiotics to the patient.

Answer: (A)

230. In shotgun injuries
(A) Each and every shot should be removed
(B) All the shots within accessible limits may be removed and thorough debridement of the tissue done
(C) Shots lodged in joints must be removed
(D) All the above are true

Answer: (B)

231. Tetanus is noticed usually in
(A) Burn cases
(B) Wounds contaminated with faecal matter
(C) Open fractures
(D) Gunshot wounds
(E) All of the above

Answer: (B)

232. Which of the following score evaluates chances of amputation in a traumatized limb:
(A) Revised trauma score
(B) Injury severity score
(C) Abbreviated injury score
(D) MES score

Answer: (D)

233. All of the following factors evaluate the chances of amputation in a limb, except
(A) Age
(B) B.P
(C) Velocity of trauma
(D) Presence of infection

Answer: (D)

234. Tarsometatarsal amputation is also known as
(A) Chopart’s amputation
(B) Lisfranc amputation
(C) Pirogoff amputation
(D) Symes amputation

Answer: (B)

235. Amputation is often not required in:
(A) Gas gangrene
(B) Buerger’s
(C) Chronic osteomyelitis
(D) Diabetic gangrene

Answer: (C)

236. In below elbow amputation the length of stump should be
(A) 10-15 cm
(B) 15-20 cm
(C) 20-25 cm
(D) 5-10 cm

Answer: (B)

237. Distance from Olecranon in amputation should be
(A) 5-10 cm
(B) 10-25 cm
(C) 15-20 cm
(D) 20-30 cm

Answer: (C)

238. In flap method of amputation which structure is kept shorter than the level of amputation:
(A) Bone
(B) Muscles
(C) Nerves
(D) Skin
(E) Vessels

Answer: (A)

239. Ring sequestrum is seen in
(A) Typhoid osteomyelitis
(B) Chronic osteomyelitis
(C) Amputation stump
(D) Tuberculosis osteomyelitis

Answer: (C)

240. Which of the following is true regarding a phantom limb:
(A) Occurs in leprosy
(B) Follows amputation
(C) Follows a psychiatric illness
(D) After filariasis

Answer: (B)

241. Myodesis is employed in amputations for all of the following indications except:
(A) Trauma
(B) Tumor
(C) Children
(D) Ischemia

Answer: (D)

242. Pain due to post-amputation neuroma is best treated by:
(A) Infrared therapy
(B) Interference therapy
(C) Ultrasound therapy
(D) Surgical Excision

Answer: (D)

243. All of the following statements about SACH feet are true, except:
(A) ‘SACH’ stands for ‘Solid Ankle Cushioned Heal’
(B) Forms the base of a lower limb prosthesis
(C) May wear out with time
(D) Wooden keel absorbs the impact of heel strike

Answer: (D)

244. Which of the following is true of shoulder joint?
(A) Composed of only 2 joints
(B) Anterior posterior gliding of a scapula of never occurs
(C) Acromioclavicular joint is more important
(D) Allows flexion, rotation and abduction

Answer: (D)

245. The Rotator cuff is composed of four of the following muscles except:
(A) Teres minor
(B) Supraspinatus
(C) Infraspinatus
(D) Teres major
(E) Subscapularis

Answer: (D)

246. Weakest portion of shoulder joint capsule is:
(A) Anterior
(B) Posterior
(C) Inferior
(D) Superior

Answer: (C)

247. Muscle crossing through the shoulder joint is:
(A) Biceps short head
(B) Biceps long head
(C) Triceps long head
(D) Coracobrachialis

Answer: (B)

248. Rotator interval is between:
(A) Supraspinatus &teresmonor
(B) Teres major &subscapularis
(C) Supraspinatus &subscapularis
(D) Subscapularis&infraspinatus

Answer: (C)

249. Luxatioerecta
(A) Tear of the glenoidal labium
(B) Inferior dislocation of shoulder
(C) Anterior dislocation of shoulder
(D) Defect in the humeral head

Answer: (B)

250. Recurrent dislocation are common in:
(A) Shoulder
(B) Patella
(C) Hip joint
(D) Elbow joint

Answer: (A)

251. Most common joint to undergo recurrent dislocation is:
(A) Shoulder joint
(B) Patella
(C) Knee joint
(D) Hip joint

Answer: (A)

252. Commonest type of shoulder dislocation:
(A) Subcoracoid
(B) Subglenoid
(C) Posterior
(D) Subclavicular

Answer: (A)

253. Recurrent dislocation is least common in
(A) Shoulder
(B) Knee
(C) Patella
(D) None

Answer: (B)

254. Recurrent dislocations are least commonly seen in:
(A) Ankle
(B) Hip
(C) Shoulder
(D) Patella

Answer: (A)

255. Attitude in subcoracoid dislocation of shoulder includes :
(A) Adduction
(B) Limb on side of body
(C) Abduction
(D) Elevation

Answer: (B)

256. The position of arm in anterior dislocation of shoulder is:
(A) By the side
(B) In abduction
(C) In adduction
(D) In external rotation

Answer: (A)

257. Bankart’s lesion is seen at
(A) Post surface of glenoid labrum
(B) Ant surface of glenoid labrum
(C) Ant part of head of humerus
(D) Post part of head of humerus

Answer: (B)

258. Bankart’s lesion involves the ___ of the glenoid labrum.
(A) Anterior lip
(B) Superior lip
(C) Antero-superior lip
(D) Antero-inferior lip

Answer: (D)

259. Recurrent dislocation of shoulder occurs, because of
(A) Incomplete labrum
(B) Superadded secondary infection
(C) Crushed glenoidal labrum
(D) Weak posterior capsule

Answer: (C)

260. Bankart’s lesion involves
(A) Anterior aspect of the head of humerus
(B) Anterior aspect of glenoid labrum
(C) Posterior aspect of glenoid labrum
(D) Posterior aspect of head of humerus

Answer: (B)

261. Hill-sachs lesion is associated with
(A) Recurrent dislocation of shoulder
(B) Recurrent dislocation of hip
(C) Perthe’s disease
(D) Fracture neck of femur

Answer: (A)

262. Hill-sachs lesion in recurrent shoulder dislocation is
(A) Injury to humeral head
(B) Rupture of tendon of supraspinatus muscle
(C) Avulsion of glenoid labrum
(D) None of the above

Answer: (A)

263. Bankart’s lesion is seen in
(A) Anterior border of head of humerus
(B) Posterior border of head of humerus
(C) Anterior glonoid cavity
(D) Posterior glenoid cavity

Answer: (C)

264. All are related to recurrent shoulder dislocation except:
(A) Hill sachs defect
(B) Bankart lesion
(C) Lax capsule
(D) Rotator cuff injury

Answer: (D)

265. The lesion associated with recurrent dislocation of shoulder include all, except:
(A) Hill-Sach’s lesion
(B) Bankart’s lesion
(C) Capsular laxity
(D) Supraspinatous tear

Answer: (D)

266. Recurrent dislocation is most common in the shoulder joint. Which one of the following is not an important cause for the same?
(A) Tear of the anterior capsule of the shoulder
(B) Associated fracture neck of the humerus
(C) Tear of the glenoid labrum
(D) Freedom of mobility in the shoulder

Answer: (B)

267. Which nerve is damaged in ant dislocation of shoulder :
(A) Axillary
(B) Median
(C) Radial
(D) Musculocutaneous

Answer: (A)

268. The nerve involved in anterior dislocation of the shoulder is:
(A) Radial nerve
(B) Axillary nerve
(C) Ulnar nerve
(D) Musculocutaneous nerve

Answer: (B)

269. Most common nerve to be damaged in dislocation of shoulder is:
(A) Axillary nerve
(B) Radial nerve
(C) Median nerve
(D) Musculocutaneous nerve

Answer: (A)

270. Anterior dislocation of shoulder is most commonly complicated by:
(A) Axillary artery injury
(B) Circumflex nerve injury
(C) Recurrent dislocation
(D) Axillary nerve injury

Answer: (B)

271. The most common complication of dislocation of shoulder joint is:
(A) Injury to brachial plexus
(B) Injury to circumflex nerve
(C) Rupture of supraspinatous muscle
(D) Rupture of deltoid muscle

Answer: (B)

272. In Anterior dislocation of the shoulder the nerve involved is:
(A) Radial nerve
(B) Circumflex nerve
(C) Ulnar nerve
(D) Median nerve

Answer: (B)

273. A patient with anterior dislocation of shoulder will most likely give a history of
(A) Abduction and internal rotation
(B) Adduction and internal rotation
(C) Abduction and external rotation
(D) Adduction and external rotation

Answer: (C)

274. In Recurrent Anterior dislocation of shoulder, the movements that causes dislocation is
(A) Flexion and internal rotation
(B) Abduction and external rotation
(C) Abduction and internal rotation
(D) Extension

Answer: (B)

275. All of the following statements about dislocation of the shoulder are true, except
(A) The injury is produced by a fall with the arm fully abducted
(B) The commonest position for the head of the humerus to move into is the subspinous
(C) The auxillary (circumflex humeral) nerve is likely to be injured
(D) The easiest way to reduce it is by simple pressure with the patient under general anesthesia with muscle relaxation

Answer: (B, D)

276. Duga’s test is helpful in
(A) Dislocation of hip
(B) Scaphoid fracture
(C) Fracture neck of femur
(D) Anterior dislocation of shoulder

Answer: (D)

277. Traumatic anterior dislocation of shoulder with sensory loss in lateral side of forearm and weakness of flexion of elbow joint, most likely injured nerve is:
(A) Radial nerve
(B) Ulnar nerve
(C) Axillary nerve
(D) Musculocutaneous nerve

Answer: (D)

278. Following anterior dislocation of the shoulder, a pt develops weakness of flexion at elbow and lack of sensation over the lateral aspect forearm; nerve injured is:
(A) Radial nerve
(B) Musculocutaneous nerve
(C) Axillary nerve
(D) Ulnar nerve

Answer: (B)

279. Traumatic glenohumeral instability on one direction with Bankarts lesion are treated by
(A) Conservative methods
(B) Surgery
(C) Rehabilitation
(D) Observation f/b inferior capsule shift

Answer: (B)

280. A patient with history of recurrent posterior dislocation of humerus is evaluated for a ‘Hill-Sach’s lesion. Which aspect of the head of humerus is likely to show this lesion in the above patient?
(A) Anteromedial
(B) Anterior
(C) Posterolateral
(D) Posterior

Answer: (A)

281. Following statement regarding dislocation of shoulder are true except:
(A) Head of humerus usually dislocates forward from shoulder joint
(B) Injury is produced by forced extension & external rotation of abducted arm
(C) In posterior dislocation, appearance of shoulder is not normal
(D) None of the above

Answer: (C)

282. Regarding Recurrent dislocation of the shoulder, which of the following is false?
(A) All traumatic dislocations will be recurrent
(B) Recurrent dislocation results when the capsule is stripped, not torn
(C) The humeral head is always within the capsule
(D) All of these

Answer: (A)

283. Which is true regarding shoulder dislocation?
(A) Posterior dislocation is often over-looked
(B) Pain is severe in anterior dislocation
(C) Radiography may misleading in posterior dislocation
(D) All of the above

Answer: (D)

284. Which is true about shoulder dislocation?
(A) Anterior dislocation is common than posterior
(B) Fixed medial rotation in posterior dislocation
(C) Kocher’s manoeuvre is effective in anterior dislocation
(D) All of the above

Answer: (D)

285. Which of the following is test of posterior glenohumeral instability:
(A) Fulcrum test
(B) Sulcus test
(C) Jerk’s test
(D) Crank test

Answer: (C)

286. A 6 year old boy has a history of recurrent dislocation of the right shoulder. On examination, the orthopedician puts the patient in the supine position and abducts his arm to 90 degrees with the bed as the fulcrum and then externally rotates it but the boy does not allow the test to be performed. The test done by the orthopedician is
(A) Apprehension test
(B) Sulcus test
(C) Dugas test
(D) MC Murray’s test

Answer: (A)

287. Patient is able to abduct, internally rotate and take his arm upto lumbosacral spine but not able to lift off. What is the probable diagnosis
(A) Subscapularis tear
(B) Teres major tear
(C) Long head of biceps tear
(D) Acromioclavicular joint dislocation

Answer: (A)

288. Lift off test is done to assess the function of:
(A) Supraspinatus
(B) Infraspinatus
(C) Teres Minor
(D) Subscapularis

Answer: (D)

289. Velpeau bandage and Sling and Swathe splint are used in?
(A) Shoulder dislocation
(B) Fracture scapula
(C) Acromioclavicular dislocation
(D) Fracture clavicle

Answer: (C)

290. All are TRUE about clavicle, EXCEPT
(A) No treatment required for fracture but rest
(B) Breaks at midpoint
(C) First bone to ossify
(D) Ossifies in membrane

Answer: (A)

291. True statement regarding Fracture of clavicle is:
(A) Most common Complication is Malunion
(B) Occurs at the jn.of medial 1/3rd& lateral 2/3rd
(C) Usually occurs due to fall on elbow
(D) Communitted fracture is common

Answer: (A)

292. Common injury to baby is:
(A) Fracture humerus
(B) Fracture clavicle
(C) Fracture
(D) Fracture femur

Answer: (B)

293. The most common bone fractured during birth
(A) Clavicle
(B) Scapula
(C) Radius
(D) Humerus

Answer: (A)

294. The most common complication of clavicle fracture is
(A) Injury to brachial plexus
(B) Malunion
(C) Stiffness of shoulder
(D) Non union

Answer: (B)

295. Clavicular fracture is usually treated by
(A) Traction
(B) Open Reduction & Internal fixation
(C) Figure of eight bandage
(D) Plate & Screw fixation

Answer: (C)

296. In treating a fractured clavicle in a 14 month old child, the best procedure is:
(A) Open reduction
(B) Shoulder cast
(C) Figure – of – eight bandage
(D) Kirshner pin

Answer: (C)

297. Fracture of clavicle is commonest at:
(A) Junction of medial 1/3rd and lateral 2/3rd
(B) Junction of medial 2/3rd and lateral 1/3rd
(C) Midpoint
(D) Scapular end

Answer: (B)

298. “Figure of Eight” bandage used commonly in the fracture of
(A) Scapula
(B) Humerus
(C) Clavicle
(D) Metacarpals

Answer: (C)

299. True about fracture clavicle is
(A) Malunion
(B) Most common site is medial 1/3rd& 2/3rd
(C) Comminuted fracture
(D) Due to fall on outstretched hand

Answer: (A, D)

300. Commonest fractures in childhood is:
(A) Femur
(B) Distal humerus
(C) Clavicle
(D) Radius

Answer: (C)

301. Treatment of fracture clavicle in an infant is best treated by:
(A) Cuff and sling
(B) Figure of 8 bandage
(C) Open reduction
(D) Shoulder cast

Answer: (B)

302. Fracture of the clavicle are very common injuries. The most frequent complication of this fracture is:
(A) Malunion
(B) Delayed union
(C) Non union
(D) Nerve injury

Answer: (A)

303. All are true regarding Clavicular fracture except:
(A) May be caused by a fall on to the outstretched arm
(B) Commonly occurs between the insertions of the caraco-Clavicular and the costoClavicular ligaments
(C) May jeopardize blood supply to the overlying skin
(D) Usually requires useful reduction

Answer: (D)

304. Fracture neck Humerus is common in:
(A) Elderly woman
(B) Young lady
(C) Elderly man
(D) All of these

Answer: (A)

305. In fracture surgical neck of humerus, the following nerve injury is common:
(A) Axillary
(B) Radial
(C) Ulnar
(D) Median
(E) Musculocutaneous

Answer: (A)

306. Treatment of choice for fracture neck of humerus is a 70 year old male:
(A) Analgesic with arm sling
(B) U-slab
(C) Arthroplasty
(D) Open reduction – Internal fixation

Answer: (A)

307. Treatment with fracture neck of humerus in a lady will be:
(A) Triangular sling
(B) Hemiarthroplasty
(C) Chest arm bandage
(D) Internal fixation

Answer: (A)

308. A 65 year female presents with impacted fracture surgical neck of humerus. Treatment of choice is:
(A) Arthroplasty
(B) Arm-Chest strapping
(C) Triangular sling
(D) Wait and watch

Answer: (C)

309. Treatment of choice in 65 year old female with impacted # neck of humerus is:
(A) Triangular sling
(B) Arm chest strapping
(C) Arthroplasty
(D) Observation

Answer: (A)

310. Most common nerve involvement in fracture surgical neck humerus :
(A) Axillary nerve
(B) Radial nerve
(C) Ulnar nerve
(D) Median nerve

Answer: (A)

311. A boy fell down from a tree and has fracture of neck humerus. He cannot raise his arm because of the involvement of:
(A) Axillary nerve
(B) Supraspinatus nerve
(C) Musculocutaneous nerve
(D) Radial nerve

Answer: (A)

312. Which of the following movements will be affected if the greater tubercle of the humerus is lost:
(A) Abduction and lateral rotation
(B) Adduction and flexion
(C) Adduction and medial rotation
(D) Flexion and medial rotation

Answer: (A)

313. Hanging cast is sued in:
(A) # Femur
(B) # Radius
(C) # Tibia
(D) # humerus

Answer: (D)

314. In a fracture shaft humerus, which of the following complication requires immediate surgery?
(A) Compound fracture
(B) Nerve injury
(C) Brachial artery occlusion
(D) Comminuted fracture

Answer: (C)

315. The most important cause of Nonunion of fracture of humeral shaft is
(A) Comminuted fracture
(B) Compound (Open) fracture
(C) Overriding of fracture ends
(D) Distraction at fracture site
(E) Operative reduction

Answer: (D)

316. First to appear amongst the ossification centres about the elbow is:
(A) Radial Head
(B) Olecranon
(C) Lateral epicondyle
(D) Capitellum

Answer: (D)

317. Three point relationship is reversed in all, except
(A) # medial epicondyle
(B) # lateral epicondyle
(C) Supracondylar #
(D) Posterior elbow dislocation

Answer: (C)

318. Three bony point relationship is maintained in:
(A) Supracondylar # humerus
(B) Dislocation of elbow
(C) # Lateral condyle
(D) Intercondylar #

Answer: (A)

319. Three point relationship is lost in all, EXCEPT
(A) Fracture of lateral epicondyle
(B) Fracture of medial epicondyle
(C) Posterior dislocation of elbow
(D) Supracondylar fracture of humerus

Answer: (D)

320. Most common nerve involved in supracondylar fracture of humerus is:
(A) Radial nerve
(B) Ulnar nerve
(C) Median nerve
(D) Anterior interosseus nerve

Answer: (D)

321. Triangular relation of Elbow is maintained in
(A) Fracture ulna
(B) Anterior dislocation of Elbow
(C) Posterior dislocation of Elbow
(D) Supracondylar fracture

Answer: (A)

322. Increased intercondylar distance is seen in fracture of all except:
(A) Olecranon
(B) Medial epicondyle
(C) Lateral epicondyle
(D) Lateral condyle

Answer: (B)

323. 3 point symmetry is NOT disturbed in which fracture
(A) Fracture ulna only
(B) Fracture radius only
(C) Fracture of radius & ulna both bones of forearm
(D) Weak posterior capsule

Answer: (B)

324. Postero lateral anconeus triangle is formed by
(A) Head of radius, lateral epicondyle, medial epicondyle
(B) Head of radius, lateral epicondyle, olecranon
(C) Olecranon, medial epicondyle, neck of radius
(D) Neck of radius, head of radius, lateral epicondyle

Answer: (B)

325. All true regarding fracture lateral condyle humerus except:
(A) Salter Harris type IV injury
(B) Most common complication of surgically treated cases is cubitus valgus deformity
(C) Tardy ulnar nerve palsy occurs
(D) Cubitusvarus occur more commonly than valgus
(E) Open reduction & internal fixation

Answer: (C)

326. Fracture of lateral condyle of humerus seen in age group of
(A) 2- 3 years
(B) 3 – 5 years
(C) 5 – 15 years
(D) 15 – 25 years
(E) 35 – 45 years

Answer: (A)

327. “Cubitus – valgus” deformity is complication of:
(A) # Lateral condyle of humerus
(B) # Interconduylar of humerus
(C) # Of the olercranon
(D) # Head of the radius

Answer: (B)

328. Tardy ulnar nerve paly seen in
(A) Medial condyle # humerus
(B) Lateral condyle # humerus
(C) Supracondylar condyle # humerus
(D) Fracture shaft humerus

Answer: (B)

329. Tardy ulnar neuritis may be due to
(A) Advanced osteo arthritis of elbow
(B) Cubitus valgus deformity
(C) Both of the above
(D) None of the above

Answer: (A)

330. A 12-year-old child presents with tingling sensation and numbness in the little finger and gives history of fracture in the elbow region 4 years back. The probable fracture is
(A) Lateral condyle fracture humerus
(B) Injury to ulnar nerve
(C) Supracondylar fracture humerus
(D) Dislocation of elbow

Answer: (C)

331. Tardy ulnar nerve palsy occur as a delayed sequel of
(A) Supracondylar fracture of humerus
(B) Posterior dislocation of elbow
(C) Fracture of lateral condyle of humerus in children
(D) Fracture of Olecranon

Answer: (A)

332. Tardy Ulnar nerve palsy is caused by
(A) Fracture lateral epicondyle of humerus
(B) Fracture medial epicondyle of humerus
(C) Elbow dislocation
(D) Supra condylar fracture of humerus

Answer: (D)

333. Which fracture requires open reduction in children?
(A) Fracture of both bones of forearm
(B) Epiphyseal separation of tibia
(C) Intercondylar fracture of femur
(D) Lateral condyle fracture of humerus

Answer: (C)

334. Open reduction in children is required for
(A) Fracture both bones of forearm
(B) Femoral condyles
(C) Lateral humeral condyle
(D) Distal tibial epiphysis

Answer: (B)

335. Which of the following is known for Non union in children, if left untreated?
(A) Inercondylar fracture of humerous
(B) Fracture shaft of humerus
(C) Fracture shaft of femur
(D) Fracture distal 1/3rd of tibia
(E) Fracture lateral condyle of humerus

Answer: (B)

336. A 6 year old child has an accident and had # elbow, after 4 years presented with tingling and numbness in the ulnar side of finger, fracture is
(A) supra condylar # humerus
(B) lateral condylar # humerus
(C) olecranon
(D) dislocation of elbow

Answer: (A)

337. “Cubitus-valgus” deformity is complication of:
(A) # Lateral condyle of humerus
(B) # Intercondylar of humerus
(C) # Of the olecranon
(D) # Head of the radius

Answer: (B)

338. A patient sustained injury to the upper limb 3 yrs back; he now presents with valgus deformity in the elbow and paresthesias over the medial border of the hand. The injury is likely to have seen:
(A) Supracondylar # humerus
(B) Lateral condyle # humerus
(C) Medial condyle # humerus
(D) Post dislocation of the humerus

Answer: (C, D)

339. Fracture lateral condyle of the humerus is a common injury in children. Which one of the following is the most ideal treatment for a displaced fracture lateral condyle of the humerus in a 7-year-old child?
(A) Open reduction and plaster immobilization
(B) Closed reduction and plaster immobilization
(C) Open reduction and internal fixation
(D) Excision of the fractured fragment

Answer: (C)

340. Growth disturbance, nonunion, elbow instability & late ulnar nerve palsy is commonly seen in
(A) Fracture supracondylar humerus
(B) Fracture medial condyle
(C) Fracture lateral condyle
(D) Fracture head radius

Answer: (C)

341. Medial epicondyle fracture results in injury to – nerve.
(A) Radial
(B) Median
(C) Ulnar
(D) Axillary

Answer: (A)

342. Most common associated injury of the fracture medial epicondyle:
(A) Elbow dislocation
(B) Monteggia fracture dislocation
(C) Fracture supracondylar humerus
(D) Vascular defecit

Answer: (A)

343. Suspected medial epicondylar fracture of humerus in a 4 year old child requires:
(A) X-ray both arms with elbow for comparison
(B) X-ray same limb only
(C) Examination under general anaesthesia
(D) POP in full flexed position

Answer: (D)

344. The most common injury in a 7 years old child due to fall on outstretched hand is
(A) Dislocation of shoulder
(B) Colle’s fracture
(C) Fracture of clavicle
(D) Supracondylar fracture of humerus

Answer: (C)

345. After falling from a height, a child lands on his fully out stretched hands. ON examination there is pain and swelling over his right elbow. Give your probable diagnosis:
(A) Fracture olecranon
(B) Post dislocation of elbow
(C) Supra condylar fracture of humerus
(D) Fracture both bones forearm

Answer: (C)

346. Most common elbow injury in adolescents is:
(A) Dislocation
(B) Physeal injury
(C) Supracondylar fracture
(D) Olecranon fracture

Answer: (A)

347. The most common elbow injury in children is
(A) Extension type of supracondylar fracture of humerus
(B) Dislocation of elbow
(C) Fracture lateral condyle of humerus
(D) Fracture medial epicondyle of humerus

Answer: (C)

348. In supracondylar fracture of humerus, the distal segment is often displaced to;
(A) Anteriorly
(B) Laterally
(C) Posterioly
(D) Medially

Answer: (C)

349. The most common type of supracondylar fracture is
(A) Neutral
(B) Flexion
(C) Extension
(D) Lateral

Answer: (D)

350. Fracture supracondylar fracture is usually caused by :
(A) Hyper flexion injury
(B) Axial rotation
(C) Extension injury
(D) Hyper extension injury

Answer: (A)

351. In flexion injuries causing supracondylar fractures, the distal fragment is often displaced to
(A) Anterior
(B) Posterior
(C) Medial
(D) Lateral

Answer: (D)

352. A 10-year-old boy presenting with a cubitusvarus deformity and a history of trauma 3 months back on clinical examination, has the preserved 3 bony point relationship of the elbow. The most probable diagnosis is
(A) Old unreduced dislocation of elbow
(B) Non-union lateral condylar fracture of humerus
(C) Malunitedintercondylar fracture of humerus
(D) Malunited supracondylar fracture of humerus

Answer: (B)

353. The malunion of supracondylar fracture of the humerus most commonly leads to:
(A) Flexion deformity
(B) Cubitusvarus
(C) Cubitus valgus
(D) Extension deformity

Answer: (D)

354. Cabitusvarus is most commonly seen in
(A) Rickets
(B) Post inflammatory epiphyseal damage
(C) Fracture lateral condyle humerus
(D) Malunited supracondylar fracture

Answer: (B)

355. The most common deformity seen in supracondylar fracture of humerus is
(A) Inability to supinate and pronate
(B) Varus
(C) Valgus
(D) None
(E) Malunion with gunstock deformity

Answer: (E)

356. The most common complication of supracondylar fracture is
(A) Osteosarcoma
(B) Genu valgum
(C) Blood vessel injury
(D) Volkman’sischaemic contracture
(E) Malunion with gun stock deformity

Answer: (C)

357. Gunstock deformity is due to
(A) Fracture of 1st metacarpal bone
(B) Fracture of lower end of radius
(C) Supracondylar fracture of humerus
(D) Lateral condylar fracture of humerus

Answer: (A)

358. Osteotomy done for mal united supracondylar fracture is:
(A) French
(B) Shanz’s
(C) McMurry’s
(D) McAlister

Answer: (E)

359. The following fractures are known for Non-union except:
(A) Fracture of lower half of tibia
(B) Fracture of neck of femur
(C) Fracture of scaphoid
(D) Fracture of patella
(E) Supracondylar fracture of humerus

Answer: (D)

360. All of the following are complications of supracondylar fracture of humerus in children, except:
(A) Compartment syndrome
(B) Myositis ossificans
(C) Malunion
(D) Non Union

Answer: (C)

361. Least common complication of closed supracondylar fracture in a 8 year old child is:
(A) Cubitus valgus
(B) Cubitusvarus
(C) Nonunion
(D) Vessel injury

Answer: (D)

362. Which complication may arise after supra-condylar fracture?
(A) Median nerve injury
(B) Damage to brainchild artery
(C) Cubitusvarus
(D) All of the above

Answer: (C)

363. Complications of supracondylar fracture of humerus are all except:
(A) Elbow stiffness
(B) Mal union
(C) Non union
(D) Myositis ossification
(E) Gun-Stock deformity

Answer: (D)

364. In supra condylar fracture of humerus, the nerve most commonly injured is
(A) Radial nerve
(B) Ulnar nerve
(C) Median nerve
(D) Auxillary nerve

Answer: (C)

365. Tardy unlar nerve palsy caused by:
(A) Supracondylar #
(B) Lateral condylar #
(C) Olecranon #
(D) Intercondylar
(E) Distal radio-ulnar dislocation

Answer: (B, A)

366. All of the following are associated with supracondylar fracture of humerus, EXCEPT?
(A) It is uncommon after 15 yrs of age
(B) Extension type fracture is more common than the flexion type
(C) Cubitusvarus deform commonly results following malunion
(D) Ulnar nerve is most commonly involved.

Answer: (D)

367. The true statement regarding supracondylar fracture of the Humerus in children
(A) Admission to hospital is essential following reduction
(B) It is due to a fall on the point of the elbow
(C) It require open reduction
(D) It is usually compound

Answer: (A)

368. A10-year-old boy presenting with a cubitusvarus deformity and a history of trauma 3 months back on clinical examination, has the preserved 3 bony pint relationship of the elbow. The most probable diagnosis is
(A) Old unreduced dislocation of elbow
(B) Non-union lateral condylar fracture of humerus
(C) Malunitedintercondylar fracture of humerus
(D) Malunited supracondylar fracture of humerus

Answer: (D)

369. The most common cause of Volkmann’s ischaemic contracture (V.I.C) in a child is:
(A) Intercondylar fracture of humerus
(B) Fracture both bone of forearm
(C) Fracture lateral condyle of humerus
(D) Supracondylar fracture of humerus

Answer: (D)

370. Vascular injury during childhood is common in fracture of:
(A) Lower end of humerus
(B) Lower end radius
(C) Upper end of femur
(D) Upper end of radius

Answer: (A)

371. The first again of Volkman’s ischemia is:
(A) Paresthesia
(B) Pain on passive extension of fingers
(C) Pain on active extension of fingers
(D) Swelling of fingers

Answer: (B)

372. The most important sign in Volkmann’s ischaemic contracture is:
(A) Pain
(B) Pallor
(C) Numbness
(D) Obliteration of radial pulse

Answer: (A)

373. Volkman’sischaemic contracture mostly involves
(A) Flexor digitorumsuperficialis
(B) Pronator teres
(C) Flexor digitorumprofundus
(D) Flexor carpi radialislongus

Answer: (C)

374. Most common muscle involved in volkmann’s ischemic contracture is
(A) Flexor pollicislongus
(B) Flexor-digitorumprofunds
(C) Flexor-indicis
(D) Abductor policis

Answer: (B)

375. EarliestIschaemic feature after reduction of Supracondylar fracture is
(A) Coldness
(B) Pain
(C) Swelling
(D) Tingling

Answer: (B)

376. The most common cause of anterior compartment syndrome is
(A) Fractures
(B) Post ischaemic swelling
(C) Superficial injury to muscles
(D) Operative trauma

Answer: (A)

377. Earliest symptom of Volkmann’s Ischaemic is
(A) Pain in flexor muscles
(B) Absence of pulse
(C) Pain on passive extension
(D) Cyanosis of limb

Answer: (C)

378. One of the following is associated with Volkmann’s ischaemic contracture
(A) Supracondylar fracture of humerus
(B) Fracture shaft humerus
(C) Intercondylar fracture of humerus
(D) Dislocation of elbow

Answer: (A)

379. In Volkman’s ischemia, surgery should be done;
(A) Immediately
(B) After 6 hours
(C) 24 hours
(D) 72 hours

Answer: (A)

380. A patient presenting with Volkmann’s Ischaemia all of the following are done except
(A) Split open the plaster of Paris cst and bandage
(B) Decompression by fasciotomy
(C) Exploration
(D) Sumpathetic ganglion blockade

Answer: (D)

381. Volkmann’s contracture:
(A) Is localized thickening of palmar fascia
(B) Develops at the ankle in a case of chronic venous ulcer
(C) Follows Ischemia of the forearm
(D) Is due to excessive scarring of the skin of the arm following a burn.

Answer: (C)

382. Volkmann’s Ischaemic Contracture is due to
(A) Injury to ulnar and median nerve
(B) Injury to median nerve alone
(C) Contracture of the palmar fascia
(D) Ischaemic vascular injury to the muscle
(E) All of the above

Answer: (D)

383. In Volkmann’s ischemia, surgery should be done within:
(A) 24 hours
(B) 36 hours
(C) 1 hour
(D) 6 hours

Answer: (C)

384. Volkmann’s ischameic contracture is commonly due to
(A) Tight
(B) palster
(C) Both
(D) None

Answer: (C)

385. What is true about compartment syndrome:
(A) Loss of pulses is reliable sign
(B) Pain on passive stretch is reliable sign
(C) Interstitial stretch is reliable sign
(D) Fasciotomy is earliest management

Answer: (B, D)

386. Volkmann’s Ischaemic contracture is due to
(A) Arterial injury
(B) Venous injury
(C) Nerve injury
(D) Increase of compartment pressure in the limb

Answer: (D)

387. All are relevant in compartment syndrome except:
(A) Fasciotomy
(B) Splitting of tight pop cast
(C) Reexploration
(D) Exercise

Answer: (D)

388. Which of the following does not involve nerve damage
(A) GuillianBarre syndrome
(B) Erb’s paralysis
(C) Volkmann’s paralysis
(D) Neurotmesis

Answer: (C)

389. The most common nerve involved in Volkamann’sischaemic contracture of forearm includes
(A) Radial
(B) Ulnar
(C) Median
(D) Posterior interosseous

Answer: (C)

390. All are correct regarding compartment syndrome
(A) Pulse is reliable indicator
(B) Pain on passive stretching
(C) Interstitial pressure > capillary pressure
(D) Hyperesthesia
(E) Fasciotomy is the earliest treatment

Answer: (A)

391. Myositis ossificans is most common around the ____ joint
(A) Knee
(B) Elbow
(C) Wrist
(D) Hip

Answer: (B)

392. Treatment of Acute Myositis Ossificans is
(A) Active mobilization
(B) Passive mobilization
(C) Infra Red Therapy
(D) Immobilization

Answer: (D)

393. Myositis ossificans is due to:
(A) Ossification
(B) New bone formation
(C) Ossification of subperiostealhaemoatoma
(D) Migration osteoblasts to haematoma
(E) All of the above

Answer: (E)

394. Treatment of choice in Acute myositis ossificans is:
(A) Immobilization of elbow
(B) Short wave daithermy
(C) Passive movements of arm
(D) Active exercises

Answer: (A)

395. Following are recognized F/O Myositis ossificans EXCEPT:
(A) It is a post traumatic ossification
(B) It follows either a posterior dislocation or a supracondylar fracture of elbow joint
(C) The complication less likely in children
(D) Diagnosis is made with certainly by skiagraphy

Answer: (C)

396. In traumatic myositis ossificans, following are true except:
(A) Hydroxyapatite deposition
(B) Common in elbow injury
(C) Periosteal hematoma & leakage
(D) Injury of tendon over muscle

Answer: (A, D)

397. In myositis ossificans mature bone is seen:
(A) At periphery
(B) In center
(C) Whole muscle mass
(D) In the joint capsule

Answer: (A)

398. Radiological feature differentiating myositis ossificans from bone tumor is:
(A) Peripheral ossificans
(B) Central lucency
(C) Discontinuity with the bone
(D) None

Answer: (C)

399. Which of the following is not true about Myositis ossificans?
(A) Associated with muscle tendon rupture
(B) Inflammation around the ruptured muscle deposition of hydroxyapatite crystals with
(C) Common in supracondylar fracture
(D) Ossification of musculo-periosteal haematoma

Answer: (A)

400. False about myositis ossificans progressive (child with heterotopic ossifications) is
(A) Pneumonia is common
(B) Life longevity is normal
(C) Most common site involved is the spine
(D) Onset is before 6 year

Answer: (B)

401. The basic pathology in Myositis Ossificans Progressive is in
(A) Muscle fibers
(B) Serum chemistry
(C) Body collagen
(D) None of the above

Answer: (A)

402. A person of 60 years age is suffering from myositis ossificans progressive. The usual cause of death would be:
(A) Nutritional deficiency
(B) Bed sore
(C) Lung disease
(D) Septicemia

Answer: (C)

403. All are true regarding myositis ossificans progressive except
(A) Usually involve childs
(B) Progressive form of normal myositis
(C) Respiratory problems
(D) Ankylosis
(E) Form normal bone

Answer: (B)

404. A patient presents with heterotopic ossification around the knee joint. The laboratory investigation of choice in him is:
(A) Alkaline phosphatase
(B) Serum calcium
(C) Serum potassium
(D) Serum acid phosphates

Answer: (A)

405. A child is spinned around by holding his hand by his father. While doing this the child started crying and does not allow his father to touch his elbow. The diagnosis is:
(A) Pulled elbow
(B) Radial head dislocation
(C) Annular ligament tear
(D) Fracture olecranon process

Answer: (A)

406. Pulled Elbow is:
(A) Disarticulation of elbow
(B) Subluxation of distal radio-ulnar joint
(C) Subluxation of proximal radio ulnar joint
(D) None of the above

Answer: (C)

407. Pulled elbow is
(A) A sprain of extensor tendons
(B) Dislocation of head of radius
(C) Fracture of lateral condyle of humerus
(D) Dislocation of elbow

Answer: (B)

408. A 30 y ears old male comes to ortho emergency with his 3 years old daughter who is crying. The father gives the history of child being swung by forearm. The most probable diagnosis is:
(A) Supracondylar humerus fracture
(B) Elbow dislocation
(C) Stress fracture
(D) Pulled elbow

Answer: (D)

409. A one and a half year old child holding her father’s hand slipped and fell but did not let go of her father’s hand. After that she continued to cry and hold the forearm in pronated position and refused to move the affected extremity. Which of the following management at this stage is most appropriate?
(A) Supinate the forearm
(B) Examine the child under GA
(C) Elevate the lib and observe
(D) Investigate for osteomyelitis

Answer: (A)

410. A 3 year old girl has developed painful elbow after being jerked by the forearms. The next thing to be done is:
(A) Cuff & collar sling immobilization.
(B) Raduction setting and AEPOP application
(C) X-ray and elevation of limb in posterior slab
(D) Fully supinate the forearm.

Answer: (D)

411. Excision of head of radius in a child should not be done because
(A) It produces instability of elbow joint
(B) It leads to secondary Osteo arthritis of elbow
(C) It causes subluxation of inferior radio-ulnar joint
(D) It cause myositis ossificans.

Answer: (C)

412. If head of the radius is removed, it will result in
(A) Lengthening of limb
(B) Valgus deformity
(C) Varus deformity
(D) No deformity

Answer: (B)

413. OR is not required in which fracture:-
(A) Patella
(B) Outer 1/3 of radius
(C) Condyle of humerus
(D) Olecranon displaced #

Answer: (B)

414. An oblique # of olecranon. If displaced proximally. The treatment is
(A) Excision &resuturing
(B) Tension band wiring
(C) Elbow is imbolised by cast
(D) Open reduction & external fixation

Answer: (B)

415. In fracture of the olecranon, excision of the proximal fragment is indicated in all of the following situations except:
(A) Old ununited fractures
(B) Non-articular fractures
(C) Fracture extending to coronoid process
(D) Elderly patient

Answer: (C)

416. Moneteggia fracture is fracture of
(A) Lower 1/3rd of radius
(B) Upper 1/3rd of radius
(C) Lower 1/3rd ulna
(D) Upper 1/3rd of Ulna

Answer: (D)

417. In Monteggia fracture, which is true about ulnar fracture and head of radius
(A) Both ulnar fracture and head of radius is displaced posteriorly
(B) Both ulnar fracture and head of radius is displaced anteriorly
(C) Ulnar fractures is posteriorly and head of radius is displaced anteriorly
(D) Ulnar fracture is anteriorly and head of radius is displaced posteriorly

Answer: (B)

418. Posterior interosseous nerve is injured in
(A) Posterior dislocation of elbow
(B) Monteggia fracture dislocation
(C) Reversed monteggia fracture dislocation
(D) Supracondylar fracture of humerus

Answer: (B)

419. Galeazzi fracture is fracture of:
(A) Upper end of ulna
(B) Lower end of ulna
(C) Upper end of radius
(D) Lower end of radius

Answer: (D)

420. Galeazzi fracture is
(A) Supracondylar fracture of the humerus
(B) Fracture of the distal radius with inferior radio ulnar joint dislocation
(C) Fracture of radius in the proximal site and dislocation of the elbow
(D) Fracture of the radial head

Answer: (B)

421. The basic principle in the treatment of fractures of both bones of the forearm is to
(A) Reduce angulation of radius and ulna
(B) Restore the normal relationship of radius and ulna
(C) Immobilize the elbow only
(D) Prevent over riding of fragments
(E) All of the above

Answer: (A, B)

422. The treatment of choice of fracture of radius and ulna in a an adult is:
(A) Plaster for 4 weeks
(B) Closed reduction and calipers
(C) Only plates
(D) Kuntscher nails

Answer: (C)

423. Fracture of both bones forearm at same level, position of the arm in plaster is
(A) Full supination
(B) 10 degree supination
(C) Full pronation
(D) Mid-prone

Answer: (D)

424. A colles fracture is
(A) Common in adolescence
(B) A fracture about the ankle joint
(C) Common in elderly women
(D) A fracture of head of the radius

Answer: (C)

425. Deformities present in colles fracture is/are:
(A) Ventral tilt
(B) Dorsalttilt
(C) Ventral displacement
(D) Dorsal displacement
(E) Shortening

Answer: (B, D, E)

426. Following displacement seen in Colle’s fracture EXCEPT:
(A) Dorsal tilt
(B) Ventral tilt
(C) Dorsal displacement
(D) Lateral displacement

Answer: (B)

427. Seen in Colles # A/E:
(A) Proximal shift
(B) Dorsal tilt
(C) Lateral tilt
(D) Pronation

Answer: (D)

428. About displacement of distal fragment in colles fracture, true is:
(A) Anteriorly and medially
(B) Posteriorly and laterally
(C) Anteriorly and laterally
(D) Posteriorly and medially

Answer: (B)

429. ‘Dinner fork deformity’ is present in case of
(A) Smith’s fracture
(B) Student’s elbow
(C) Colles’ fracture
(D) All of these

Answer: (C)

430. In colles fracture not seen is:
(A) Proximal impaction
(B) Lateral rotation
(C) Dorsal angulation
(D) Medial rotation

Answer: (D)

431. Position of wrist in cast of colle’s fracture is:
(A) Palmar deviation & pronation
(B) Palmar deviation & supination
(C) Dorsal deviation & pronation
(D) Dorsal deviation & supination

Answer: (A)

432. Most common complication of Colles # :
(A) Malunion
(B) Avascular necorsis
(C) Finger stiffness
(D) Rupture of EPL tendon

Answer: (C)

433. Most common complication of colle’s fracture is:
(A) Stiffness of fingers
(B) Sudeck’s dystrophy
(C) Nonunion
(D) Tendon rupture

Answer: (A)

434. The commonest complication of Colle’s fracture is:
(A) Malunion
(B) Non-union
(C) Sudeck’sosteodystrophy
(D) Stiffness of fingers

Answer: (D)

435. Commonest complication of Colles’ fracture is:
(A) Nonunion
(B) Malunion
(C) Vascular injury
(D) Sudeck’sosteodystrophy

Answer: (B)

436. All of the following can be complications of a malunitedColles fracture except:
(A) Rupture of flexor policies longus tendon
(B) Reflex sympathetic dystrophy (RSD)
(C) Carpal tunnel syndrome
(D) Carpal instability

Answer: (A)

437. Rupture of extensor pollicislongus occurs four weeks after:
(A) Colle’s fracture
(B) Radial styloid fracture
(C) Smith’s fracture
(D) Scaphoid fracture
(E) All of these

Answer: (A)

438. Which tendon gets involved in Colle’s fracture?
(A) Abductor pollicislongus
(B) Extensor pllicisbrevis
(C) Extensor pollicislongus
(D) All the above

Answer: (C)

439. Complications of Colle’s fracture include all of the following except:
(A) Malunion
(B) Nonunion
(C) Sudeck’sosteodystrophy
(D) Rupture of EPL tendon

Answer: (B)

440. Complications of Colles’ fracture are all except
(A) Malunion
(B) Non union
(C) Sudeck’s dystrophy
(D) Rupture of extensor policislongus

Answer: (B)

441. Not a complication of Colles’ fracture
(A) Stiffness of wrist
(B) Stiffness of shoulder
(C) Carpal tunnel syndrome
(D) Wrist drop

Answer: (D)

442. The complication of Colles’ fracture is
(A) Radial nerve palsy
(B) Stiffness of wrist joint
(C) Ulnar nerve palsy
(D) None of the above

Answer: (B)

443. Complications of fracture (#) radius are:
(A) Volmann ischemic contracture
(B) Myositis ossificans
(C) Infection
(D) Tendon rupture
(E) Angiodysplasia

Answer: (A, B, C, D)

444. A 50 year old lady sprained her ankle2 months back from which she made a steady recover. 2 months after the injury she gradually developed severe pain in her right ankle with significant limitation of ankle movement. Clinical examination reveals edema and shiny skin. What is the likely diagnosis?
(A) Fibromyalgia
(B) Complex Regional pain syndrome Type I (CRPS I)
(C) Complex Regional pain syndrome Type II (CRPS II)
(D) Peripheral Neuropathy

Answer: (B)

445. A lady presents with swelling of hand with shiny skin. She has a history of fracture radius and kept on POP cast for 4 weeks after which she develops this. Give the most likely diagnosis
(A) Myositis ossification
(B) Rupture of external pollicslongus tendon
(C) Reflex sympathetic dystrophy
(D) Malunion

Answer: (C)

446. Sudeck’s atrophy is associated with:
(A) Osteoporosis
(B) Osteophyte formation
(C) Osteopenia
(D) Osteochondritis

Answer: (C)

447. Sudeck’s atrophy is more common in:
(A) MalunitedColles’ fracture
(B) Malunited fracture femur
(C) Pott’s fracture
(D) Carries spine

Answer: (A)

448. Stellate ganglion block is useful in:
(A) Sudeckosteodystrophy
(B) Compound palmar ganglion
(C) Tenosynovitis
(D) Osteoarthritis of first CMC joint

Answer: (A)

449. Regarding sudeck’sosteosdystrophy all are true except
(A) Burning pain
(B) Stiffness & swelling
(C) Erythematous & cyanotic discolouration
(D) Self limiting& good prognosis

Answer: (D)

450. Management of Smith’s fracture is
(A) Open reduction and fixation
(B) Plaster cast with forearm in pronation
(C) Closed reduction with below-elbow cast
(D) Above-elbow cast with forearm in supination

Answer: (D)

451. Barton’s fracture of the wrist
(A) Involves radio carpal subluxation
(B) Is a severe form of a colles’ fracture
(C) Is often treated by open reduction and internal fixation
(D) All of the above

Answer: (A)

452. All are injuries of lower end of radius except
(A) Smith’s fracture
(B) Colle’s fracture
(C) Night stick fracture
(D) Barton’s fracture

Answer: (C)

453. Which one of the following statements is not correct regarding fracture of the scaphoid
(A) It is the most commonly fractured carpal bone
(B) Persistent tenderness in the anatomical snuffbox is highly suggestive of fracture
(C) Immediate X-ray of hand may not reveal fracture line
(D) Mal union is a frequent complication

Answer: (D)

454. Most common site of scaphoid fracture is
(A) Waist
(B) Proximal fragment
(C) Distal fragment
(D) Tilting of the lunate

Answer: (A)

455. Main risk in fracture Scaphoid is
(A) Non union
(B) Malunion
(C) Delayed union
(D) Avascular necrosis
(E) Carpal tunnel syndrome

Answer: (D)

456. Complication of fracture scaphoid is
(A) Avascular necrosis of distal part
(B) Injury to radial artery
(C) Injury to radial nerve
(D) Avascular necrosis of proximal part

Answer: (D)

457. The proximal fragment of scaphoid after fracture is predisposed for Avascular Necrosis because:
(A) Retrograde blood flow to the proximal fragment
(B) Difficulty in immobilizing the proximal fragment
(C) Fracture configuration of the proximal fragment is usually comminuted
(D) Proximal Fragment articulates with the radius

Answer: (A)

458. Avascular necrosis of bone is most commonly seen in
(A) Calcaneus
(B) Cervical spine
(C) Scaphoid
(D) Scapula

Answer: (C)

459. The best radiological view for fracture scaphoid is:
(A) AP
(B) PA
(C) Lateral
(D) Oblique

Answer: (D)

460. Avascular necrosis of bone is most common in
(A) Scapula
(B) Scaphoid
(C) Calcaneus
(D) Cervical spine

Answer: (B)

461. Oblique view is required to diagnose fracture of
(A) Capitate
(B) Scaphoid
(C) Navicular
(D) Hamate

Answer: (B)

462. Carpal bone which fracture commonly:
(A) Scaphoid
(B) Lunate
(C) Hammate
(D) Pisciform

Answer: (A)

463. A patient reported with a history of fall on an outstretched hand, complains of pain in the anatomical snuffbox and clinically no deformities visible. The diagnosis is:
(A) Colles’ fracture
(B) Lunate dislocation
(C) Barton’s fracture
(D) Scaphoid fracture

Answer: (D)

464. If an adolescent boy falls on a out-stretched hand, the most common bone to be injured is
(A) Fracture of lower end of radius
(B) Fracture of both bones of forearm
(C) Scaphoid fracture
(D) Supracondylar fracture of humerus

Answer: (C)

465. In Scaphoid fracture, important views are all except:
(A) AP
(B) Lateral
(C) Oblique
(D) Cone

Answer: (D)

466. Which of the following Scaphoid fracture is most prone to develop Avanscular necrosis?
(A) Fracture of waist of scaphoid
(B) Fracture of tubercle
(C) Fracture of distal pole
(D) All of the above

Answer: (A)

467. Fracture scaphoid is usually seen in:
(A) Elderly male
(B) Elderly postmenopausal female
(C) Young active adult
(D) Children

Answer: (C)

468. In children fracture scaphoid is through rare but usually involves:
(A) Waist
(B) Proximal pole
(C) Neck
(D) Distal pole

Answer: (D)

469. 19 year old boy had a history of fall on out stretched hand while playing. He developed slight radial side pain and tenderness. On examination pressure along the axis of thumb is painful and x rays are normal.
(A) Fracture lower end radius
(B) Wrist sprain
(C) Scaphoid fracture
(D) Perilunate dislocation

Answer: (C)

470. In nonunion of scaphoid vascularized muscle pedicle graft is taken from.
(A) Pronator teris
(B) Brachioradialis
(C) Pronator quadratus
(D) Extensor pollicis longus

Answer: (C)

471. Glass holding cast is also known as:
(A) Coll’s cast
(B) Smit’s cast
(C) Sarmento cast
(D) Scaphoid cast

Answer: (D)

472. In classical scaphoid cast position of wrist is:
(A) Dorsal & ulnar flexion
(B) Dorsal & radial flexion
(C) Ventral & ulnar flexion
(D) Ventral & radial flexion

Answer: (B)

473. Following are the common sites of Avascular necrosis, EXCEPT:
(A) Proximal half of scaphoid
(B) The body of talus
(C) Patella
(D) Head of the femur

Answer: (C)

474. Which of the following statement(s) is/are true?
(A) Oedema & tenderness over the anatomical snuff box is the pathognomonic features of Fracture of the scaphoid
(B) Normally the radial styloid is 1/2” lower than the ulnar
(C) Dinner fork deformity is characteristic of Colle’s fracture
(D) All of the above

Answer: (D)

475. Internal fixation is probably needed in all of the following except
(A) Fracture condyle of humerus
(B) Fracture neck of femur
(C) Fracture of Olecranon
(D) Fracture of scaphoid

Answer: (D)

476. The most common nerve involvement is dislocation of Lunate is
(A) Median nerve
(B) Anterior interosseus
(C) Posterior interosseus
(D) Median nerve

Answer: (A)

477. Which carpal bone fracture causes median nerve involvement?
(A) Scaphoid
(B) Lunate
(C) Trapezium
(D) Trapezoid

Answer: (B)

478. The commonly inured carpal bone next to scaphoid is
(A) Trapeium
(B) Trapezoid
(C) Lunate
(D) Capitate
(E) Hamate

Answer: (A)

479. Bennett’s fracture is dislocation of base of _____ metacarpal:
(A) 4th
(B) 3rd
(C) 2nd
(D) 1st

Answer: (D)

480. The term Bennett’s fracture is used to describe
(A) Fracture-dislocation of metacarpophalangeal joint of thumb
(B) Interphalangeal fracture dislocation of thumb
(C) Anterior marginal fracture of distal end of radius
(D) Fracture dislocation of trapezometacarpal joint

Answer: (D)

481. A Bennett’s fracture is difficult to maintain in a reduced position mainly because of the pull of the
(A) Flexor pollicislongus
(B) Flexor pollicisbrevis
(C) Extensor pollicisbrevis
(D) Abductor pollicislongus

Answer: (D)

482. A Bennet’s fracture is difficult to maintain in reduced position because of the pull of
(A) Extensor pollicislongus
(B) Extensor pollicisbrevis
(C) Abductor pollicislongus
(D) Abductor pollicisbrevis

Answer: (C)

483. During fixation of Bennett’s fracture, which muscle hinder it
(A) Extensor pollicisbrevis
(B) Flexor pollicisbrevis
(C) Flexor pollicislongus
(D) Abductor pollicislongus

Answer: (D)

484. A cricketer got injured while holding a catch, following which he complained of pain over the base of thumb. Which structure is most likely to be injured.
(A) Volar plate
(B) Extensor pollicislongus
(C) Abductor pillicislongus
(D) Ulnar collateral ligament

Answer: (D)

485. Game Keeper’s thumb is
(A) Ulnar collateral
(B) Radial collateral ligament injury of MCP joint
(C) Radial collateral ligament injury of CMC joint
(D) Ulnar collateral ligament injury of CMC joint

Answer: (A)

486. Mallet finger is
(A) Avulsion fracture of extensor tendon of distal phalanx
(B) Fracture of distal phalanx
(C) Fracture of middle phalanx
(D) Fracture of proximal phalanx

Answer: (A)

487. Avulsion of extensor tendon gives rise to
(A) Mallet finger
(B) Dupuytren’s contracture
(C) Trigger finger
(D) Swan neck deformity

Answer: (A)

488. In hand surgery which area is called no man’s land
(A) Proximal phalanx
(B) Distal phalanx
(C) Between distal phalanx crease and proximal phalanx
(D) Wrist

Answer: (C)

489. Middle palmar space ends distally
(A) Along the digital sheaths
(B) Into the flexor tendon sheaths
(C) Into the web space
(D) By mixing with the superficial palmer space

Answer: (C)

490. In pelvis fracture, the amount of blood loss is around
(A) 1-4 units
(B) 2-4 units
(C) 2-6 units
(D) 4-8 units

Answer: (D)

491. If a patient with a suspected fracture of the pelvis has some bleeding from the urethra and is unable to pass urine
(A) He should be encouraged to pass urine after being given antibiotics and analgesics
(B) He should be immediately catheterized in the ward
(C) A hot water bottle should be given followed by injection of carbachol
(D) He should be prepared for surgery and catheterization attempted in the O.T.

Answer: (D)

492. Jumper’s fracture is seen in:
(A) Calcaneum
(B) Tibia
(C) Pelvis
(D) Neck femur

Answer: (C)

493. Open book and bucket handle injuries are seen in:
(A) Spine
(B) Pelvis
(C) Femur
(D) Humerus

Answer: (B)

494. True about Crescent fracture is:
(A) Anteroposterior instability with rotational stability
(B) Diastasis of pubis with pubic rami fracture
(C) Antero-posterior compression is the mechanism of injury
(D) Fracture of the iliac bone with sacroiliac disruption

Answer: (D)

495. Late complication of Acetabular fracture
(A) Avascular necrosis of head of femur
(B) Avascular necrosis of lilac crest
(C) Fixed deformity of the hip joint
(D) Secondary osteoarthritis of hip joint

Answer: (D)

496. All of the following areas are commonly involved sites in pelvic fracture except:
(A) Pubic rami.
(B) Alae of ileum.
(C) Acetabula
(D) Ischialtuberosities.

Answer: (D)

497. Kocher Langenbeck approach for emergency acetabular fixation is done in all except:
(A) Open fracture
(B) Progressive sciatic nerve injury
(C) Recurrent dislocation inspite of closed reduction and traction
(D) Morel-Lavallee lesion

Answer: (D)

498. Emergency treatment of acetabular # area ll except:
(A) Recurrent dislocations despite fixation with traction
(B) Open acetabular fracture
(C) Progressive aciatic nerve involvement
(D) Morel-Lavallee lesion

Answer: (D)

499. Which is not true about Langenbeckkocher operation?
(A) Adequate exposal of posterior segment
(B) Anterior segment is not Visualized adequately
(C) Superior exposure is very adequate
(D) Sciatic nerve injury in 10 percent in the cases

Answer: (C)

500. Main blood supply to the head and neck of femur comes from
(A) Lateral circumflex femoral Artery
(B) Medial circumflex femoral Artery
(C) Artery of LigamentumTeres
(D) Popliteal Artery

Answer: (B)

501. An elderly woman was admitted with a fracture of the neck of right femur which failed to unite. On examination an avascular necrosis of the head of femur was noted. The condition would have resulted most probably from the damage to:
(A) Superior gluteal artery
(B) Inferior gluteal artery
(C) Acetabular branch of obturator
(D) Retinacular branches of circumflex femoral arteries

Answer: (D)

502. Pauvel’s angle is
(A) Neck shaft angle of femur
(B) The difference between neck shaft angle between two femurs of a patient
(C) Formed by joining a line extended from fracture line of femur neck to an arbitrary line depiciting the horizontal plane
(D) None of the above

Answer: (C)

503. Garden’s classification is applicable to
(A) Intertrochanteric fracture
(B) Fracture neck of femur
(C) Epiphyseal separation
(D) Posterior dislocation of hip

Answer: (B)

504. Garden – I fractures are also known as
(A) Complete fracture with out displacement
(B) Complete fracture with minimal (partial) displacement
(C) Complete fracture with full displacement
(D) Valgus impaction fractures

Answer: (D)

505. In fracture neck femur all the trabeculae of pelvis and femur are in alignment in which stage:
(A) Stage I
(B) Stage II
(C) Stage III
(D) Stage IV

Answer: (B)

506. Increase in Pauwel’s angle indicate
(A) Good prognosis
(B) Impaction
(C) More chances of displacement
(D) Trabecular alignment disrupted

Answer: (C)

507. A 60 year old female lands up in emergency with history of fall trauma, the attitude of limb is extension and external rotation, the most probable diagnosis is
(A) Intra capsular # neck of femur
(B) Posterior dislocation of hip
(C) Intutrochanteric #
(D) Acetabulam #

Answer: (A)

508. A woman aged 60 years suffers a fall; her lower limb is extended and externally rotated; likely diagnosis is:
(A) Neck of femur #
(B) Intertrochanteric femur #
(C) Postr dislocation of hip
(D) Anterior dislocation of hip

Answer: (A)

509. 65-year-old lady falls from height. On examination the leg is extended and externally rotated. Diagnosis is
(A) Fracture of acetabulum
(B) Intertrochanteric fracture
(C) Neck femur fracture
(D) Posterior dislocation of hip

Answer: (C)

510. The commonest hip injury in the elderly patients is:
(A) Stress #
(B) Extracapsular #
(C) Impacted # neck of femur
(D) Sub capital capsular fracture neck of femur

Answer: (B)

511. Concerning fractures of the neck of the femur which statement is considered now to be incorrect
(A) It is common in elderly women
(B) It can occur in young adults due to fatigue
(C) It can occur in young adults following severe violence applied in the long axis of the femur
(D) The bone fractures in an elderly woman because the falls.

Answer: (C)

512. 80 years old female after fall developed inability to walk with external rotation deformity on examination SLR is not possible and broadening of trochanter is present. The possible diagnosis is:
(A) # Neck femur
(B) # Inter trochanteric femur
(C) # Subtrochanteric femur
(D) # Greater trochanter

Answer: (B)

513. A 60-year-old man fell in bathroom and was unable to stand on right buttock region ecchymosis with external rotation of the led and lateral border of foot touching the bed. The most probable diagnosis is:
(A) Extra capsular fracture neck of femur
(B) Anterior dislocation of hip
(C) Intra capsular fracture neck of femur
(D) Posterior dislocation of hip

Answer: (A)

514. Features of fracture neck of femur includes
(A) Flexion at hip and lateral rotation
(B) Flexion at hip abduction
(C) Shortening and lateral rotation
(D) Shortening and flexion

Answer: (C)

515. Occult fracture of neck femur are best diagnosed by:
(A) Bone scan
(B) MRI
(C) X-Ray
(D) CT scan

Answer: (B)

516. An old lady had a history of fall in bathroom once and couldn’t move. Afterwards, she had led in externally rotated position. There was tenderness in Scarp’s triangle and limb movement could not seen done due to pain. No hip fracture was seen on X-ray. Next step
(A) MRI
(B) Repeat X-ray after one week
(C) Joint aspiration
(D) Give analgesic and manipulate

Answer: (A)

517. Nonunion is a very common complication of intracapsular fractures of the neck of femur. Which of the following is not a very important cause for the same?
(A) Inadequate immobilization
(B) Inadequate blood supply
(C) Inhibitory effect of synovial fluid
(D) Stress at fracture site due to muscle spasm

Answer: (D)

518. All of the following can occure as complication of fracture of the neck of femur except:
(A) Shortening
(B) Non-union
(C) Mal-union
(D) Avascular necrosis

Answer: (C)

519. Commonest complication of extra capsular intertrochanteric fracture of neck of femur is:
(A) Non union
(B) Ischemic necrosis
(C) Malunion
(D) Pulmonary complications

Answer: (C)

520. In Which one of the following femoral fractures is Avascular necrosis common?
(A) Pertrochanteric
(B) Transcervical
(C) Sub-Trochanteric
(D) Shaft of femur

Answer: (B)

521. Avascular necrosis of head of femur occurs commonly at:
(A) Transcervical region
(B) Trochanteric region
(C) Subcapital region
(D) Subchondral region

Answer: (C)

522. The most common complication of intracapsular fracture neck of femur is
(A) Mal union
(B) Osteoarthritis
(C) Non-Union
(D) Shortening

Answer: (C)

523. Commonest complication of Trans-cervical fracture of femur is
(A) Non union
(B) Malunion
(C) Avascular necrosis
(D) All of the above

Answer: (C)

524. AVN is seen in which type of # of femur
(A) Interochantric #
(B) Subcapital #
(C) Trans cervical #
(D) Basal #

Answer: (B, C, D)

525. The most common site of fracture neck of femur that causes avascular necrosis is
(A) Sub-capital
(B) Intertrochanteric
(C) Trans-cervical
(D) Basal

Answer: (A)

526. Best treatment for fracture neck femur in a 65 year old lady is
(A) POP cast
(B) Gleotomy
(C) Bone grafting and compression
(D) Hemireplacementarthroplasty

Answer: (D)

527. 65 year old man presented with fracture neck femur 3 days after injury, treatment of choice is:
(A) Multiple screw fixation
(B) Mc-Murray osteotomy
(C) Hemi-arthroplasty
(D) Total hip replacement

Answer: (C)

528. A 50 years male with fracture neck of femur comes after 3 days, treatment of choice is
(A) hemiarthroplasty
(B) total hip replacement
(C) hipspika
(D) CR & IF

Answer: (D)

529. Treatment of choice in fracture neck of femur in a 40-year old male presenting after 2 days is
(A) Hemiarthroplasty
(B) Closed reduction and Internal fixation by cancellous screws
(C) Closed reduction and Internal fixation by Austin Moore pins
(D) Plaster and rest

Answer: (B)

530. Treatment of choice for one week old fracture neck femur at 65 years age is:
(A) Hemi-replacement arthroplasty
(B) Closed reduction and internal fixation by cannulatedcancellous screws
(C) Closed reduction and internal fixation by Austin more pins
(D) Total hip replacement

Answer: (B)

531. In the case of 65 year old person with fracture neck of femur the treatment of choice is
(A) Closed reduction
(B) Closed reduction with internal fixation
(C) Open reduction
(D) Replacement of head and neck of the femur with a prosthesis

Answer: (B)

532. In 65 year old male with history of fracture neck of femur 6 weeks old, treatment of choice
(A) SP nailing
(B) Mc. Murray’s osteotomy
(C) Hemiarthroplasty
(D) None

Answer: (C)

533. The treatment of choice of a 4 weeks old Femoral neck fracture in a 55 years old man is
(A) Open reduction and internal fixation
(B) Mc Murray’s Osteotomy
(C) Hemi replacement arthroplasty
(D) Total hip replacement

Answer: (C)

534. In the case of a 70 year old lady with intra capsular fracture of the neck of femur, the ideal treatment would be
(A) Closed traction
(B) Hemiarthropalsty
(C) Internal fixation with nail
(D) Internal fixation with nail and plate

Answer: (B)

535. 3 days old intracapsular fracture neck of femur in a 50 year old patient is treated by
(A) POP hip spika
(B) Total hip replacement
(C) Cortical screw fixation
(D) Hemi replacement arthroplasty

Answer: (D)

536. Femoral neck fracture of three weeks old in an young adult should be best treated one of the following:
(A) Total hip replacement
(B) Reduction of fracture and femoral osteotomy with head
(C) Prosthetic replacement of femoral head
(D) Reduction of fracture and multiple pin or screw fixation
(E) Upper femoral displacement osteotomy

Answer: (D)

537. A 65 year old patient who presents 10 days after fracture neck femur is best managed by
(A) Internal fixation
(B) Replacement arthroplasty
(C) Mc Murray’s procedure
(D) Traction for 6 weeks

Answer: (A)

538. Prosthetic replacement of femoral head is usually indicated for
(A) Fresh intracapsular fracture head of femur in old patients
(B) Fresh intracapsular fracture of femoral neck in a young adult
(C) Unreduced posterior dislocation of hip
(D) Untreated femoral neck fracture in a patient over 65 years
(E) Pathological femoral neck fracture due to secondaries

Answer: (D)

539. Treatment of choice for a 4 week old femoral neck fracture in a 55yer old male is
(A) Open reduction and internal fixation
(B) Mac Murray’s osteotomy
(C) Hemi replacement arthroplasty
(D) Total hip replacement

Answer: (C)

540. Prosthesis at head of femur applied in:
(A) 40 years young male with # head of femur
(B) 40 year young male with # neck of femur
(C) 40 year young male with posterior dislocation of hip
(D) 65 year old male with non united fracture neck of femur

Answer: (D)

541. Fractured neck of femur is associated with all except
(A) Causes shortening of the leg
(B) Causes internal rotation of the leg
(C) May be pathological
(D) May be treated with hemiarthroplasty

Answer: (B)

542. Which of the following fractures would best be treated by Open reduction?
(A) Fracture of the femoral shaft of the child
(B) Collies’ fracture
(C) Displaced fracture of the femoral neck
(D) Fracture of humeral shaft

Answer: (C)

543. Trochanteric fracture of femur is best treated by
(A) Dynamic hip screw
(B) Inlay plates
(C) Plaster in abduction
(D) Plaster in abduction and internal rotation

Answer: (A)

544. Prosthetic replacement of femoral head is indicated for one of the following sites of fractures
(A) Inter-trochanteric fracture of femoral neck
(B) Subcapital fracture neck femur
(C) Transtrochanteric fracture femur
(D) Basal fracture of femoral neck

Answer: (B)

545. Mc Murray’s osteotomy is based on the following principle
(A) Biological
(B) Bio mechanical
(C) Bio technical
(D) Mechanical

Answer: (B)

546. Meyer’s procedure is a method for treatment of
(A) Recurrent shoulder dislocation
(B) Habitual dislocation of patella
(C) Congenital dislocation of hip
(D) Fracture neck of femur

Answer: (D)

547. A fracture neck femur in a child is best treated by
(A) Spica in abduction
(B) Spica in abduction + internal rotation
(C) Masterly inactivity
(D) Open reduction and internal fixation

Answer: (B)

548. In a 10 year old male transcervical fracture neck femur is best treated by:
(A) Spica
(B) Austin Moore pins
(C) K-Wires
(D) C.C.screw

Answer: (D)

549. Commonest dislocation of the hip is
(A) Posterior
(B) Anterior
(C) Central
(D) None

Answer: (A)

550. Which is true about dislocation of hip joint?
(A) Posterior dislocation is commoner
(B) In posterior dislocation whole lower limb is rotated medially
(C) In anterior dislocation whole lower limb is rotated laterally
(D) All of the above

Answer: (D)

551. Flexion, adduction and internal rotation is characteristic posture in
(A) Anterior dislocation of hip joint
(B) Posterior dislocation of hip joint
(C) Fracture of femral head
(D) Fracture shaft of femur

Answer: (B)

552. The attitude of limb in traumatic dislocation of hip joint is
(A) Flexion, adduction, external rotation
(B) Flexion, adduction, internal rotation
(C) Flexion, adduction, and external rotation
(D) Flexion and adduction only

Answer: (B)

553. A patient with hip in adduction and medial rotation and is unable to move probable diagnosis is
(A) Posterior dislocation head of femur
(B) Fracture shaft of femur
(C) Fracture neck of femur
(D) Sciatica

Answer: (A)

554. The pain around the hip with flexion, adduction & internal rotation of lower limb in a young adult after road traffic accident is suggestive of:
(A) Intracapsular fracture of the femoral neck
(B) Extra capsular fracture of the femoral neck
(C) Posterior dislocation of hip
(D) Anterior dislocation of hip

Answer: (C)

555. Traumatic dislocation of hip is characterized by
(A) Adduction internal rotation deformity
(B) Abduction external rotation deformity
(C) Adduction external rotation deformity
(D) Abduction internal rotation deformity

Answer: (A)

556. Dashboard injury results in
(A) Anterior dislocation of hip
(B) Posterior dislocation of hip
(C) Central dislocation of hip
(D) Fracture neck femur

Answer: (B)

557. Vascular sign of Narath is noticed in
(A) Fracture neck of femur
(B) Perthes disease
(C) Posterior dislocation of hip
(D) All of the above

Answer: (C)

558. A 20 year old male, following a road traffic accident was brought to the casualty. His right leg is shortened, internally roated and adducted. The diagnosis is
(A) Fracture neck of femur
(B) Anterior dislocation of the hip
(C) Posterior dislocation of the hip
(D) Trochanteric fracture of the femur

Answer: (C)

559. Post dislocation of hip leas to which attitude of the lower leg:
(A) Adduction, external rotation and flexion
(B) Adduction, internal rotation and flexion
(C) Abduction, external rotation and flexion
(D) Abduction, internal rotation and flexion

Answer: (B)

560. Deformity of posterior dislocation of hip
(A) Flexion, adduction, Internal rotation
(B) Flexion, abduction, external rotation
(C) Extension abduction, Internal rotation
(D) Extension adduction, external rotation

Answer: (A)

561. Maximum shortening of limbs occur in:
(A) Trochantric # femur
(B) Post dislocation of hip
(C) # neck femur
(D) Anterior dislocation of hip

Answer: (B)

562. A 30-year old male was brought to the casually following a road traffic accident. His physical examination revealed that his right lower limb was short, internally rotated, and fixed and adducted at the hip. The most likely diagnosis is:
(A) Fracture neck of femur
(B) Trochanteric femur
(C) Central fracture dislocation of hip
(D) Posterior dislocation of hip

Answer: (D)

563. Sciatic nerve palsy may occur in the following injury
(A) Posterior dislocation of hip joint
(B) Fracture neck of femur
(C) Trochanteric fracture
(D) Anterior dislocation of hip

Answer: (A)

564. Kumar, a 31 yrs old motorcyclist sustained injury over his Right hip join. X-ray revealed a posterior dislocation of the Right hip joint. The clinical altitude of the affected lower limb will be:
(A) External rotation, extension & abduction
(B) Internal rotation, flexion & adduction
(C) Internal rotation, extension & abduction

(D) External rotation, flexion & abduction


Answer: (B)

565. Pipkin fracture is defined as:
(A) Head of radius fracture
(B) Head of femur fracture
(C) Fracture dislocation of ankle
(D) Fracture neck of femur
(E) Fracture of C6 Spine

Answer: (B)

566. In traumatic injury of the hip:
(A) Anterior dislocation is common
(B) Posterior dislocation is common
(C) Visceral injury usually associated with fracture femur
(D) Open reduction is always necessary

Answer: (B)

567. Following is true in treatment of post, dislocation:
(A) Closed reduction under anaesthesia
(B) Open reduction
(C) Skeletal traction
(D) Soft tissue

Answer: (A)

568. In anterior dislocation of hip, the posture of lower limb will be
(A) Abduction, externally rotated and extension
(B) Abduction, externally rotated and flexion
(C) Abducted externally rotated and flexion
(D) Adducted, internally rotated and flexion

Answer: (B)

569. Flexion, abduction and external rotation with limb length discrepancy is seen in:
(A) Posterior dislocation of hip
(B) Central dislocation of hip
(C) Anterior dislocation of hip
(D) Fracture neck of femur (c)

Answer: (C)

570. Flexion, abduction and external rotation at hip joint with limb length discrepancy is seen in
(A) Fracture neck of femur
(B) Anterior dislocation of hip
(C) Posterior dislocation of hip
(D) None

Answer: (B)

571. Deformity in anterior dislocation of hip is
(A) Ext. rotation, abduction, flexion
(B) Ext. rotation, adduction flexion
(C) Int. rotation, abduction flexion
(D) Int. rotation, adduction flexion

Answer: (A)

572. A 32 year old male presented to the casually with pain in the left hip region following RTA. On examination there is Shortening of left lower limb by 7 cm along with an obvious Flexion and External rotation deformity. A mass is palpable in the left gluteal region which moves with movement of the femur. Most likely X-ray finding would be:
(A) Posterior dislocation of Hip with Neck in full profile
(B) Dislocation of hip with lesser trochanter n full profile
(C) Fracture roof of acetabulum with central dislocation
(D) Acetabular fracture with posterior dislocation of hip

Answer: (B)

573. In per rectal examination, femoral head is palpable in
(A) Anterior dislocation of hip
(B) Posterior dislocation of hip
(C) Central dislocation of hip
(D) Lateral dislocation of hip

Answer: (C)

574. Line joining ant. sup iliac spine to ischial tuberosity and passes a greater trochanter
(A) Nelaton’s line
(B) Showmakers line
(C) Cniene’s line
(D) Perkins line

Answer: (A)

575. Trendelenberg test is positive due to injury to:
(A) Sup gluteal nerve
(B) Inf gluteal nerve
(C) Obutrator nerve
(D) Tibial nerve

Answer: (A)

576. “Trendelenburg sign” is positive in damage of the following nerve :
(A) Inferior gluteal nerve
(B) Pudendal nerve
(C) Superior gluteal nerve
(D) Posterior tibial nerve

Answer: (C)

577. Trendelenbrug’s test positive in all EXCEPT:
(A) Posterior dislocation of hip
(B) Poliomyelitis
(C) # Neck of femur
(D) Tuberculosis of hip joint

Answer: (D)

578. Trendelenberg’s sign is –ve in Inter-Trochanteric fracture because of:
(A) Gluteus medius
(B) Gluteus maximus
(C) Gluteus minimus
(D) Tensor fascia lata

Answer: (D)

579. All of the following names are associated with tests/operations around the h I joint except
(A) Bryant
(B) Shenton
(C) Mc. Murray
(D) Salter
(E) Nelton

Answer: (D)

580. Telescopic test is useful to diagnose
(A) Perthe’s disease
(B) Intracapsular fracture neck of femur
(C) Malunited Trochanteric fracture
(D) Ankylosis of hip joint

Answer: (B)

581. Subrochanteric fractures of femur can be treated by all of the following methods except:
(A) Skeletal traction on Thomas’ splint.
(B) Smith Petersen Nail.
(C) Condylar blade plate.
(D) Ender’s nail.

Answer: (B)

582. The femur is fractured at birth at
(A) Upper third of shaft
(B) Middle third of shaft
(C) Lower third of shaft
(D) Neck region

Answer: (A)

583. In upper one third femoral shaft fracture, the displacement of proximal segment is
(A) Flexion abduction and external rotation
(B) Flexion, adduction and external rotation
(C) Flexion, abduction and internal rotation
(D) Flexion, adduction and internal rotation

Answer: (C)

584. Maximum shortening of lower limb is seen in:
(A) # shaft femur
(B) # Neck femur
(C) # intertochanter
(D) Transcervical #

Answer: (A)

585. Not true about fracture shaft femur in infant is:
(A) Child abuse is commonest cause
(B) Heals rapidly
(C) Traction is all that usually required
(D) Fat embolism is common
(E) Sclera must be examined

Answer: (D)

586. Fracture shaft of femur in children of less than 2 years old is treated by:
(A) Open reduction
(B) External fixation
(C) Gallow’s traction
(D) Closed reduction

Answer: (C)

587. A 3 year old child presents with fracture of femoral shaft had immobilized on traction constantly for two months. The next step of management is:
(A) Hip Spica and if necessary internal fixation
(B) Gallow traction for 2 months
(C) Open reduction and Kuntscher’slover leaf intramedullary nailing or plating
(D) Traction by Thomas splint

Answer: (A)

588. Fracture femur is children is treated by:
(A) Open reduction
(B) Gallow’s splint
(C) Intra medullary nailing
(D) Closed reduction &splintage

Answer: (B)

589. A 4 year old female brought to casualty department with multiple fracture ribs, and inconspicuous history from parents. On examination show multiple bruise and healed fractures. The provable diagnosis is:
(A) Polytrauma for evaluation
(B) Flail chest
(C) Munchausen’ syndrome
(D) Battered baby syndrome

Answer: (D)

590. Fracture of femur at the level of isthmus is best treated by
(A) Intramedullary nail fixation
(B) Plate and screws
(C) Closed method
(D) External fixation

Answer: (A)

591. Intramedullary fixation is ideal in a case of fracture of shaft of femur when there is
(A) A transverse fracture
(B) A compound fracture
(C) Soft tissue interposition between the fractured ends
(D) Such a fracture in a child

Answer: (A)

592. Treatment of choice for old non-united fracture of shaft of femur
(A) Compression plating
(B) Bone grafting
(C) Nailing
(D) Compression plating with bone grafting

Answer: (D)

593. Treatment of Non-union of # shaft femur
(A) Open reduction with external fixation
(B) Excision of the bone
(C) Bone grafting with internal fixation with K-Nail
(D) All of the following.

Answer: (C)

594. Best treatment of 3 weeks old, fracture shaft femur with nonunion is
(A) Bone graft with internal fixation
(B) External fixation
(C) Internal fixation only
(D) Prosthesis

Answer: (A)

595. Fracture shaft of femur in adult unites by
(A) 3 to 4 weeks
(B) 3 to 5 weeks
(C) 3 to 4 months
(D) 3to 6 months
(E) None of these

Answer: (C)

596. Clinical feature of fat embolism includes all except:
(A) Tachypnoea
(B) Systemic hypoxia may occur
(C) Fat globules in urine are diagnostic
(D) Manifests after several days of trauma
(E) Petechaie in the anterior chest wall

Answer: (ALL)

597. True about Post traumatic fat embolism syndrome:
(A) Fracture mobility is a risk factor
(B) Associated diabetes pose a risk
(C) Bradycardia occurs
(D) Thrombocytopenia
(E) On ABG PaO2< 60 mm Hg on FIO2< 0.4

Answer: (A, D, E)

598. True about Post traumatic fat embolism syndrome is A/E
(A) Bradycardial occur
(B) Systemic hypoxia may occur
(C) Fracture mobility is a risk factor
(D) Associated diabetes pose a risk
(E) More common in closed fracture
(F) Liposuction is used for treatment

Answer: (D, F)

599. Factors favoring fat embolism in trauma patient:
(A) Diabetes Mellitus
(B) Mobility of joint
(C) Resp. failure
(D) Hypovolemic shock

Answer: (B, D)

600. Commonest sit of fracture leading to fat-embolism is:
(A) Tibia #
(B) Femur #
(C) Humerus #
(D) Ulna #

Answer: (B)

601. A 30 year old man had road traffic accident and sustained fracture of femur. Two days later he developed sudden breathlessness. The most probable cause can be:
(A) Pneumonia.
(B) Congestive heart failure.
(C) Bronchial asthma.
(D) Fat Embolism.

Answer: (D)

602. A 64 year old hypertensive obsese female was undergoing surgery for fracture femur under general anaesthesia. Intra-operatively her end-tidal carbon dioxide decreased to 20 from 40 mm of Hg. followed by hypotension and oxygen saturation of 85%. What could be the most probable cause?
(A) Fat embolism
(B) Hypovolemia
(C) Brochospasm
(D) Myocardial infarction

Answer: (A)

603. Ramesh sing, a 40 yrs man, was admitted with fracture shaft femur following a road traffic accident. On 2n day he became disoriented. He was found to be tachypnoeic, and had conjunctivalpetechiae.
(A) Pulmonary embolism
(B) Sepsis syndrome
(C) Fat embolism
(D) Hemothorex

Answer: (C)

604. The management of fat embolism includes all of the following except:
(A) Oxygen
(B) Heparinization
(C) Low molecular weight dextran
(D) Pulmonary Embolectomy

Answer: (D)

605. Fat embolism syndrome characterized by :
(A) Tachycardia
(B) Hypoxemia
(C) Fat globules in urine
(D) Thrombocytosis

Answer: (A)

606. True about fat embolism
(A) Seen one week after injury
(B) Patechie
(C) Bradycardia
(D) Tachycardia
(E) ↑ Incidence in multiple #

Answer: (B, D, E)

607. A 22 year old male is admitted with fracture of the left femur. Two days later, he becomes mildly confused, has a respiratory rate of 40/min and scattered petechial rash on his upper torso. Chest X-ray shows patchy alveolar opacities bilaterally. His arterial blood gas analysis is abnormal. The most likely diagnosis is
(A) Cerebral oedema with early neurogenic pulmonary oedema
(B) Pulmonary thrombo-embolism
(C) Chest contusion
(D) Fat embolism

Answer: (D)

608. Fat embolism may ensue following
(A) Fracture of spine and ribs
(B) Fractured fibula
(C) Fracture of skull bone
(D) Fracture of long bones only

Answer: (D)

609. Which of the following is true about fat embolism?
(A) Usually ensues after fracture of the lower limb
(B) Uncommon complication of fracture
(C) Spontaneously reversible process
(D) All of the above

Answer: (A)

610. After an operation on femur bone, chest X-ray shows widespread mottling throughout the lung field like a snowstorm. It is diagnostic of
(A) Fat embolism
(B) Shock lung
(C) Bronchopneumonia
(D) Atelectasis

Answer: (A)

611. Fat embolism is characterized by
(A) Petechealhaemorrhages
(B) Closed fractures of femur
(C) Aggregation of chylomicrons
(D) Fall in the haemoglobin
(E) Fat globules in sputum & urine

Answer: (ALL)

612. True about fat embolism:
(A) Petechia in the anterior chest wall
(B) Bradycardia
(C) Fat globules in urine
(D) Occurs after 1st week of poly trauma
(E) Thrmbocytopenia

Answer: (A, C, E)

613. A person with multiple injuries develops fever, restlessness, tachycardia, tachypnea and perimumbilical rash. The likely diagnosis is:
(A) Air embolism
(B) Fat embolism
(C) Pulmonary embolism
(D) Bacterial pneumonitis

Answer: (B)

614. In fracture of femur popliteal artery is commonly damaged by
(A) Proximal fragment
(B) Distal fragment
(C) Muscle haematoma
(D) Tissue swelling

Answer: (B)

615. The classical example of muscular violence is
(A) # of fibula
(B) # of patella
(C) # of clavicle
(D) all of the these

Answer: (B)

616. Transverse fracture of the patella with separation of fragments is best treated by
(A) Closed reduction with cylinder cast
(B) Open reduction with screw fixation of the fragments
(C) Blind fixation of the two fragments with Kirschner wire
(D) Open reduction with Kirschner wire fixation of the fragment an tension band wiring

Answer: (D)

617. In transverse fracture of the patella, the treatment is
(A) Excision of a small fragment
(B) Wire fixation
(C) Plaster cylinder
(D) Patellectomy

Answer: (B)

618. A comminuted fracture of the Patella should be treated by
(A) Inserting screws and wires
(B) Physiotherapy alone
(C) Removal of all the patella
(D) Removal of smallest piece only
(E) Plastering & Immobilization

Answer: (C)

619. Treatment of displaced transverse fracture of patella:
(A) POP
(B) Tension band wiring
(C) Screw
(D) Patellectomy

Answer: (B)

620. Displaced transverse # patella, Rx is:
(A) Patellectomy
(B) Tension band wiring
(C) POP cast
(D) Non operative

Answer: (B)

621. Tube (Cylinder) cast s applied for the fracture of –
(A) Shoulder
(B) Hip
(C) Pelvis
(D) Knee

Answer: (D)

622. What is acceptable angulation after reducing the fracture tibia
(A) 5
(B) 10
(C) 15
(D) 20

Answer: (A)

623. In fracture neck of fibula, the following nerve is involved
(A) Common peroneal nerve
(B) Anterior tibial nerve
(C) Posterior tibial nerve
(D) Medial popliteal nerve

Answer: (A)

624. Healing below knee joint is slow because of
(A) Decreased subcutaneous fat
(B) Increased movement
(C) Weight bearing
(D) Poor vascularity

Answer: (D)

625. Non union is a common feature of fracture of
(A) Supracondylar humerus
(B) Clavicle
(C) Lower tibia
(D) Coracoid process

Answer: (C)

626. Patellar tendon bearing P.O.P cast is indicated in the following fracture:
(A) Patella
(B) Tibia
(C) Medial malleolus
(D) Femur

Answer: (B)

627. Which of the following causes acute compartment syndrome most frequently
(A) Fractures
(B) Postischemic swelling
(C) Exercise initiated syndrome
(D) Soft tissue injury

Answer: (A)

628. Compartment syndrome is treated by
(A) Fasciotomy
(B) Bicarbonate
(C) Chloride rich fluid
(D) Early aggressive fluid

Answer: (A)

629. A patient develops compartment syndrome (Swelling, pain and numbness) following manipulation & plaster for fracture of both bones of leg. What is the best treatment?
(A) Split the plaster
(B) Infusion of low molecular wt dextran
(C) Elevate the leg after splitting the plaster
(D) Do operative decompression of fascial compartment

Answer: (D)

630. Characteristic features of the acute compartment syndrome in the lower leg include all of the following except
(A) Acute pain on employing the stretch test
(B) Normal pulses
(C) Normal sensation distally
(D) Venous occlusion

Answer: (C)

631. In posterior compartment syndrome which passive movement causes pain?
(A) Dorsiflexion of foot
(B) Foot inversion
(C) Toe dorsiflexion
(D) Toe planter flexion

Answer: (C)

632. A patient has 2 months POP cast for tibial fracture left leg. Now he needs mobilization with a single crutch. You will use this crutch on which side:
(A) Left side
(B) Right side
(C) Any side
(D) Both side

Answer: (B)

633. The stability of the ankle joint is maintained by all of the following, except
(A) Plantar cancaneonavicular (spring) ligament
(B) Deltoid ligament
(C) Lateral ligament
(D) Shape of the superior talar articular surface

Answer: (A)

634. The most commonly affected component of the lateral collateral ligament complex in an “ankle sprain” is the:
(A) Middle component
(B) Anterior component
(C) Posterior component
(D) Deeper component

Answer: (B)

635. Ankle sprain is due to:
(A) Rupture of anterior talo-fibular ligament
(B) Rupture of poster talo-fibular ligament
(C) Rupture of deltoid ligament
(D) Rupture of Clacaneo-fibular ligament

Answer: (A)

636. Inversion injury at the ankle can cause all of the following EXCEPT:
(A) Fracture tip of lateral melleolus
(B) Fracture base of the 5th metatarsal
(C) Sprain of Ext. Digitorumbrevis
(D) Fracture of sustentaculamtali.

Answer: (C)

637. The mechanism of injury of transverse fracture of medial malleolus results due to
(A) Abduction injury
(B) Adduction injury
(C) Rotation injury
(D) Direct injury

Answer: (A)

638. True about Eponymous fractures is A/E
(A) A. Montegia # is # of the proximal third of ulna with radial head dislocation
(B) B. Galeazzi # is # of the distal third of the radius with dislocation of the distal radio-ulnar joint
(C) C. Colles # is # at cortico-cancellous junction of the distal-end of radius with dorsal tilt
(D) D. Potts # Trimalleolar ankle #
(E) E. Bennet’s # is Oblique intra-articular # of the base of the 1st metacarpal

Answer: (D)

639. Cotton’s fracture is
(A) Avulsion fracture of C7
(B) Trimaoleolar
(C) Bimalleolar
(D) Burst fracture of atlas
(E) None of these

Answer: (B)

640. Fracture involving both the malleoli is
(A) Cotton’s fracture
(B) Pott’s fracture
(C) Duputren’s fracture
(D) Dupuytren’s fracture

Answer: (B)

641. Pott’s fracture is fracture of
(A) Lower end of tibia
(B) Lower end of tibia + fibula
(C) Lower end of tibia + Calcaneum
(D) Calcaneum + Talus

Answer: (B)

642. Avascular necrosis is a complication of
(A) Fracture of talus
(B) Fracture of medial condyle of femur
(C) Olecranon fracture
(D) Radial head fracture

Answer: (A)

643. Avascular necrosis is a complication of
(A) Distal fracture both bones leg
(B) Distal fracture fibula
(C) Bimalleolar
(D) Fracture talus

Answer: (D)

644. MC comp. of # talus is
(A) Avascular necrosis
(B) Non union
(C) Osteoarthritis of ankle joint
(D) Osteoarthritis of neck of talus
(E) Non union

Answer: (A)

645. Fracture of talus without displacement in x-ray would lead to:
(A) Osteoarthritis of ankle
(B) Osteonecrosis of head of talus
(C) Avascular necrosis of body of talus
(D) Avascular necrosis of neck of talus
(E) Non union

Answer: (A, C, E)

646. In which type of fracture, the tuber-joint angle is reduced to about half
(A) Crush fracture of calcaneum
(B) Fracture neck of humerus
(C) Dislocation of shoulder
(D) Split fracture of calcaneum

Answer: (A)

647. Giassane’s angle intraarticular fracture of calcaneum:
(A) Reduced
(B) Increased
(C) Not changed
(D) Variable

Answer: (B)

648. Bohler’s angle is decreased in fracture of
(A) Calcaneum
(B) Talus
(C) Navicular
(D) Cuboid

Answer: (A)

649. Bohler angle measurement gives the reference for-
(A) Calcaneus
(B) Talus
(C) Navicular
(D) Cuboid

Answer: (A)

650. Neutral triangle is seen radiologically in:
(A) Neck femur
(B) Proximal humerus
(C) Calcaneus
(D) Talus

Answer: (C)

651. # of calcaneous management depending upon:
(A) Type of fracture
(B) Subtalar joint dislocation
(C) Duration of presentation
(D) Degree of displacement

Answer: (ALL)

652. Calcaneum is associated most commonly with which #
(A) # rib
(B) # vertebrae
(C) # skull
(D) # fibula

Answer: (B)

653. Fall on heel with fracture oscalcis is associated with commonly:
(A) Fracture clavicle
(B) Fracture vertebra
(C) Fracture femur
(D) Posterior dislocation of hip

Answer: (B)

654. Least common complication of fall from height is
(A) Fracture base of skull
(B) Fracture calcaneum
(C) Fracture fibula
(D) Fracture 12th thoracic vertebra

Answer: (C)

655. Fatigue fractures (Stress fractures) are most commonly seen in:
(A) Metatarsals
(B) Tibia
(C) Fibula
(D) Neck of femur

Answer: (A)

656. What type of fracture of petrous temporal bone has got the highest chance of facial nerve paralysis?
(A) Longitudinal
(B) Transverse
(C) Oblique
(D) All have equal incidence

Answer: (B)

657. Motorcyclist’s fracture:
(A) Ring fracture
(B) Comminuted fracture
(C) Separation of suture between anterior and posterior half of skull.
(D) Fracture base of skull

Answer: (A)

658. The commonest cause of spinal cord injuries in our country is:
(A) Road traffic accident
(B) Fall from a height
(C) Fall into well
(D) House collapse

Answer: (B)

659. Dislocation without fracture is seen in:
(A) Sacral spine
(B) Lumbar spine
(C) Cervical spine
(D) Thoracic

Answer: (C)

660. ‘Whip-lash’ injury is caused due to:
(A) A fall from a height
(B) Acute hyperextension of the spine.
(C) A blow on top to head.
(D) Acute hyper flexion of the spine.

Answer: (B)

661. Regarding whiplash injury, a true statement includes
(A) Contusion of the spinal cord and fracture of vertebra
(B) Fracture of vertebral body
(C) Spinal cord injury without vertebral fracture
(D) Vertebral fracture without spinal cord injury

Answer: (C)

662. All are true regarding whiplash injury except
(A) Lumbar spine is commonly involved
(B) Fractures are not common
(C) Paresthesia and chronic pain
(D) Hyperextension injury
(E) Sprains and strains without radiological findings

Answer: (A)

663. Jefferson’s # is:
(A) C1
(B) C2
(C) C2 C1
(D) C2 C3

Answer: (A)

664. Jefferson # is:
(A) # of atlas
(B) # of axis
(C) # of spinous process of C7
(D) # of any cervical vertebra

Answer: (A)

665. All of the following are true about fracture of the atlas vertebra, except:
(A) Jefferson fracture is the most common type.
(B) Quadriplegia is seen in 80% cases.
(C) Atlantooccipital fusion may sometimes be needed.
(D) CT scans should be done for diagnosis.

Answer: (B)

666. Hangman’s fracture is fracture of C2
(A) Dens fracture
(B) Lamina
(C) Pars interarticularis
(D) Spinous process

Answer: (C)

667. True regarding Hangman’s fracture is
(A) Odotoid process fracture of C2
(B) Spondylolisthesis of C2 over C3
(C) Whiplash injury
(D) Fracture of hyoid bone

Answer: (B)

668. Hangman’s fracture is
(A) Subluxation of C5 over C6
(B) Fracture dislocation of C2
(C) Fracture dislocation of ankle joint
(D) Fracture of odontoid

Answer: (B)

669. Regarding Hangman’s fracture true is
(A) High post admission mortality
(B) Most common axis fracture
(C) Surgical treatment is necessary
(D) Union almost always occurs

Answer: (D)

670. Burst fracture of cervical spine is due to
(A) Whiplash injury
(B) Fall of weight on neck
(C) Vertical compression injury
(D) Car accident

Answer: (C)

671. Tear droop fracture of lower cervical spine implies:
(A) Wedge compression fracture
(B) Axial compression fractures
(C) Flexion-rotation injury with failure of anterior body
(D) Flexion compression failure of body

Answer: (D)

672. Most common type of injury to spinal cord is-
(A) Flexon
(B) Extension
(C) Compression
(D) Flexon-rotation

Answer: (D)

673. Burst Fracture of spine is a-
(A) Compression Fracture
(B) Extension injury
(C) Direct injury
(D) Flexion-rotation

Answer: (A)

674. In a Pt. with head injury, unexplained hypotension warrants evaluation of:
(A) Upper cervical spine
(B) Lower cervical spine
(C) Thoracic spine
(D) Lumbar spine

Answer: (B)

675. The compression fracture is commonest in
(A) Cervical spine
(B) Upper thoracic spine
(C) Lower thoracic spine
(D) Lumbosacral region

Answer: (C)

676. Seat belt injury is
(A) Tear drop fracture
(B) Wedge fracture
(C) Chance fracture
(D) Whiplash injury

Answer: (C)

677. Chance fracture is/are
(A) Mainly ligamentous injury
(B) Neurological involvement is common
(C) Flexion compression injury
(D) Seat belt injury

Answer: (D)

678. Spinal shock is associated with:
(A) Increased spinal reflexes
(B) Absent spinal reflexes
(C) Loss of autonomic reflexes
(D) Bizarre reflexes

Answer: (B, C)

679. In spinal shock
(A) Knee jerk is the first reflex to return
(B) High thoracic lesions are commonly associated with more severely associated with more severe neurological deficits
(C) Failure of return of cord activity within 48 hours in a very poor prognostic sign
(D) Both B & C

Answer: (D)

680. Return of Bulbocavernous reflex in spinal shock
(A) Sign of recovery from spinal shock
(B) Partial lesion of spinal cord
(C) Complete transection of spinal cord
(D) Incomplete transection of spinal cord

Answer: (A)

681. Complete transaction of the spinal cord at the C7 level produces all of the following effects except:
(A) Hypotension
(B) Limited respiratory effort
(C) Anaesthesia below the level of the lesion
(D) Areflexia below the level of the lesion

Answer: (D)

682. A 40 years old male after RTA, attains spinal injury. His lower limb power is greater than that of upper limb and sacral sensations are present. Type of spinal cord lesion is:
(A) Central cord syndrome
(B) Anterior cord syndrome
(C) Posterior cord syndrome
(D) Complete spinal cord injury

Answer: (A)

683. A patient presented with Saddle anaesthesia, bladder & bowel are normal and muscle power is normal. The diagnosis is
(A) Cauda equine syndrome
(B) L3-L4 root involvement
(C) Conusmedullaris lesion
(D) L4-L5 disc prolapsed

Answer: (C)

684. Symmetrical areflexicblader bowel & lower limb occur in
(A) Cauda equine syndrome
(B) Conusmedullaris syndrome
(C) Nerve root damage
(D) Brown sequerd syndrome

Answer: (B)

685. Cause of atonic bladder is:
(A) Injury to sacral plexus
(B) Injury to upper thoracic cord
(C) Pregnancy
(D) UTI

Answer: (A)

686. A paralysed bladder following spinal injury is best managed by:
(A) Gibbon’s catheter
(B) Malicot catheter
(C) Foley’s catheter
(D) Metallic catheter

Answer: (C)

687. A patient involved in a road traffic accident presents with quadriparesis, sphincter disturbance, sensory level up to the upper border of sternum and a respiratory rate of 35/minute. The likely level of lesion is
(A) C1-C2
(B) C4-C5
(C) T1-T2
(D) T3-T4

Answer: (B)

688. Wrist flexion and finger extension test the following nerve root.
(A) C6
(B) C7
(C) C8
(D) T1

Answer: (B)

689. Little finger of the hand corresponds to which dermatome?
(A) C6 dermatome
(B) C7 dermatome
(C) C8 dermatome
(D) T1 dermatome

Answer: (C)

690. Which movement is not affected in L5 root involvement
(A) Knee extension
(B) Knee flexion
(C) Toe extension
(D) Thigh adduction

Answer: (D)

691. In case of unconscious patient spinal injury is assessed by
(A) Absence of response to painful stimulus
(B) Absence of deep reflexes
(C) Abdominal respiration
(D) Spinal tenderness

Answer: (A, B, C)

692. Patient develops myelopathy post trauma. What dose of methyl prednisolone is to be given:
(A) 30 mg/kg within 3hrs
(B) 45 mg/kg with 6hrs
(C) 60 mg/kg within 9hrs
(D) 75 mg/kg within 12hrs

Answer: (A)

693. Emergency treatment required in:
(A) Fracture humerus
(B) Fracture pelvis
(C) Vertical compression injury
(D) Car accident

Answer: (C)

694. In spinal cord injury, the patient should be transferred with p ad and bandage in the following position:
(A) Supine
(B) Prone
(C) Lateral
(D) Semi prone

Answer: (A)

695. Position for transport of a patient with fracture lumbar spine in a patient:
(A) Neutral
(B) Hyper flexion
(C) Alternating
(D) Hyperextension

Answer: (A)

696. Careless handling of a suspected case of cervical spine injury may result in:
(A) Injury to the spinal cord leading to quadriparesis or quadriplegia
(B) Intracranial haemorrhage with cerebral imitation or unconsciousness.
(C) Cervical haematoma with compression of brachial vessels
(D) Complete paralysis of the affected upper extremity

Answer: (A)

697. A scooter is hit from behind. The rider is thrown off and he lands with his head hitting the kerb. He does not move, complains of severe pain in the neck and is unable to turn his he ad. Well-meaning onlookers rush up to him and try to make him sit up. What would be the best course of action in this situation?
(A) He should be propped up and given some water to drink
(B) He should not be propped up but turned on his face and rushed to the hospital
(C) He should be turned on his back and a support should be placed behind his neck and transported to the nearest hospital
(D) He should not be moved at all but carried to the nearest hospital in the same position in which he has been since his fall.

Answer: (C)

698. The most important step in primary management of a patient with fracture vertebral column:
(A) Careful transport of patient
(B) Maintenance of airway
(C) Treatment of shock
(D) None of the above

Answer: (B)

699. On accident there is damage of cervical spine, first line of management is:
(A) X-ray
(B) Turn head to side
(C) Maintain airway
(D) Stabilize the cervical spine

Answer: (C)

700. Bulbocavernosus reflex is elicited by A/E
(A) Glans pens
(B) Clitoris
(C) Tug of foley catheter
(D) Peri anal

Answer: (D)

701. A young woman met with an accident and had mild quadriparesis. Her lateral X-ray cervical spine revealed C5-C6 fracture dislocation. Which of the following is the best line of management?
(A) Immediate anterior decompression
(B) Cervical traction followed by instrument fixation
(C) Hard cervical collar and bed rest.
(D) Cervical laminectomy.

Answer: (B)

702. Percutaneous vertebroblasty is indicated in all except
(A) Tuberculosis
(B) Metastatis
(C) Osteoporosis
(D) Hemangioma

Answer: (A)

703. Substance that is used for vertebroplasty is:
(A) Poly methyl methacrylate
(B) Poly ethyl methacrylate
(C) Poly methyl ethacrylate
(D) Poly ethyl ethacrylate

Answer: (A)

704. Disc prolapse commonly occurs at:
(A) L4-L5
(B) L5-S1
(C) C5-C6
(D) C4-C5
(E) C3-C4

Answer: (A, B)

705. Commonest site of Disc prolapse is
(A) C5-C6
(B) T8-T9
(C) L4-L5
(D) L5-S1

Answer: (C)

706. The most common site of Disc prolapse is
(A) L2-L3
(B) L3-L4
(C) L4-L5
(D) L5-S1

Answer: (C)

707. The most common site for Prolapse of intervertebral disc is
(A) Cervical region
(B) Lower thoracic region
(C) Upper thoracic region
(D) Lumber region

Answer: (D)

708. After L4-S1 the next commonest site of intervertebral disc prolapse is
(A) C6-C7
(B) T12-L1
(C) L1-L2
(D) L2-L3

Answer: (A)

709. The most common cause of acute sciatica is due to
(A) Trauma
(B) Secondaries of spine
(C) Acute prolapsed intervertebral disc
(D) Tuberculosis of spine

Answer: (C)

710. A building contractor suddenly complains of lower backache which increase on bending down. He has
(A) Renal colic
(B) Tuberculosis of spine
(C) Disc prolapse
(D) Fibrositis’

Answer: (C)

711. The most important single special investigation in lumbar disc prolapse is
(A) Epidurography
(B) Myelography
(C) MRI
(D) Discography
(E) Spinal venography

Answer: (C)

712. Management in case of rupture of disc at L5, S1 is
(A) Emergency removal of disc
(B) Joint fusion
(C) Immobilization for 2 weeks with spinal brace
(D) Traction

Answer: (C)

713. A 44-year –old man presented with acute onset of low backache radiating to the right lower limb. Examination revealed SLRT <40° on the right side, weakness of extensor halluces longus on the right side, sensory loss in the first web space of the right foot and brisk knee jerk. Which of the following is the most likely diagnosis:
(A) Prolapsed intervertebral disc L4-5
(B) Spondylolysis L5-S1
(C) Lumbar canal stenosis
(D) Spondylolisthesis L4-5

Answer: (A)

714. A previously healthy 45 years old laborer suddenly develops acute lower back pain with right-leg pain & weakness of dorsiflexion of the right great toe. Which of the following is true:
(A) Immediate treatment should include analgesics muscle relaxants & back strengthening exercises
(B) The appearance of the foot drop indicate early surgical intervention
(C) If the neurological sign resolve with in 2 to 3 weeks but low back pain persists, the proper treatment would include fusion of affected Lumbar vertebra.
(D) If the neurological signs fail to resolve within 1 week, Lumbar laminectomy and excision of any herniated nucleus pulposus should be done.

Answer: (B)

715. A middle aged lady presents with complaints of lower back pain. On examination there is weakness of extension of right great toe with no sensory impairment. An MRI of the lumbosacral spine would most probably reveal a prolapsed intervertebral disc at what level?
(A) L3-L4
(B) L4-L5
(C) L5-S1
(D) S1-S2

Answer: (B)

716. A patient while lifting a heavy weight presents with sudden onset pain in the lower back radiating along the postero-lateral thigh and lateral leg to the big toe with numbness. The most likely diagnosis is:
(A) L5 fracture
(B) L3-L4 Disc prolapsed
(C) L4-L5 Dis prolapsed
(D) L5-S1 Disc prolapsed

Answer: (C)

717. Which of the following is not recommended in the treatment of Chronic Low Back Pain:
(A) NSAIDS
(B) Bed Rest for 3 months
(C) Exercises
(D) Epidural steroid Injection

Answer: (B)

718. All of the following are included as y ellow flag sings of low back pain, except:
(A) History of systemic steroids use
(B) Reliance on Passive Treatment
(C) Social Isolation
(D) Belief that back pain is severely disabling

Answer: (A)

719. Athlete is sitting on the edge of table with knees flexed at 90 degree. When he extends his knee fully, what will happen to the tibial tuberosity in relation to patella
(A) No change
(B) Movement of TT towards medial border of patella
(C) Movement of TT towards lateral border of patella
(D) Movement of TT towards centre of patella

Answer: (C)

720. In “Bounce home” test of knee joint, end feel is described as all except?
(A) Bony
(B) Empty
(C) Springy
(D) Firm

Answer: (B)

721. Best diagnostic procedure for ant. Cruciate ligament injury is
(A) Lachman’s test
(B) Pivot shift test
(C) Anterior drawer test
(D) Mc Murray’s test

Answer: (A)

722. Lachman test is positive in
(A) Anterior cruciate ligament injury
(B) Posterior Cruciate ligament injury
(C) Medial meniscus injury
(D) Lateral meniscus injury

Answer: (A)

723. What would be the most reliable test for an acutely injured knee of a 27 year old athlete
(A) Anterior drawer test
(B) Posterior drawer test
(C) Lachman test
(D) Steinmann test

Answer: (C)

724. Which one of the following tests will you adopt while examining a knee joint where you suspect an old tear of anterior cruciate ligament?
(A) Posterior drawer test
(B) Mc Murray test
(C) Lachman test
(D) Pivot shift test

Answer: (C)

725. Which of the following is the SAFEST test to be performed in a patient with acutely injured knee joint?
(A) Lachman test
(B) Pivot shift test
(C) Mc Murray’s test
(D) Apley’s grinding test

Answer: (A)

726. In anterior cruciate ligament tear, which of these tests are positive:
(A) Lachman test
(B) McMurry’s test
(C) Anterior drawer test
(D) Posterior drawer test
(E) Apley’s test

Answer: (A)

727. Drawer sign seen in
(A) Cruciate ligament injury
(B) Scurvy
(C) Perthes’s disease
(D) Hyperparathyroidism

Answer: (A)

728. Positive pivot shift test in knee is because of injury to:
(A) Post cruciate ligament
(B) Ant. Cruciate ligament
(C) Medial collateral ligament
(D) Posterior elbow ligament

Answer: (B)

729. Which activity will be difficult to perform for a patient with an anterior cruciate deficient knee joint?
(A) Walk downhill
(B) Walk uphill
(C) Sit cross leg
(D) Getting up from sitting

Answer: (A)

730. Posterior cruciate ligament-true statement is
(A) Attached to the lateral femoral condyle
(B) Intra synovial
(C) Prevents posterior dislocation of tibia
(D) Relaxed in full flexion

Answer: (C)

731. About posterior cruciate ligament true statement is-
(A) Prevent posterior displacement of Tibia
(B) Attaches to lateral femoral condyle.
(C) Intra synovial
(D) Inserted on medial femoral condyle.

Answer: (A)

732. The pivot test is for
(A) Anterior cruciate ligament
(B) Posterior cruciate ligament
(C) Median meniscus
(D) Lateral meniscus

Answer: (A)

733. A twisting injury of knee in flexed position would result in injury to all except
(A) Meniscal tear
(B) Capsular tear
(C) Anterior cruciate ligament
(D) Fibular collateral ligament

Answer: (D)

734. Torsion of knee results in injury most commonly to:
(A) Anterior cruciate ligament
(B) Medial meniscus
(C) Fibular collateral ligament
(D) Tibial collateral ligament

Answer: (D)

735. Structural integrity of collateral ligaments are tested by:
(A) Varus/valgus stress test in full flexion
(B) Varus/valgus stress test in full extension
(C) Varus/valgus stress test in 30° of flexion
(D) Varus/valgus stress test in 90° of flexion

Answer: (C)

736. A patient met with Road traffic Accident with subsequent injury to the left knee. Dial test was positive. What could be the cause:
(A) Medial Collateral Ligament Injury
(B) Medial Meniscal Injury
(C) Lateral Meniscus Tear
(D) Posterolateral Corner Injury

Answer: (D)

737. Physiological locking involves
(A) Internal rotation of femur over stabilized tibia
(B) Internal rotation of tibia over stabilized femur
(C) External rotation of tibia over stabilized femur
(D) External rotation of femur over stabilized tibia

Answer: (A)

738. Which type of injury causes more damage to the semilunar cartilage in the Knee
(A) Flexion and extension at the ankle
(B) Rotation on a flexed knee
(C) Rotation on an extended knee
(D) Squatting position

Answer: (B)

739. Injury to the medial meniscus is rather impossible when the knee joint does not
(A) Extend
(B) Flex
(C) Rotate
(D) Abduct adduct

Answer: (C)

740. The most common type of mode of meniscal injury is
(A) Extension of knee
(B) Flexion of knee
(C) Flexion and rotation movement
(D) Extension and rotation of movement

Answer: (C)

741. Which of the following statements about ‘Menisci’ is not true:
(A) Medial meniscus is more mobile than lateral
(B) Lateral meniscus covers more tibial articular surface than lateral
(C) Medial meniscus is more commonly injured than lateral
(D) Menisci are predominantly made up of type I Collagen

Answer: (A)

742. Medial meniscus of knee joint is injured more often than the lateral meniscus because the medial meniscus is relatively
(A) More mobile
(B) Less mobile
(C) Thinner
(D) Attached lightly to femur

Answer: (B)

743. Bucket handle tear at knee joint is due to:
(A) Injury to medial collateral ligament
(B) Injury to lateral collateral ligament
(C) Injury to ligamentum patellae
(D) Injury to menisci

Answer: (D)

744. Locking of knee joint can be caused by:
(A) Osgood shalter
(B) Loose body in knee joint
(C) Tuberculosis of knee
(D) Medial meniscal partial tear

Answer: (D, B)

745. A patient gives a H/O twisting strain and locking of the knee joint, the most likely diagnosis is:
(A) Avulsion of tibial tubercle
(B) Meniscal tear
(C) Tearing of lateral collateral ligament
(D) Tear of anterior cruciate ligament

Answer: (B)

746. Which of the following correct in medical meniscus tear
(A) Rotation of femur on tibia
(B) Menisci do not heal
(C) Locking and unlocking episodes
(D) Menisci should be excised
(E) All of the about are correct

Answer: (E)

747. It is wise to keep and repair the meniscus rather than removing it when the injury is to which of the following?
(A) Medial part of meniscus
(B) Mid part of meniscus
(C) lateral part of meniscus
(D) Associated with collateral ligament injury

Answer: (C)

748. McMurray’s test is positive in injury of:
(A) Anterior cruciate ligament
(B) Posterior cruciate ligament
(C) Medical meniscus injury
(D) Lateral Meniscus injury
(E) Popliteal bursitis

Answer: (C, D)

749. Mc. Murray’s sign is seen in injury to
(A) Medial meniscus
(B) Medial collateral ligament
(C) Anterior cruciate ligament
(D) Posterior cruciate ligament

Answer: (A)

750. All of the following features in the knee are recognized to be consistent with a torn medial meniscus, except
(A) Excessive forward glide
(B) Giving way
(C) Locking
(D) Mc. Murray’s sign

Answer: (A)

751. A 18 years old boy was playing football, when he suddenly twisted his knee on the ankle and he fell down. He got up after 10 minutes and again started playing, but next day his knee was swollen and he could not move it. The most probable cause is:-
(A) Medial meniscus tear
(B) Anterior cruciate ligament tear
(C) Medical collateral ligament injury
(D) Posterior cruciate ligament injury

Answer: (A)

752. Investigation for injury of knee cartilage
(A) Aspiration
(B) Arthrography
(C) X-ray
(D) Arthroscopy

Answer: (D)

753. Which is the investigation of choice for a sport injury of the knee?
(A) Ultrasonography
(B) Plain radiography
(C) Arthrography
(D) Arthroscopy

Answer: (D)

754. Athletic sustained an injury around the knee joint suspecting cartilage damage, which of the following is an investigation of choice?
(A) Pain X ray
(B) Clinical examination
(C) Arthroscopy
(D) Arthrotomy

Answer: (C)

755. Commonest dangerous complication of posterior dislocation of knee is
(A) Popliteal artery injury
(B) Sciatic nerve injury
(C) Ischaemia of lower leg compartment
(D) Femoral artery injury

Answer: (A)

756. Management of hemarrthrosis
(A) Compression bandage
(B) Needle aspiration
(C) P.O.P cast
(D) Rest to the joint
(E) Synovectomy in recurrent cases

Answer: (ALL)

757. The most common site for ligamentous injuries are those of the
(A) Shoulder joint
(B) Elbow
(C) Knee joint
(D) Ankle joint

Answer: (D)

758. Injury around the ankle joint occur at
(A) Inversion of foot
(B) Eversion of foot
(C) Internal rotation of foot
(D) External rotation of foot

Answer: (A)

759. The most common ligament injured around ankle joint is
(A) Anterior talofibular
(B) Deltoid ligament
(C) Posterior talofibular
(D) Spring ligament

Answer: (A)

760. Most common cause of insertional tendonitis of tendoachilles is:
(A) Overuse
(B) Improper shoe wear
(C) Runner and jumpers
(D) Steroid injections

Answer: (A)

761. Ruptured tendon is most commonly seen in
(A) Stab injury
(B) soft Tissue tumour
(C) Overuse
(D) Congenital defect

Answer: (C)

762. The operative procedure known as “micro-fracture” is done for the
(A) Delayed union of femur.
(B) Non union of tibia
(C) Loose bodes of ankle joint
(D) Osteochondrial defect of femur

Answer: (D)

763. Indications of arthoplasty:
(A) Osteoarthritis
(B) Rheumatoid arthritis
(C) Ankylosingspondylosis
(D) Gout
(E) Fracture neck femur

Answer: (ALL)

764. Major indication (s) for arthroplasty:
(A) Osteoarthritis of hip
(B) Ankylosis of elbow
(C) Ununitedtibial fracture
(D) Ununited femoral neck fracture
(E) TB spine

Answer: (A, B, D)

765. Metal on Metal articulation should be avoided in:
(A) Osteonecrosis
(B) Young female
(C) Inflammatory arthritis
(D) Revision surgery

Answer: (B)

766. Aseptic loosening in cemented total hip replacement, occurs as a result of hypersensitivity response to:
(A) Titanium debris
(B) High density polythene debris
(C) N. N-Dimethyltryptamine (DMT)
(D) Free radicals

Answer: (B)

767. Watson Jones operation is done for?
(A) Neglected Club foot
(B) Muscle paralysis
(C) Valgus deformity
(D) Hip replacement

Answer: (D)

768. Watson Jones operation is done for:
(A) Polio
(B) Muscle paralysis
(C) Neglected clubfoot
(D) Chronic ankle instability.

Answer: (D)

769. A patient developed breathlessness and chest pain, on second postoperative after a total hip replacement. Echo-cardiography showed right ventricular dilatation and tricuspid regurgitation. What is the most likely diagnosis.
(A) Acute MI
(B) Pulmonary embolism
(C) Hypertensive shock
(D) Cardiac tamponade

Answer: (B)

770. What is the most common cause of death after total hip replacement?
(A) Infection
(B) Deep vein thrombosis
(C) Pulmonary embolism
(D) Pneumonia

Answer: (C)

771. A sixty five year old lady presents with a long standing history of pain and swelling in her right knee. Pain is significantly interfering with her activities of daily living. Radiological evaluation shows grade III changes of osteoarthritis. Which of the following is the recommended line of management?
(A) Conservative management
(B) Arthroscopic lavage washout
(C) Partial knee replacement
(D) Total knee replacement

Answer: (D)

772. A 68 yr old man came with pain and swelling of right knee. Ahlbeck grade 2 osteoarthritic changes were found on investigation. What is the further management:
(A) Conservative
(B) Arthroscopic washout
(C) High tibial osteotomy
(D) Total knee replacement

Answer: (D)

773. All of the following statements about High Tibial osteotomy are true, Except:
(A) Magnitude of correction achieved is greater than 30°
(B) Indicated in Unicompartmental osteoarthritis
(C) Performed through cancellous bone
(D) Recurrence is a long term complication

Answer: (A)

774. Site for 1st order bone grafting:
(A) Pelvis
(B) Tibial metaphysis
(C) Medial malleolus
(D) Femoal condyle
(E) Greater trochanter

Answer: (A)

775. Standard site of primary bone graft:
(A) Pelvis
(B) Greater trochanter
(C) Medial malleolus
(D) Lateral malleolus
(E) Fibula

Answer: (A)

776. Cancellous bone graft taken from:
(A) Femoral condyles
(B) Pelvis
(C) Greater trochanter
(D) Tibial metaphysic

Answer: (ALL)

777. Which of the following is ideal site for harvesting bone graft?
(A) Iliac crest
(B) Distal end of humerus
(C) Distal end of femur
(D) Fibula

Answer: (A)

778. Which of the following is/are not included in management of intra-articular fracture:
(A) Arthrodesis
(B) Excision
(C) Aspiration
(D) K Wire
(E) Plaster of Paris cast

Answer: (NONE)

779. Usual treatment (s) of fracture involving articular surface is/are
(A) Arthrodesis
(B) Excision
(C) Skeletal traction
(D) Internal fixation
(E) P.O.P slab

Answer: (ALL)

780. Which of the following condition should be given most priority in case of fracture:
(A) Open fracture
(B) Dislocated fracture
(C) Vascular injury
(D) Malunited fracture
(E) Compartment syndrome

Answer: (A, B, C, E)

781. In the management of long bone fracture following can be done:
(A) Intramedullary nailing
(B) Plating
(C) External fixation
(D) Tension band wiring
(E) Screw

Answer: (A, B, C)

782. Long bone # fixation done with:
(A) Intramedullary Nail
(B) Compression plate
(C) External fixation
(D) Screw
(E) Tension band wiring

Answer: (A, B, C)

783. Action of intramedullary ‘K’ nail is
(A) Two-point fixation
(B) Three-point fixation
(C) Compression
(D) Weight concentration

Answer: (B)

784. All of the following are indications for open reduction and internal fixation of fractures except
(A) Compound fracture
(B) Unsatisfactory closed reduction
(C) Multiple trauma
(D) Intra-articular fracture

Answer: (A)

785. Which fracture in children requires open reduction:
(A) Fracture tibial epiphysis
(B) Fracture shaft of femur
(C) Fracture both bones forearm
(D) Fracture femoral condyle

Answer: (A)

786. Open reduction & internal fixation is done for all of the following #s except:
(A) Patella #
(B) Olecranon #
(C) Volar Barton’s #
(D) # Lateral condyle of humerus

Answer: (C)

787. Treatment of choice for fracture lower 1/4th of tibia in non-union with multiple scarred wounds and discharging sinuses and about 4 cm shortening of leg?
(A) Ilizarov fixator
(B) Plate
(C) External fixation
(D) Intramedullary nail

Answer: (A)

788. Surgical excision is contraindicated in:
(A) Olecranon process
(B) Patella
(C) Head of radius
(D) Lateral condyle humerus

Answer: (D)

789. Contraindication for skin traction:
(A) Dermatitis
(B) Vascularity compromised status of limb
(C) Abrasions
(D) Hypopigmentation (vitiligo)
(E) Bony deformity

Answer: (A, B, C)

790. Gallows traction is used in fracture of:
(A) Head of femur
(B) Neck of femur
(C) Shaft of femur
(D) Humerus

Answer: (C)

791. Skeletal traction is given by:
(A) K-wire
(B) Pavlik harness
(C) Denham Pin
(D) Steinmann’s pin
(E) Rush pin, Cancellos screw

Answer: (A, D)

792. All of the following are used for giving skeletal traction, except:
(A) Steimann’s pin
(B) Kirschner’s wire
(C) Bohler’s stirrup
(D) Rush pin

Answer: (D)

793. During reconstruction of an amputated limb, which of the following is done first:
(A) Vein ligation
(B) Artery ligation
(C) Nerve ligation
(D) Fixation of bone

Answer: (D)

794. Which of the following structures is fixed first during implantation of an amputated digit:
(A) Bone
(B) Artery
(C) Vein
(D) Nerve

Answer: (A)

795. Pollicization is
(A) Amutation of thumb
(B) Equalization of fingers
(C) Toe to thumb transplantation
(D) Reconstruction of thumb

Answer: (D)

796. Which is not a deep heat therapy
(A) Short wave diathermy
(B) Ultrasound therapy
(C) Infrared therapy
(D) Microwave therapy

Answer: (C)

797. Consider the following nerve injuries
(1) Axonotmesis
(2) Neurotmesis
(3) Neuropraxia the correct sequence of these injuries in terms of ‘increasing’ severity of damage to the nerve fibres and sheath, is
(A) 1, 2 and 3
(B) 3, 2 and 1
(C) 2, 1 and 3
(D) 3, 1 and 2

Answer: (D)

798. Rate of regeneration of severed nerve is
(A) 0.1 mm/day
(B) 1 mm/day
(C) 1 cm/day
(D) None

Answer: (B)

799. “Tinel’s sign” indicates
(A) Neurofibroma
(B) Injury to peripheral nerves
(C) Atrophy of nerves
(D) Regeneration of nerves

Answer: (D)

800. Tourniquet paralysis is an unfortunate complication leads to
(A) Neuropraxia
(B) Axonotmesis
(C) Neurotmesis
(D) None of the above

Answer: (A)

801. Nerve suturing in a clean cut injury is done best in
(A) 6 hours
(B) 12 hours
(C) After one day
(D) After two day

Answer: (A)

802. In Seddon’s classification, complete division of nerve is
(A) Neuropraxia
(B) Axonotmesis
(C) Neurotmesis
(D) None of the above

Answer: (C)

803. Following indicate better prognosis is in injury except
(A) Neuroproxia
(B) Younger age
(C) Pure motor nerve injury
(D) Proximal injury

Answer: (D)

804. Prognosis after secondary nerve suturing is better in pure than in mixed ones. Based on this criterion, which one of the following nerves should be given the best result after suturing in identical conditions?
(A) Common peroneal nerve
(B) Radial nerve
(C) Ulnar nerve
(D) Median nerve

Answer: (B)

805. Best results of nerve suturing are seen in
(A) Median nerve
(B) Ulnar nerve
(C) Sciatic nerve
(D) Radial nerve

Answer: (D)

806. All are true regarding brachial plexus injury, except:
(A) Preganglionic lesions have a better prognosis than postganglionic lesions
(B) Erb’s palsy causes paralysis of the abductors and external rotators of the shoulder
(C) In Klumpke’s palsy, Horner’s syndrome may be present on the ipsilateral side
(D) Histamine test is useful to differentiate between the preganglionic and postganglionic lesions

Answer: (A)

807. Most common cause of neurological deficit in upper limb is
(A) Polio
(B) Erb’s palsy
(C) C1-C2 dislocation
(D) Fracture dislocation of cervical spine

Answer: (B)

808. A pole vaulter had a fall during pole vaulting and had paralysis of the arm. Which of the following investigations gives the best recovery prognosis:
(A) Electromyography
(B) Muscle biopsy
(C) Strength Duration Curve
(D) Creatine phosphokinase levels

Answer: (C)

809. A 20 year old male involved in single vehicle rollover accident presented with inability to abduct the shoulder and flex the elbow. There was associated wasting of the supraspinatus, deltoid and biceps muscle. Which of the following is the most likely diagnosis:
(A) Shoulder Dislocation
(B) Upper Trunk Injury
(C) Lateral Cord Injury
(D) Medial Cord Injury

Answer: (B)

810. In a full term newly born male baby, on routine check up soon after delivery has weak Moro’s reflex on right hand but intact Grasp’s reflex with weak abduction and supination movement of hand is found. This will suggest that most likely injury would be at
(A) C5, 6nerve
(B) C7, C8 nerve
(C) C8, T1 nerve
(D) T1, T2 nerve

Answer: (A)

811. The nerve roots involved in Erb’s palsy are
(A) C3 C4
(B) C4 C5
(C) C5 C6
(D) C6 C7

Answer: (C)

812. Which of the following statement is wrong about Erb’s palsy?
(A) Abductors of the shoulder are weak
(B) Upper part of brachial plexus is involved
(C) Supinators are normal
(D) Internal rotation of arm

Answer: (C)

813. Erb’s point is located at the junction of
(A) Anterior primary rami of C5 and C6
(B) Anterior primary rami of C8 and T1
(C) Superior and middle trunk of brachial plexus
(D) None of the above

Answer: (A)

814. Erb’s palsy is due to involvement of
(A) Upper trunk of brachial plexus
(B) Middle trunk of brachial plexus
(C) Lower trunk of brachial plexus
(D) Long thoracic nerve

Answer: (A)

815. A young boy who was driving motorcycle at a high speed collided with a tree & was thrown on his right shoulder. Though there was no fracture, his right arm was medially rotated and forearm pronated. The following facts concerning this patient are correct, except
(A) The injury was at Erb’s point
(B) A lesion of C5 and C6 was present
(C) The median and ulnar nerves were affected
(D) Supraspinatus, Infraspinatus, Subclavius& Biceps brachii were paralyzed

Answer: (C)

816. All of the following muscles undergo paralysis after injury to C5 and C6 spinal nerves except:
(A) Biceps.
(B) Corachobrachialis.
(C) Brachialis.
(D) Brachioradialis.

Answer: (D, B)

817. A 19 year old boy fell from the motar bike on his shoulder. he doctor diagnosed him a case of Erbs paralysis. The following signs and symptoms will be observed except:
(A) Loss of abduction at shoulder joint
(B) Loss of lateral rotation
(C) Loss of pronation at radioulnar joint.
(D) Loss of flexion at elbow joint.

Answer: (C)

818. Lower branch of brachial plexus injury leads to
(A) Erb’s palsy
(B) Klumpke’s palsy
(C) Bell’s palsy
(D) Wrist drop

Answer: (B)

819. The lesion in Klumpke’s paralysis is at
(A) Cervical plexus
(B) Lower brachial
(C) Upper brachial
(D) Sacral plexus

Answer: (B)

820. The mode of injury of Klumpke’s paralysis is
(A) Forcible increase in the neck shoulder angle (As during child birth)
(B) Forcible rotation of upper arm
(C) Forcible hyperabduction of upper arm (falling person trying to catch an object)
(D) Fracture shaft humerus

Answer: (C)

821. A 45 years male present with abrupt onset, pain weakness, loss of contour of shoulder and wasting of muscle of arm on 5th day of tetanus toxoid immunization in deltoid. Likely cause is
(A) Radial nerve entrapment
(B) Thoracic outlet syndrome
(C) Brachial neuritis
(D) Hysteria

Answer: (C)

822. All of the following are features of musculocutaneous nerve injury at axilla except:
(A) Loss of flexion of shoulder
(B) Loss of flexion at elbow
(C) Loss of supination of forearm
(D) Loss of sensation on radial side of forearm

Answer: (A)

823. All of the following features can be observed after the injury to axillary nerve, except:
(A) Loss of rounded contour of shoulder
(B) Loss of sensation along lateral side of upper arm
(C) Loss of overhead abduction
(D) Atrophy of deltoid muscle

Answer: (C)

824. Which of the following nerve injuries produce the deformities?
(I) Upper trunk I. Wrist drop
(II) Ulnar nerve II. Porter’s tip hand
(III) Axillary nerve III. Claw hand
(IV) Radial nerve IV. Flatterning of the shoulder
(A) I(i), II(ii), III(iii), IV(iv)
(B) I(ii), II(iii), III(iv), IV(i)
(C) I(iii), II(iv), III(ii), IV(i)
(D) I(iv), II(iii), III(ii), IV(i)

Answer: (B)

825. Match the followings:
(I) Erb’s paralysis I. Lower trunk
(II) Klumpke’s paralysis II. Axillary nerve injury
(III) Crutch paralysis III. Radial nerve injury
(IV) Fractures surgical neck humerus IV. Upper trunk injury
(A) I(iv), II(iii), III(ii), IV(i)
(B) I(iii), II(ii), III(iv), IV(i)
(C) I(iv), II(i), III(iii), IV(ii)
(D) I(i), II(iii), III(iv), IV(ii)

Answer: (C)

826. Match the following
(I) Erb’s paralysis
(II) Klumpke’s paralysis
(III) Crutch paralysis
(IV) Fracture surgical neck humerus
(i) Lower trunk injury
(ii) Axillary nerve injury
(iii) Radial nerve injury
(iv) Upper trunk injury
(A) I(iv), II(iii), III(ii), IV(i)
(B) I(iii), II(ii), III(iv), IV(i)
(C) I(iv), II(i), III(iii), IV(ii)
(D) I(i), II(iii), III(iv), IV(ii)

Answer: (C)

827. A patient presented with claw hand after a Supracondylar fracture was reduced and plaster applied. The diagnosis is:
(A) Medial nerve injury
(B) Volkmann’s ischaemic contracture
(C) Ulnar nerve injury
(D) Dupuytren’s contracture

Answer: (B)

828. Dislocation of which one of the following carpal bones can present as medal nerve palsy?
(A) Scaphoid
(B) Hamate
(C) Lunate
(D) Trapezium

Answer: (C)

829. Injury of median nerve at wrist is best detected by
(A) Action of abductor pollicisbrevis
(B) Action of flexor pollicisbrevis
(C) Loss of sensation of radial half of palm
(D) Loss of sensation of tip of ring finger

Answer: (A)

830. Pointing index sign in seen in __ nerve palsy
(A) Ulnar
(B) Radial
(C) Median
(D) Axillary

Answer: (C)

831. Anterior interosseous nerve is a branch of
(A) Musculocutaneous
(B) Radial
(C) Median
(D) Ulnar

Answer: (C)

832. Median nerve is injured during
(A) Elbow dislocation
(B) Fracture lateral epicondyle of humerus
(C) Fracture medial epicondyle of humerus
(D) Supracondylar fracture of humerus

Answer: (A, D)

833. Median nerve injury at the wrist causes
(A) Claw hand
(B) Loss of apposition of thumb
(C) Policeman’s tip deformity
(D) Saturday night palsy

Answer: (B)

834. ‘Ape thumb deformity’ is observed in lesions of
(A) Radial nerve injury
(B) Ulnar nerve injury
(C) Median nerve injury
(D) Circumflex humeral nerve injury
(E) Posterior interosseous nerve injury

Answer: (C)

835. Compression of a nerve within the carpal tunnel products inability to:
(A) Abduct the thumb
(B) Adduct the thumb
(C) Flex the distal phalanx of the thumb
(D) Oppose the thumb

Answer: (D)

836. Median nerve lesion at the wrist causes all of the following, Except:
(A) Thenar Atrophy
(B) Weakness of adductor Pollicis
(C) Weakness of 1st and 2ndLumbricals
(D) Weakness of Flexor PollicisBrevis

Answer: (B)

837. Median nerve injury at wrist, is commonly tested by:
(A) Contraction of abductor pollicisbrevis
(B) Contraction of flexor pollicisbrevis
(C) Loss of sensation on palm
(D) Loss of sensation on ring finger

Answer: (A)

838. A cut injury on wrist causes inability of thumb to touch the tip of little finger, the nerve likely to be damaged is:
(A) Ulnar
(B) Radial
(C) Median
(D) Deep branch of ulnar nerve

Answer: (C)

839. A boy presents with complaints of hypoaesthesia and wasting of thenar eminence. The nerve most likely to damaged in this patient:
(A) Musculocutaneous nerve
(B) Median nerve
(C) Ulnar nerve
(D) Radial nerve

Answer: (B)

840. Nerve damaged due to lunate dislocation (in carpal tunnel):
(A) Median & ulnar
(B) Median
(C) Ulnar
(D) Radial

Answer: (B)

841. Ape thumb deformity is seen in involvement of :
(A) Median nerve
(B) Ulnar nerve
(C) Radial nerve
(D) Axillary nerve

Answer: (A)

842. Tardy ulnar nerve palsy is due to:
(A) Cubitus valgus
(B) Fixation of nerve in the groove by osteoarthritis
(C) Excision of elbow joint
(D) Fracture of internal condyle

Answer: (A)

843. Trady ulnar nerve palsy is seen with
(A) Lateral humeral condyle fracture
(B) Supracondylar fracture
(C) Medial humeral condyle fracture
(D) Fracture capitulum

Answer: (A)

844. The “Card test” tests the function of
(A) Median nerve
(B) Ulnar nerve
(C) Axillary nerve
(D) Radial nerve

Answer: (B)

845. Froment’s sign in present in paralysis of ____ injury
(A) Ulnar
(B) Radial
(C) Median
(D) Axillary

Answer: (A)

846. Claw hand is seen in
(A) Cervical rib Carpal tunnel syndrome
(B) Multiple scelerosis
(C) Radial nerveinjury
(D) Ulnar nerve injury

Answer: (D)

847. Following an incised wound in the front of wrist, the subject is unable to oppose the tips of the little finger and the thumb. The nerve (s) involved is/are
(A) Ulnar nerve alone
(B) Median nerve alone
(C) Median nerve ulnar nerves
(D) Radial and ulnar nerves

Answer: (C)

848. “Ulnar paradox’ is related with the following
(A) Lumbricals
(B) Intrinsic muscle
(C) EPL
(D) Ulnar half of FDP

Answer: (B)

849. Ulnar nerve injury at wrist results in
(A) Unable to grasp card between the fingers
(B) Claw hand
(C) Inability to abduct or adduct the fingers
(D) All of the above

Answer: (D)

850. Disability of hands is maximum with a lesion of
(A) Median nerve at elbow
(B) Median nerve at wrist
(C) Ulnar nerve at elbow
(D) Ulnar nerve at wrist
(E) Radial nerve lesion

Answer: (D)

851. Ulnar nerve severed above elbow causes
(A) Complete loss of sensation in 4th and 5th finger
(B) Paralysis of all lumbricals
(C) Paralysis of all interosseia
(D) Paralysis of flexor carpi ulnaris
(E) Paralysis of flexor profundus

Answer: (B, C, D, E)

852. A factory worker has laceration proximal to wrist with no opposition of thumb and fingers, loss of sensation over lateral aspect of thumb and fingers. Diagnosis is injury to
(A) Median nerve + Ulnar nerves
(B) Median + Radial nerves
(C) Radial + Ulnar nerves
(D) Median nerve only

Answer: (A)

853. Claw hand occurs in
(A) Volkmann’s ischaemic Contracture
(B) Dupuytren’s contracture
(C) Ulnar nerve injury
(D) Sudeck’s atrophy

Answer: (C)

854. Froment’s sign is characteristically seen in
(A) Ulnar nerve injury
(B) Median nerve injury
(C) Radial nerve injury
(D) Intercostobrachial nerve injury

Answer: (A)

855. Froment’s sign is positive due to paralysis of
(A) Opponenspollicis
(B) Abductor pollicis
(C) Adductor pollicis
(D) Adductor halluces longus

Answer: (C)

856. A boy presents with injury to medial epicondyle of the humerus. Which of the following would not be seen
(A) Weakness of the ulnar deviation and flexion
(B) Complete paralysis of the IIrd and IVth digits
(C) Atrophy of the hypothenar eminence.
(D) Decreased sensation of the hypothenar eminence.

Answer: (B)

857. All of the following muscles are paralyzed in trauma to median nerve at the wrist except
(A) Adductor pollicis
(B) Abductor pollicis
(C) First two lumbicals
(D) Muscles of the Thenar eminence

Answer: (A)

858. Ulnar nerve injury at wrist involves following except:
(A) Plamarinterossei
(B) Opponenspollicis
(C) Dorsal interossei
(D) Adductor pollicis

Answer: (B)

859. Low ulnar nerve palsy is characterized by:
(A) Claw hand
(B) Sensory loss of medial four digits
(C) Weakness of grips
(D) Inability to extend at M.C.P. joint
(E) Inability to abduct the thumb

Answer: (A, C)

860. Injury to the Ulnar nerve at the wrist causes paralysis of
(A) Apposition of the thumb
(B) Abduction of the carpo-metacarpal joint of the thumb
(C) Adduction of the thumb
(D) Flexion of the MCP joint of the middle finger

Answer: (C)

861. Claw hand is caused by lesion of –
(A) Ulnar nerve
(B) Median nerve
(C) Axillary nerve
(D) Radial nerve

Answer: (A)

862. In a patient with Claw Hand due to Leprosy, the deformity would be classified as:
(A) Grade 0
(B) Grade I
(C) Grade II
(D) Grade III

Answer: (C)

863. A patient with leprosy presents with clumsiness of hand. His ulnar nerve is affected. Clumsiness is due to palsy of which muscle:
(A) Extensor carpi ulnaris
(B) Abductor pollicisbrevis
(C) Oppenenspollicis
(D) Inerosseous muscle

Answer: (D)

864. A patient comes to the emergency department after alcohol binge previous night and sleeping on arm chair. In the morning he is unable to move his hand and diagnosis of ulnar nerve palsy is made. What is the next line of management
(A) Wait and Watch
(B) Knuckle Bender Splint
(C) Immediate exploration of nerve
(D) Do EMG study after 2 days

Answer: (B)

865. A person is not able to extend his metacarpophalangeal joint. This is due injury to which nerve
(A) Ulnar nerve
(B) Radial nerve injury
(C) Medial nerve injury
(D) Post. Introsseus nerve injury

Answer: (D)

866. A 30-year-old male underwent excision of the right radial head. Following surgery, the patient developed inability to extend the fingers and thumb of the right hand. He did not have any sensory deficit. Which one of the following is the most likely cause?
(A) Injury to Posterior interosseus nerve
(B) Iatrogenic injury to common extensor origin
(C) Injury to anterior interosseus nerve
(D) High radial nerve palsy

Answer: (A)

867. Which of the following statements about ‘Low’ Radial nerve palsy is not true:
(A) Loss of nerve supply to brachioradialis
(B) Loss of nerve supply to extensor carpi radialisbrevis
(C) Loss of nerve supply to extensor pollicisbrevis
(D) Loss of sensation over first dorsal web space

Answer: (A)

868. Commonest cause of wrist drop is
(A) Intramuscular injection
(B) Fracture humerus
(C) Dislocation of elbow
(D) Dislocation of shoulder

Answer: (B)

869. Injury to which of the following nerves is most likely to result in Wrist drop?
(A) Radial
(B) Ulnar
(C) Median
(D) Musculocutaneous

Answer: (A)

870. Saturday night palsy involves nerve
(A) Radial
(B) Ulnar
(C) Median
(D) Axillary
(E) Any of the above

Answer: (A)

871. In fracture of distal half of humerus, the nerve injured is
(A) Axillary
(B) Median
(C) Radial
(D) Ulnar

Answer: (C)

872. After injury to radial nerve in radial groove, first sign of recovery is
(A) Tinel’s sign
(B) Power in extensor corpiradialislongus
(C) Power in Brachioradialis
(D) Power in Triceps

Answer: (A)

873. Feature of radial nerve injury at spiral groove
(A) No wrist drop
(B) Sensory loss over deltoid
(C) Thumb, finger, wrist drop
(D) Wrist droop + Extensors of Forearm paralysis

Answer: (C)

874. Radial nerve is most commonly injured in fracture of
(A) Mid shaft of humerus
(B) Neck of humerus
(C) Lateral condyle of humerus
(D) Lower 1/3rd of humerus

Answer: (A)

875. Writ drop results in injury of
(A) Ulnar nerve
(B) Median nerve
(C) Posterior interosseus nerve
(D) Radial nerve

Answer: (D)

876. Following structures pass through spiral groove of humerus
(A) Radial nerve
(B) Radial artery
(C) Anterior circumflex humeral artery
(D) Posterior circumflex humeral artery

Answer: (A, E)

877. Injury to radial nerve – mid humerus level following occurs:
(A) Paralysis of extensors of hand & wrist
(B) Equally diagnosed by sensory and motor testing
(C) Can be approached by splitting latissimusdorsi
(D) Paralysis of Triceps

Answer: (A)

878. Damage to the radial nerve in the spinal groove spares which muscle
(A) Lateral head of triceps
(B) Long head of triceps
(C) Medial head of triceps
(D) Anconeus

Answer: (B)

879. Injury to radial nerve in lower part of spiral groove:
(A) Spares nerve supply to extensor carpi radialislongus
(B) Results in paralysis of anconeus muscle
(C) Leaves extensions at elbow joint intact
(D) Weakness pronation movement

Answer: (C)

880. Cock up splint is used in management of
(A) Ulnar nerve palsy
(B) Brachial plexus palsy
(C) Radial nerve palsy
(D) Combined ulnar & Median nerve palsy

Answer: (C)

881. Non dynamic splint is:
(A) Banjo
(B) Opponens
(C) Cock up
(D) Brand

Answer: (C)

882. Cock up splint is used in:
(A) Median nerve injury
(B) Radial nerve injury
(C) Ulnar nerve injury
(D) Volkman’s ischemic contracture.

Answer: (B)

883. A 25-yr-old lady sustained a lacerated wound on the back of right thigh by a horn of a bull. The wound was sutured two months later she developed foot drop and an ulcer on the dorsum of the foot. The most likely diagnosis is
(A) Chronic ischaemia to limbs due to Popliteal artery injury
(B) Partial injury to sciatic nerve
(C) Complete division of sciatic nerve
(D) Injury to hamstring muscles

Answer: (B)

884. Section of lateral Popliteal nerve at the neck of fibula causes:
(A) Foot drop
(B) Trophic ulcers on toes
(C) Atrophy of calf muscle
(D) Pain in the leg

Answer: (A)

885. Foot drop result because of injury to
(A) Superficial peroneal nerve
(B) Deep peroneal nerve
(C) Posterior tibial nerve
(D) Anterior tibial nerve

Answer: (B)

886. Sudden hyperflexion of high over abdomen (Mcrobert’s procedure), which of the following nerve is commonly involved:
(A) Common peroneal nerve
(B) Obturator nerve
(C) Lumbosacral trunk
(D) Lateral cutaneous nerve of thigh.

Answer: (D)

887. Injury to the common peroneal nerve at the lateral aspect of head of fibula results in all of the following except:
(A) Weakness of ankle dorsi-flexion.
(B) Foot drop.
(C) Loss of ankle reflex.
(D) Sensory impairment on lateral aspect of leg extending to the dorsum of foot.

Answer: (C)

888. Common peroneal nerve is related to
(A) Shaft of tibia
(B) Neck of fibula
(C) Lower tibio-fibular joint
(D) Shaft of fibula

Answer: (B)

889. Carpel tunnel syndrome is due to compression of :
(A) Radial nerve
(B) Ulnar nerve
(C) Palmer branch of the Ulnar nerve
(D) Median nerve

Answer: (D)

890. Which of the following does NOT predispose to carpal tunnel syndrome
(A) Hypertension
(B) Hypothyroidism
(C) Pregnancy
(D) Acromegaly

Answer: (A)

891. True about carpel tunnel syndrome:
(A) Paraesthesia of the lateral 2 fingers
(B) Atropy of the thenar eminence
(C) Atropy of the hypothenar eminence
(D) Claw hand
(E) Tinel sign is positive

Answer: (A, E)

892. Phalen’s test is positive in
(A) Carpal tunnel syndrome
(B) De Quervain’s disease
(C) Tennis elbow
(D) Ulnar bursitis

Answer: (A)

893. Consider the following statements about Carpal tunnel syndrome
(1) It may occur in Acromegaly
(2) It may occur in pregnancy
(3) It causes delayed ulnar nerve conduction
(4) It may be associated with wasting of abductor pollicisbrevis of these statements
(A) 1, 2 and 3 are correct
(B) 2, 3 and are correct
(C) 1, 2 and 4 are correct
(D) 1, 3 and 4 are correct

Answer: (C)

894. In carpal tunnel syndrome, features are of
(A) Compression of ulnar nerve
(B) Compression of median nerve under the flxor retinaculum
(C) Anaesthesia over thenar eminence
(D) Atrophy of hypothenar muscles

Answer: (B, C)

895. Carpal tunnel syndrome can be caused by the following except?
(A) Diabetes Mellitus
(B) Amyloidosis
(C) Addison’ Disease
(D) Hypothyroidism

Answer: (C)

896. True about carpal tunnel syndrome is A/E
(A) More common in males
(B) Acromegaly can cause
(C) Positive tinel’s sign
(D) Nerve conduction study done
(E) Transverse carpal ligament are cut to relieve symptom

Answer: (A)

897. A 52 year old lady complaints of nocturnal pain over the thumb, index finger and middle finger. Which of the following tests is not required to reach a diagnosis:
(A) Phalen’s test
(B) Finkelstein test
(C) Tinel’s sign
(D) Tourniquet test

Answer: (B)

898. True about carpal tunnel syndrome:
(A) Occur in pregnancy
(B) Affects medial 3½ finger
(C) Associated with Hypothyroidism
(D) Froment sign positive
(E) Median nerve involvement is present

Answer: (A, C, E)

899. Entrapment neuropatheis commonly affect the following nerves except:
(A) Tibial
(B) Femoral
(C) Lateral cutaneous nerve of thigh
(D) Common digital nerve

Answer: (B)

900. MeralgiaParaesthetic involves
(A) Axillary nerve
(B) Sural nerve
(C) Median nerve
(D) Lateral cutaneous nerve of high

Answer: (D)

901. Most common cause of tarsal tunnel syndrome:
(A) Osteoarthritis
(B) Ankylosing spondylitis
(C) Psoriatic arthritis
(D) Rheumatoid arthritis

Answer: (D)

902. Commonest cause for neuralgic pain in foot is:
(A) Compression of communication between medial and lateral planter nerves
(B) Exaggeration of longitudinal arches
(C) Injury to deltoid ligament
(D) Shortening of planter aponeurosis

Answer: (A)

903. In a patient with a history of burning pain localized to the plantar aspect of the foot, the differential diagnosis must include:
(A) Peripheral vascular disease
(B) Tarsal coalation
(C) Tarsal tunnel syndrome
(D) Planter fibromatosis

Answer: (C)

904. The classical flexion and rotation deformities at hip and knee joint, as a sequelae of poliomyelitis, are due to the contracture of
(A) Tensor fascia lata
(B) Gastrocnemius
(C) Tendo Achilles
(D) Hamstrings

Answer: (A)

905. All are possible during contraction of Tensor fasciae latae except
(A) Adduction of Hip joint
(B) Flexion of Hip joint
(C) Abduction of Hip joint
(D) Flexion of the knee

Answer: (A)

906. In a post polio case, ilio-tibial tract contracture is likely to result in:
(A) Extension at the hip and knee
(B) Extension at the hip
(C) Flexion at the hip and the knee
(D) Extension at the knee

Answer: (C)

907. Test for tight iliotibial band is:
(A) Ober’s test
(B) Osber’s test
(C) Simmand’s test
(D) Charnley’s test

Answer: (A)

908. Post Poliomyelitis, a patient has grade II power in Gastrocnemius, grade III is Peroneus, grade IV in Tibialis Anterior. The deformity is
(A) Calcaneovalgus
(B) Equinovarus
(C) Calcaneovarus
(D) Genu valgus

Answer: (C)

909. In 3 year child with polio paralysis, tendon transfer operation is done at
(A) 2 months after the disease
(B) 2 years after the disease
(C) 6-12 months after the disease
(D) After skeleton maturation

Answer: (B)

910. You have treated the simple and undisplaced fracture of shaft of right tibia in a nine year girl with above knee plaster cast. Parents want to know the prognosis of union of the fractured limb which was affected by poliomyelitis four years ago. What is the best possible advice will you offer to the parents?
(A) Fracture will unite slowly
(B) Fracture will not unite
(C) Fracture will unite normally
(D) Fracture will unite on attaining puberty

Answer: (C)

911. Stance phase muscle among the following is
(A) Quadriceps
(B) Hamsring
(C) Gastrocnemius-soleus
(D) Tibialis anterior
(E) Peroneus longus

Answer: (A)

912. Which of the commonest fracture in children?
(A) Fracture clavicle
(B) Supracondylar fracture
(C) Green stick fracture of lower end of radius
(D) All of the above

Answer: (C)

913. Which statements pertaining to green stick fracture is except?
(A) Any fracture (#) in child
(B) Is generally incomplete
(C) # only in rickety children
(D) All of the above

Answer: (B)

914. Which of the following statements pertaining to green stick fracture is true?
(A) Any fracture in a child
(B) Fracture only in rickety children
(C) Only if there is no deformity
(D) All of the above

Answer: (C)

915. A 6-year-old child falls in right-sided forearm region and develops fracture in dorsal surface of mid region of radius. The best treatment is
(A) Antibiotics & sedative
(B) Bone plating and external fixation
(C) Slab with wait for bone remodeling
(D) Break the cortex other side and immobilization by POP

Answer: (D)

916. In children, all are true except:
(A) Dislocations are rare
(B) Comminuted fractures are common
(C) Thick periosteum
(D) Soft bones

Answer: (B)

917. In children best remodeling is seen in fracture with
(A) Angulation in diaphysis
(B) Angulation in metaphysis
(C) Rotation in diaphysis
(D) Rotation in metaphysic

Answer: (B)

918. An 8yrs child is brought by parents to the casualty with a spiral fracture of Femur and varying degree of Ecchymosis all over body. The Etiology is:
(A) Hit & run accident
(B) Battered Baby Syndrome
(C) Hockey Stick injury
(D) Osteogenicimperfecta

Answer: (B)

919. A one year old child presented with multiple fractures seen in various stages of healing. The most probable diagnosis in the case is:
(A) Scurvy
(B) Rickets
(C) Battered baby syndrome
(D) Fall from height

Answer: (C)

920. Type VI Rang’s injury induces
(A) Transverse fracture of metaphysic with longitudinal extension into physis.
(B) Open injury with loss of physis.
(C) Thurston Holland’s sign
(D) Perichondrial ring injury
(E) Intraarticular fractures

Answer: (D)

921. Perichondrial ring is
(A) Seen around foramen magnum
(B) Seen around epiphyseal plate
(C) More prominent in adults
(D) Shear strength increases with age
(E) Seen around all fetal diaphysis and joint cavity

Answer: (B)

922. An 8 year old boy with a history of fall from 10 feet height complains of pain in the right ankle. X-ray taken at that time are normal without any fracture line. But after 2 years, he developed a calcaneovalgus deformity. The diagnosis is
(A) Undiagnosed malunited fracture
(B) Avascular necrosis talus
(C) Tibial epiphyseal injury
(D) Ligamentous injury of ankle joint

Answer: (C)

923. Epiphyseal enlargement occurs in
(A) Paget’s disease
(B) Sheurmann’s disease
(C) Epiphyseal dysplasia
(D) Hemophilia

Answer: (D)

924. Epiphyseal dysgenesis is a feature of
(A) Hyperparathyroidism
(B) Hypoparathyroidism
(C) Hypothyroidism
(D) Hyperthyroidism

Answer: (C)

925. Phocomelia is best described as
(A) Defect in development of long bones
(B) Defect in development of flat bones
(C) Defect of intramembranous ossification
(D) Defect of cartilage replacement by bone

Answer: (A)

926. Phocomelia is
(A) Absence of short bones
(B) Complete absence of extremities
(C) Defects of long bones of limb
(D) Partial absence of extremities

Answer: (C)

927. Madelung’sdeoformity involves the following:
(A) Knee
(B) Wrist
(C) Hip
(D) Elbow

Answer: (B)

928. Cause of congenital pseudoarthrosis is
(A) Intrauterine fracture
(B) Neurofibromatosis
(C) Fibrous dysplasia
(D) Unknown

Answer: (B)

929. Musculoskeletal abnormalities in neurofibromatosis is
(A) Hypertrophy of limb
(B) Scoliosis
(C) Pseudo arthrosis
(D) All

Answer: (D)

930. The common features of Neurofibromatosis include al, except
(A) Optic glioma
(B) Dumbellneurofibroma
(C) Scoliosis
(D) Periventricular calcifications

Answer: (D)

931. Congenital pseudo arthrosis is seen in the following
(A) Hip joint
(B) Femur
(C) Radius – ulna
(D) Tibia-fibula

Answer: (D)

932. Pseudoarthrosis can be due to all except
(A) Congenital
(B) Post inflammatory
(C) Trauma
(D) None of the above

Answer: (B)

933. Pseudoarthrosis is seen in all of the following except
(A) Idiopathic
(B) Fracture
(C) Osteomylitis
(D) Neurofibromatosis

Answer: (C)

934. Pseudoarthrosis of tibia is best treated by
(A) Internal fixation
(B) Internal fixation and bone grafting
(C) Above knee POP cast
(D) Below knee POP cast

Answer: (B)

935. Causes of a painless limp since infancy includes
(A) Congenital dislocation of hip
(B) Infantile coax vara
(C) Poliomyelitis
(D) All of the above

Answer: (D)

936. All the following are causes of a painful limb except
(A) Slipped femoral epiphysis
(B) TB of the hip
(C) Perthes disease
(D) Infantile CoxaVara

Answer: (D)

937. Antalgic hip gait is related to which of the following
(A) Waddling gait
(B) Trendelenberg gait
(C) Painful hip gait
(D) Short leg gait

Answer: (C)

938. Congenital Coxavara is treated by
(A) Fixation by SP Nail
(B) Osteotomy
(C) Bone grafting
(D) Traction
(E) Splint

Answer: (B)

939. Coxavara is found in
(A) Perthe’s disease
(B) Tuberculosis
(C) Rickets
(D) Rheumatoid arthritis

Answer: (A)

940. Transient synovitis (toxic synovitis) of the hip is characterized by all of the following, except:
(A) May follow upper respiratory infection
(B) ESR and white blood cell counts are usually normal
(C) Ultrasound of the joint reveals widening of the joint space
(D) The hip is typically held in adduction and internal rotation

Answer: (D)

941. A 7year old boy with abrupt onset of pain in hip with hip held in abduction. Hemogram is normal. ESR is raised. What is the next line of management.
(A) Hospitalize and observe
(B) Ambulatory observation
(C) Intravenous antibiotics
(D) USG guided aspiration of hip

Answer: (D)

942. True about Perthe’s disease:
(A) Congenital in nature
(B) Occur in Children b/w 4-10 yr age group
(C) Involve knee joint
(D) Involve distal femur
(E) Involve head of femur

Answer: (B, E)

943. Pethe’s disease is
(A) Fracture of femoral shaft
(B) Osteochondritis of femoral epiphysis
(C) Infarction of femoral head
(D) Fracture dislocation of femoral neck.

Answer: (B)

944. A 10 years old male with pain in left hip & limping on examination restricted abduction & internal rotation, probable diagnosis is
(A) Septic arthritis of hip
(B) Tuberculosis arthritis of hip
(C) Cong dislocation of hip
(D) Perthes disease

Answer: (D)

945. Perthe’s disease is treated by:
(A) High dose of calcium with steroids
(B) Total hip replacement
(C) Supervised containment of femora head in acetabulum
(D) Relieving weight bearing

Answer: (C)

946. Legg-Calves-Perthes’ disease is
(A) Mild pyogenic arthritis
(B) Slipped femoral epiphysis
(C) Avascular necrosis of head of femur
(D) Low grade tuberculosis of hip

Answer: (C)

947. The commonest cause of limp in a child of seven years is:
(A) T.B. hip
(B) C.D.H
(C) Perthe’s disease
(D) Slipped upper femoral epiphysis

Answer: (C)

948. Legg-Calve-Perthes’ disease is
(A) Femoral epiphyseal osteochondritis
(B) Oteochondritis of tibial tubercle
(C) Slipped upper femoral epiphysis
(D) Spinal osteochondritis

Answer: (A)

949. The late radiographic changes seen in active Perthes’ disease are all except
(A) Narrow joint space
(B) A widened femoral neck
(C) An irregular density of the epiphysis
(D) An irregular epiphyseal line

Answer: (A)

950. Legg-Calve-Perthes disease is commonly seen in the age group of
(A) 1-3 years
(B) 3-10 years
(C) 10-20 years
(D) 20 year & above

Answer: (B)

951. Which one of the following is the investigation of choice for evaluation of suspectedPerthes’ disease?
(A) Plain X-ray
(B) Ultrasonography (US).
(C) Computed Tomography (CT).
(D) Magnetic Resonance Imaging (MRI).

Answer: (D)

952. A six year old boy presents to the emergency department with a painful limp. Clinical examination reveals tenderness in the femoral triangle and some limitation of hip movements. An X-ray was done which was normal. Which of the following should be the next course of action?
(A) Wait and Watch/Observation
(B) Ultrasonography
(C) Aspiration
(D) MRI Scan

Answer: (D)

953. Regarding Perth’s disease all of the following are correct except
(A) Osteonecroticosteochondritis
(B) Decreased bone age
(C) Male predominance
(D) Restricted abduction
(E) Age of onset has no affect on course

Answer: (E)

954. True about perthe’s disease is A/E:
(A) Avascular necrosis of femoral head
(B) Onset before 10 years of age
(C) Osteotomy is used for treatment
(D) Limb shortening
(E) Joint space obliterated

Answer: (E)

955. A 12 year old with rapid increase in weight and height over the past one year presents with difficulty in sitting cross legged and squatting. The knee would go into axilla every time the flexes her hip and knee. he most likely diagnosis is:
(A) Perths disease
(B) Slipped Capital Femoral Epiphysis
(C) Congenital Coxa Vera
(D) Synovitis of knee

Answer: (B)

956. An 11 year old 70 kg. child presents with limitation of abduction and internal rotation. There is tenderness in scarpas triangle. On flexing the hip the limb is abducted. The diagnosis is:
(A) Perthes disease
(B) slipped capital femoral epiphyses
(C) Observation hip
(D) Tuberculosis hip

Answer: (B)

957. 14 year old boy with 78 kg weight & hypothyroidism developed sudden onset of severe pain & tenderness on left hip as a result of minor fall. Most likely diagnosis is
(A) Fracture neck femur
(B) SCFE
(C) Perthes
(D) Hip Dislocation

Answer: (B)

958. Trethowan’s sign is seen in
(A) Perthe’s disease
(B) CDH
(C) SCFE
(D) Fracture neck femur

Answer: (C)

959. Ortolani’s test is done for
(A) Congenital dislocation of hip
(B) Traumatic dislocation of hip
(C) Rheumatoid arthritis
(D) Tuberculous arthritis

Answer: (A)

960. Which of the following movement is restricted in congenital dislocation of hip (CDH), in infant?
(A) Flexions
(B) Extension
(C) Rotation
(D) Full abduction in 90° flexion

Answer: (D)

961. CDH is due to
(A) Large acetabulum
(B) Rotation of femur
(C) Small neck femur
(D) Small femoral head

Answer: (D)

962. Which of the following test is useful in diagnosis of congenital dislocation of hip?
(A) Barlow’s test
(B) Thomas test
(C) Hibb’s test
(D) Laguerres test

Answer: (A)

963. Barlow’s test is done for testing:
(A) CDH in child
(B) CDH in infancy
(C) Femoral neck fracture
(D) Slipped femoral epiphysis

Answer: (B)

964. In a newborn child, abduction and internal rotation produces a click sound. It is
(A) Otorolani, sign
(B) Telescoping sign
(C) Lachman’s sign
(D) Mc. Murray’s sign

Answer: (A)

965. In congenital dislocation of hip, not true is
(A) Real shortening
(B) Telescoping
(C) Trendelenburg test
(D) Head of femur downwards
(E) Asymmetrical crease

Answer: (D)

966. Congenital dislocation of hip is more commonly seen in
(A) Caucasians
(B) Negroes
(C) Japanese
(D) Eskimos

Answer: (A)

967. Commonest deformity in congenital dislocation of hip
(A) Small head of femur
(B) Angle of torsion
(C) Decreased neck shaft angle
(D) Shallow acetabulum

Answer: (A, D)

968. True about CDH:
(A) Usually b/I
(B) Genu valgum
(C) Waddling gait
(D) Shenton’s line broken
(E) Spine lordosis present

Answer: (C, D, E)

969. All of the following statements are true about development dysplasia (DDH) of the hip, except:
(A) It is more common in females
(B) Oligohydraminos is associated with a higher risk of DDH
(C) The hourglass appearance of the capsule may prevent a successful closed reduction
(D) Twin pregnancy is a known risk factor

Answer: (D)

970. The treatment of Congenital dislocation of hip after childhood is
(A) Leave alone
(B) Surgery should be done if symptomatic
(C) Conservative measures to be taken
(D) None of the above

Answer: (B)

971. Von Rosen Splint in used in
(A) CTEV
(B) CDH
(C) Fracture shaft of femur
(D) Fracture tibia

Answer: (B)

972. Absolute indication of X-ray of pelvis in case of congenital dislocation of hip are all EXCEPT
(A) Positive family history
(B) Breach presentation
(C) Unstable hip
(D) Shortening of limb

Answer: (D)

973. True about B/L CDH:
(A) Exaggerated lordosis
(B) B/L genu valgum
(C) Wadding gait
(D) Shenton’s line broken
(E) Short stature

Answer: (A, C, D)

974. Seen in B/L congenital hip dislocation is/are:
(A) Short strature
(B) Wandering acetabulum
(C) Waddling gait
(D) Lumbar Iordosis

Answer: (A, D, C)

975. Congenital bilateral dislocation of hip shows
(A) Waddling gait
(B) Lordosis
(C) +veTrendelenburg test
(D) +ve non Rosen’s sign

Answer: (B, C)

976. Which of the following is seen in bilateral congenital dislocation of hip
(A) Waddling Gait
(B) Shenton’s line is broken
(C) Trendelenderg test positive
(D) Allis test positive

Answer: (A, B, C)

977. Traumatic dislocation of epiphyseal plate of femur occurs:
(A) Medially
(B) Laterally
(C) Posteriorly
(D) Rotationally
(E) Anteriorly

Answer: (B, E)

978. Commonest presentation of congenital dislocation of knee is
(A) Varus
(B) Valgus
(C) Flexion
(D) Hyperextension

Answer: (D)

979. True about Genu Valgum:
(A) Physiological condition
(B) Pathological condition
(C) Both physiological & pathological
(D) Same as bow-legs
(E) Known as knock knee

Answer: (A, B, C, E)

980. Most common cause of genuvalgum in children is
(A) Osteoarthritis
(B) Rickets
(C) Paget’s disease
(D) Rheumatoid arthritis

Answer: (B)

981. Pappu, 7 years old young boy, had fracture of lateral condyle of femur. He developed malunion as the fracture was not reduced anatomically. Malunion will produce:
(A) Genu vulgum
(B) Genu varum
(C) Genu recurvatum
(D) Dislocation of knee

Answer: (A)

982. Which statement is true regarding gene varum (bowleg):
(A) In infants, it may be considered normal
(B) Occurs due to epiphyseal dysplasia
(C) Seldom associated with tibial angulation
(D) Affects only tibia but never femur
(E) Does not require any treatment

Answer: (A, B, C, E)

983. Blount’s disease is:
(A) Genu valgus
(B) Tibia vera
(C) Flat foot
(D) Genu recurvatum

Answer: (B)

984. All are false regarding genu varum except
(A) Orthosis is a must only during weight
(B) Orthosis is recommended during day time
(C) Orthosis is recommended during night time
(D) Orthosis is recommended full time

Answer: (D)

985. Critical age of osteotomy for genu varus is
(A) 4 years
(B) 6 years
(C) 8 years
(D) 10 years

Answer: (A)

986. Blount’s disease is characterized by:
(A) Genu valgum
(B) Genu varum
(C) Genu recurvatum
(D) Meniscal injury

Answer: (B)

987. Blount’s disease is associated with all of the following except
(A) Genu varum
(B) Genu Recurvatum
(C) Internal Tibial Torsion
(D) External Tibial Torsion

Answer: (D)

988. An athletic teenage girl complains of anterior knee pain on climbing stairs and on getting u after prolonged sitting. Which of the following is the most likely diagnosis?
(A) Chondramalacia Patellae
(B) Plica syndrome
(C) Bipartite Patella
(D) Patellofemoralosteoardin

Answer: (A)

989. Nail patella syndrome is characterized by:
(A) Iliac horn
(B) Sacral horn
(C) Absent patella
(D) Knee deformity
(E) Dislocation of patella

Answer: (A, C, D, E)

990. Rocker bottom foot is seen in:
(A) Congenital vertical talus
(B) Excessive correction of Grice procedure
(C) Arthrogryposis
(D) Holding club foot in too long corrected position
(E) Fore dorsiflexion against equinusvarus

Answer: (A)

991. CTEV is caused by all except
(A) Neurological disorder
(B) Mostly idiopathic
(C) Spinafibida
(D) Cubitusvarus
(E) Arthrogryposis multiplex

Answer: (A, B, C, E)

992. Club foot is commoner among
(A) Males
(B) Binovular twins
(C) Females
(D) Uniovular twins

Answer: (A)

993. The most common congenital anomaly among the following is encountered in our country
(A) Congenital Pseudoarthrosis of Tibia
(B) Congenital dislocation of hip
(C) Congenital talipesequinovarus
(D) Multiple congenital contractures

Answer: (C)

994. The club foot characteristically involves
(A) Foot and ankle
(B) Foot, ankle and leg
(C) Foot only
(D) Foot, ankle, leg and knee joint

Answer: (B)

995. The most common deformity seen in club foot is
(A) Talipesequinovarus
(B) Equines
(C) Equino valgus
(D) Calcaneoequinovarus

Answer: (A)

996. Various congenital deformity of the ankle joint occurs at
(A) Calcanionavicular joint
(B) Talocalcaneal joint
(C) Talonavicular joint
(D) Tarsonavicular joint

Answer: (C)

997. Most important pathology in club foot is:
(A) Congenital talonavicular dislocation
(B) Tightening of Tendoachilles
(C) Calcaneal fracture
(D) Lateral derangement

Answer: (A)

998. Talipesequinovarus is
(A) Equinus, inversion, abduction
(B) Equines, inversion, adduction
(C) Equines, eversion, abduction
(D) Equines, eversion, abduction

Answer: (B)

999. Which of the following is true of Talipescalcaneovalgus?
(A) Dorsiflexed and inverted
(B) Dorsiflexed and everted
(C) More serious than CTEV
(D) Plantar flexed

Answer: (B)

1000. The crucial component of the club foot deformity is
(A) Contracture of Tendoachilles
(B) Under development of calf and personeal muscles
(C) Subluxation of the talo-navicular joint
(D) Lateral derangement of tarsal bones

Answer: (C)

1001. The world “Talipes” refers to
(A) Long feet with spidery toes
(B) Flat feet
(C) Club feet
(D) Hammer toes

Answer: (C)

1002. All are true statement regarding talipesequinovarus except:
(A) Cubitus valgus
(B) Inversion of the foot
(C) Abduction of the forefoot
(D) Arthrogryposis multiplex congenital causes it
(E) Treatment should start after 3 month

Answer: (A, C, E)

1003. A newborn child presents with inverted foot & dorsum of foot can not touch anterior tibia. The most probable diagnosis is:
(A) CTEV
(B) Congenital vertical talus
(C) Arthrogryposis Multiplex
(D) Flat foot

Answer: (A)

1004. The ideal treatment of bilateral idiopathic clubfoot in a newborn is:
(A) Manipulation by mother
(B) Manipulation and Dennis Brown splint
(C) Manipulation and casts
(D) Surgical release

Answer: (A)

1005. Club foot in a new born in is treated by
(A) Surgery
(B) Manipulation by the mother
(C) Dennis Brown splint
(D) Strapping

Answer: (B)

1006. Treatment of clubfoot in new born is
(A) Manipulation and corrective splint
(B) Corrective splint
(C) Nothing to be done for 6 months
(D) Surgical

Answer: (A)

1007. Early CTEV is treated by
(A) CTEV cast from 1st postnatal day
(B) Manipulation
(C) Manipulation & Strapping
(D) Operative repair as early as possible

Answer: (C)

1008. Treatment of club foot should begin:
(A) As soon as possible after birth
(B) 1 month after birth
(C) 1 year after birth
(D) None of the above

Answer: (A)

1009. Treatment of club foot in new born is:
(A) Observation till 6 months of age
(B) Corrective splint application
(C) Manipulation alone
(D) Manipulation & corrective splint

Answer: (D)

1010. Treatment of club foot in new born is
(A) Observation till 6 months of age
(B) Corrective splint application
(C) Manipulation and corrective splint
(D) Tendon releases operation

Answer: (C)

1011. In correction of clubfoot by manipulation, which deformity should be corrected first:
(A) Forefoot adduction
(B) Varus
(C) Upper end tibia
(D) Calcaneum

Answer: (A)

1012. Child 3¼ years is treated for CTEV by
(A) Triple arthrodesis
(B) Postero medial soft tissue release
(C) Lateral wedge resection
(D) Tendo Achilles lengthening and posterior capsulatomy

Answer: (B)

1013. Treatment for chronic cases of club foot is:
(A) Triple arthrodesis
(B) Dorso medial release
(C) Amputation
(D) None

Answer: (A)

1014. Club foot seen in a 15 year old could be treated successfully by a:
(A) Appropriate footwear
(B) Soft tissue operation
(C) Triple arthrodesis
(D) Quadrple fusion

Answer: (C)

1015. ‘Pseudoarthrosis’ in Triple fusion is seen at the joint of
(A) Calcaneocuboid
(B) Calcaneonavicular
(C) Naviculocuboid
(D) Talonavicular

Answer: (D)

1016. Triple fusion involves the following
(A) Subtalar, calcaneocuboid, talonavicular
(B) Subtalar, calceneum, tibiofibular
(C) Subtalar, tibiofibular, calcaneocuboid
(D) Tibiofibular, cuboid, calcaneum

Answer: (A)

1017. Triple arthrodesis involves:
(A) Calcaneocuboid, talonavicular and talocalcaneal
(B) Tibiotalar, calcaneocuboid and talonavicular
(C) Ankle joint, calcaneocuboid and talonavicular
(D) None of the above

Answer: (A)

1018. Triple arthrodesis is NOT done before-skeletal maturation because of:
(A) Shortening of foot
(B) Recurrence of deformity
(C) Inadequate fusion
(D) Complete correction not possible

Answer: (C)

1019. Rocker bottom foot is due to
(A) Over corrected club foot
(B) Poliomyelitis
(C) Vertical talus
(D) Deformities of spine

Answer: (A, C)

1020. In Klippel-Feil syndrome, the patient has all of the following clinical features except:
(A) Low hair line
(B) Bilateral Neck webbing.
(C) Bilateral shortness of sterno mastoid muscles
(D) Gross limitations of neck movements.

Answer: (C)

1021. The characteristic triad of Klippel-Feil syndrome includes all of the following, Except
(A) Short neck
(B) Low hair line
(C) Limited neck movements
(D) Elevated scapula

Answer: (D)

1022. Sprengel’s shoulder is due to deformity
(A) Scrapula
(B) Humerus
(C) Clavicle
(D) Vertebra

Answer: (C)

1023. Sprengel’s deformity of scapula
(A) Undescended/Elevated scapula
(B) Undescended neck of scapula
(C) Exostosis scapula
(D) None of the above

Answer: (A)

1024. Sprengel’s deformity is
(A) Absence of clavicle
(B) Acomioclavicular dislocation
(C) Congenital elevation of scapula
(D) Recurrent dislocation of shoulder

Answer: (C)

1025. Spina bifida occulta is
(A) Infection of the spine
(B) Traumatic fracture of the spine
(C) Congenital fusion of the body of the Vertebra
(D) Congenital on fusion of the spinal lamina

Answer: (D)

1026. All the following are true in infantile torticollis, EXCEPT:
(A) It arises before birth
(B) There is facial asymmetry
(C) Commonest form of wryneck
(D) Infarction of sterno-cleidomastoid muscle

Answer: (A)

1027. Which of the following is not true is case of congenital tortiocollis
(A) Seen only in cases of breech vaginal delivery
(B) It can disappear spontaneously
(C) It is also known as sternomastoidtumour
(D) Untreated, neglected cases can result in plagiocephaly.

Answer: (A)

1028. About congenital torticollis all are true except:
(A) Always associated with breech extraction
(B) Spontaneous resolution in most cases
(C) 2/3rd cases have palpable neck mass at birth
(D) Uncorrected cases develop plagiocephaly

Answer: (A)

1029. Progression of congenital scoliosis is least likely in which of the following vertebra anomalies
(A) Fully segmented Hemivertebra
(B) Wedge vertebra
(C) Block vertebra
(D) Unilateral unsegmented bar with Hemivertebra

Answer: (C)

1030. HariVerdhman, 9 yrs old child, presents with scoliosis, hairy tuft in the skin of back and neurological deficit. Plain X-rays reveal multiple vertebral anomalies & a vertical bony spur overlying lumbar spine on AP view. The most probable diagnosis is:
(A) Dorsal dermal sinus
(B) Diastometamyelia
(C) Tight filumterminale
(D) Caudal regression syndrome

Answer: (B)

1031. Vertebra plana seen in:
(A) Eosinophilia granuloma
(B) Trauma
(C) Paget’s disease
(D) Malignancy
(E) Ewing’s sarcoma

Answer: (A)

1032. RisserLocaliser cast is used in the management of:
(A) Kyphosis
(B) Spondylolysthesis
(C) Idiopathic scoliosis
(D) Lordosis

Answer: (C)

1033. Bone dysplasia is due to
(A) Faulty nutrition
(B) Faulty development
(C) Trauma
(D) Parathyroid tumour

Answer: (B)

1034. The following is false of Achondroplasia
(A) Autosomal dominant
(B) Mental retardation
(C) Due to gene mutation
(D) Shortening of limbs present

Answer: (B)

1035. Mode of inheritance of Achondroplasia is
(A) Autosomal dominant
(B) Autosomal recessive
(C) X-liked dominant
(D) X-linked recessive

Answer: (A)

1036. All are true of Achondroplasia except
(A) Autosomal dominant
(B) Shortening of limbs is present
(C) Usually due to gene mutation
(D) Mental retardation

Answer: (D)

1037. “Trident hand” seen in
(A) Achondroplasia
(B) Mucopolysaccharoidosis
(C) Diphysealachlasia
(D) Cleido-cranial dystosis

Answer: (A)

1038. A short statured patient brought to Orthopaedics OPD with a X ray showing flattened vertebral with beak. The probable diagnosis is
(A) Achondroplasia
(B) Ochronosis
(C) Eosinophilic granuloma
(D) Calve’s disease

Answer: (A)

1039. The features of Achondroplasia include all, except
(A) Defective head
(B) Mental retardation
(C) Autosomal recessive
(D) Familial

Answer: (C)

1040. Absent lateral 1/3rd of clavicle is seen in:
(A) Hyperparathyroidism
(B) Turner’s syndrome
(C) Fibrous dysplasia
(D) Cleidocranialdysostosis

Answer: (D)

1041. Cleidocranialdysostosis may show:
(A) Wide foramen magnum
(B) Absence of clavicles
(C) Coax vara
(D) All of the above

Answer: (D)

1042. A 9-year old child with high arched palate has shoulders meeting in front of his chest. he has
(A) Erb’s palsy
(B) Cleidocranialdysostosis
(C) Chondro-Osteodystrophy
(D) Cortical hyperostosis

Answer: (B)

1043. Absent clavicles are seen in
(A) Cleidocranialdysostosis
(B) Achondroplasia
(C) Moriquo’s disease
(D) Oliver’s disease

Answer: (A)

1044. The characteristics of Morquio’s disease include
(A) Spinal kyphosis
(B) Subnormal/normal intelligence
(C) Excessive excretion of keratosulphate in urine
(D) Dwarfism
(E) All of the above

Answer: (E)

1045. Osteogenesis imperfect is due to
(A) Defective mineralization of bone
(B) Fracture mobilization of bone
(C) Abnormal collagen defect
(D) Excessive osteoid formation

Answer: (C)

1046. Osteogenesis imperfect is due to the following
(A) Excessive Osteoblastic activity
(B) Defective Osteoid formation
(C) Defective osteoclast function
(D) Defective mineralisation of bone

Answer: (B)

1047. Osteogenesis imperfect is defect in
(A) Bone
(B) Calcification
(C) Cartilage
(D) Collagen

Answer: (D)

1048. Osteogenesisimperfecta
(A) Autosomal dominant (AD)
(B) Autosomal Recessive (AR)
(C) Both AD and AR
(D) Sex linked dominant
(E) None of the above

Answer: (C)

1049. Multiple bone fractures are seen in
(A) Rickets
(B) Osteomalacia
(C) Scurvy
(D) Osteogenesisimperfecta

Answer: (D)

1050. In which of the following condition bilateral symmetrical fractures occur?
(A) Rickets
(B) Osteopetrosis
(C) Osteogenesisimperfecta
(D) Fluorosis

Answer: (C)

1051. Multiple bone fracture in a new born is seen in
(A) Scurvy
(B) Syphilis
(C) Osteogenesisimperfecta
(D) Morquio’s syndrome

Answer: (C)

1052. Wormian bones are seen in
(A) Osteogenesisimperfecta
(B) Scheurmanns disease
(C) Paget’s disease
(D) Osteoclastoma

Answer: (A)

1053. Wormian bones are not noted in
(A) Fibrous dysplasia
(B) Osteogenesisimperfecta
(C) Cretinism
(D) Rickets

Answer: (B)

1054. Not true about Osteogenesisimperfecta
(A) Impaired healing of fracture
(B) Deafness
(C) Laxity of joints
(D) Fragile fracture

Answer: (A)

1055. Following are true about Osteogenesisimperfeta except
(A) Usually green stick fracture
(B) Generally do not unite without T/t
(C) Fracture frequency decreases during puberty
(D) Fracture seen intrauterine life and during parturition

Answer: (B)

1056. All are seen in osteogenesisimperfecta except
(A) Blue sclera
(B) Bilateral Hip dislocation
(C) Lax ligament
(D) Osteoporosis

Answer: (B)

1057. Not true about Osteogenesisimperfecta is
(A) Deafness
(B) Fractures
(C) Cataract
(D) Blue Sclera

Answer: (C)

1058. All are features of Osteogenesisimperfecta except:
(A) Blue sclera
(B) Multiple # s
(C) Cataract
(D) Hearing loss

Answer: (C)

1059. Osteogenesisimperfecta is characterized by:
(A) Blue sclera
(B) Also known as brittle bone disease
(C) Collagen defect
(D) A/W otosclerosis
(E) Autosomal dominant

Answer: (A, B, E)

1060. True about osteogenesisimperfecta:
(A) Autosomal dominant
(B) Known as marble bone disease
(C) Blue sclera
(D) Associated with otosclerosis
(E) Defect in collagen-I

Answer: (A, C, D, E)

1061. Parental determination of osteogenesisimperfecta is done by:
(A) Acid phosphatase
(B) Alkaline phosphatase
(C) Abnormal Pro-α chain
(D) None of these

Answer: (C)

1062. Brittle bones disease is
(A) Osteoporosis
(B) Osteopetrosis
(C) Osteogenesisimperfecta
(D) Osteomalacia

Answer: (C)

1063. Marble bone disease is
(A) Osteitisfragils
(B) Osteopetrosis
(C) Osteosarcoma
(D) Paget’s disease

Answer: (B)

1064. Albers schonberg disease is:
(A) Osteopetrosis
(B) Osteoporosis
(C) Osteochondritis
(D) Osteomalacia

Answer: (A)

1065. A 3 year male presented with progressive anemia, hepatosplenomegaly and osteomyelitis of jaw with pathological fracture, x-ray shows chalky white deposits on bone, probable diagnosis is
(A) Osteopetrosis
(B) Osteopoikilocytosis
(C) Alkaptonuria
(D) Myositis-ossificansprogressiva

Answer: (A)

1066. Raju, a 10 yrs old child, presents with predisposition to fractures, anemia, hepatosplenomegaly and a diffusely increased radiographic density of bones. The most likely diagnosis is
(A) Osteogenesisimperfecta
(B) Pyenodysotosis
(C) Myelofibrosis
(D) Osteopetrosis

Answer: (D)

1067. All of the following features are characteristic of Osteopetrosis except:
(A) Pancytopenia
(B) Abnormally slow rates of fracture healing
(C) Compression of cranial nerves
(D) Mandibular osteomyelitis

Answer: (B)

1068. Muscle most commonly affected by congenital absence is
(A) Pectoralis major
(B) Semi membranosus
(C) Teres minor
(D) Gluteus maximus

Answer: (A)

1069. Bone density on X-ray is/are seen in:
(A) Uric acid crystal deposition
(B) # and collapse of cancellous bone
(C) Increased thickening of trabeculae
(D) Periosteal reaction
(E) AVN

Answer: (B, C, D, E)

1070. Increased Bone density in X-ray seen in:
(A) Increased thickening of trabeculae
(B) Fracture & Collapse of cancellous bone
(C) Defective mineralization
(D) Myossitisossificans
(E) Relative disuse atrophy & surrounding bone response

Answer: (A, B)

1071. Increased bone density in X-ray seen in:
(A) Collapse cancellous bone
(B) Periosteal reaction
(C) Paget’s disease
(D) AVN
(E) Osteomyelitis

Answer: (A, B, C, D)

1072. There is considerable morbidity and mortality due to bone disease, more frequently amongst women. The most common cause of bone disease in India is:
(A) Nutritional Deficiency
(B) Steroid Induced osteoporosis
(C) Paget’s Disease
(D) Sarcoidosis

Answer: (A)

1073. Increased bone density occurs in:
(A) Cushing syndrome
(B) Hypoparathyroidism
(C) Fluorosis
(D) Hyperthyroidism

Answer: (C)

1074. Increased density in skull vault is seen in:
(A) Hyperparathyroidism
(B) Multiple myeloma
(C) Fluorosis
(D) Renal osteodystrophy

Answer: (C)

1075. What is the diagnostic radiological finding skeletal fluorosis
(A) Sclerosis of sacroiliac joint
(B) Interroseous membrane ossification
(C) Osteosclerosis of vertebral body
(D) Ossification of ligaments of knee joint

Answer: (B)

1076. The following is a radiological feature of Skeletal Fluorosis
(A) Intervertebral ligament calcification
(B) Sacral sclerosis
(C) Osteolytic lesion in pelvis
(D) None

Answer: (A)

1077. Manifestations of Fluorosis includes
(A) Stiffness of back ligaments
(B) Caries teeth (due to deficiency of fluorine)
(C) Genu valgum
(D) Dental changes
(E) Stiffnesss of bones and tendons (a, d)

Answer: (A, D, E)

1078. Jaw tumour is seen in
(A) Osteoporosis
(B) Osteomalacia
(C) Osteopetrosis
(D) Caffey’s disease

Answer: (D)

1079. Caffey’s disease is
(A) Renal osteodystrophy
(B) Infantile cortical hyperostosis
(C) Osteomyelitis of jaw in children
(D) Chronic osteomyelitis in children

Answer: (B)

1080. Caffey’s disease occurs in
(A) Infants below 6 months
(B) Above years
(C) Above 10-20 years
(D) 20-40 years

Answer: (A)

1081. Osteomyelitis of Jaw is seen in
(A) Osteomalacia
(B) Osteopoikilosis
(C) Osteoporosis
(D) Caffey’s disease

Answer: (D)

1082. “Rugger Jersey Spine” is seen in:
(A) Fluorosis
(B) Achondroplasia
(C) Renal osteodystrophy
(D) Marfan’s syndrome

Answer: (C)

1083. Rugger jersey spine in CRF is due to:
(A) Osteomalacia
(B) Trauma
(C) Hyperparathyroidism
(D) Aluminiumosteodystrophy
(E) Osteopetrosis

Answer: (C)

1084. Metabolic bone disease is caused by excess intake of which vitamin:
(A) A
(B) B
(C) C
(D) D
(E) E

Answer: (A)

1085. Hypervitaminosis of which of the following will cause bony abnormalities:
(A) Vit. A
(B) Vit. D
(C) Vit. C
(D) Vit. E
(E) Vit. K

Answer: (A, B, D)

1086. A Bald child with swollen abdomen, hyperosteous bones with mental retardation has
(A) Hypervitaminosis A
(B) Hypervitaminosis D
(C) Down’s syndrome
(D) Tuberous sclerosis

Answer: (B)

1087. Action of Vitamin D is that it
(A) Stimulates bone marrow
(B) Increases calcium loss
(C) Stimulates absorption of calcium
(D) Stimulates osteoclasts.

Answer: (C)

1088. Osteomalacia is associated with:
(A) Decreases in osteoid volume
(B) Decrease in osteoid surface
(C) Increase in osteoid maturation time
(D) Increase in mineral apposition rate

Answer: (C)

1089. Drug induced Osteomalacia is known to be associated with the use of:
(A) Steroids
(B) Heparin
(C) Phenytoin
(D) Gentamycin

Answer: (C)

1090. The cause of osteomalacia is
(A) Phenytoin
(B) Malabsorption
(C) Vincristine
(D) Cushing’s syndrome

Answer: (A, B)

1091. The commonest cause of osteomalacia in our country is:
(A) Repeated systemic infections
(B) Idiopathic steatorrhoea
(C) Dieteic deficiency of vitamin D & calcium
(D) Poor protein intake
(E) Chronic renal disease

Answer: (C)

1092. Osteomalacia causes include
(A) Phenytoin
(B) Malabsorption
(C) Indoor stay
(D) Vincristine
(E) Cushing’s syndrome

Answer: (A, B, C)

1093. Osteomalacia/Rickerts may be seen in A/E
(A) Neurofibroma
(B) Osteoblastoma
(C) Hemangiopericytoma
(D) Ewings sarcoma

Answer: (D)

1094. Rickets is seen in A/E
(A) Dietary deficiency
(B) Hypocalcemia
(C) Malabsorption
(D) Fanconi’s syndrome

Answer: (B)

1095. Basic pathological defect in rickets is
(A) Decreased osteoblastic activity
(B) Non functional osteoclast
(C) Defective osteoclasticresoprtion of uncalcified osteoid and cartilage
(D) Defective proliferation of physis.

Answer: (C)

1096. Bony lesion in Rickets is due to
(A) Increased parathoromone leading to osteoclastic activity
(B) Reduced dietary calcium absorption
(C) Increased calcium excretion
(D) None

Answer: (B)

1097. Decreased mineralization of Epiphyseal plate in a growing child is seen in:
(A) Rickets
(B) Osteomalacia
(C) Scurvy
(D) Osteoporosis

Answer: (A)

1098. Defective mineralization of proximal zone of cartilage and epiphysis is a feature of:
(A) Rickets
(B) Scurvy
(C) Osteomalacia
(D) Syphilis

Answer: (A)

1099. Rickets of vitamin D deficiency is associated with
(A) Subperiostealhaemorrhage
(B) Ricketrirosary at costochondral junction produced by subluxation of sternal plate
(C) Apt to develop during growth in low birth weight infants
(D) Inorganic serum phosphate concentration 4.5-6.5 mg/dl

Answer: (B)

1100. In nutrional rickets changes occurs at the:
(A) Metaphysis
(B) Epiphysis
(C) Diaphysis
(D) All of these

Answer: (A)

1101. Rickets in infancy is characterized by the following except:
(A) Craniotabes
(B) Rachitic rosary
(C) Wide open fontanelles
(D) Bow legs

Answer: (D)

1102. Biochemical changes seen in osteomalacia:
(A) Serum Ca2+↑
(B) Serum Ca2+↓
(C) Serum Normal ca2+
(D) ↑ ALP
(E) ↓ 1, 25 (OH) 2D

Answer: (B, C, E)

1103. Biochemical abnormality seen in nutritional rickets:
(A) Hyperphosphaturia
(B) Hyperphosphatemia
(C) ALP
(D) PTH
(E) Hypocalcemia

Answer: (E)

1104. Note seen in Rickets
(A) Cupping of ends of bones
(B) Irritability
(C) Craniotabes
(D) Decreased alkaline phosphatase

Answer: (D)

1105. The metabolic indicator of Rickets is
(A) Low serum Ca
(B) Low PO4
(C) ↑Alkaline phosphatase
(D) ↑ Urinary hydroxyl proline

Answer: (C)

1106. Which of the following is a persistant biochemical marker of rickets:
(A) S. Ca++
(B) S. Alkaline phosphate
(C) S. Acid phosphate
(D) S. phosphate

Answer: (B)

1107. A diet deficient in calcium will most commonly result in
(A) Osteomalacia
(B) Rickets
(C) Osteoporosis
(D) Osteitisfibrosa

Answer: (A)

1108. Pseudo fracture is seen in:
(A) Pseudo parathyoridism
(B) Hypoparathyroidism
(C) Osteomalacia
(D) Osteoporosis

Answer: (C)

1109. Which is true about Osteomalacia?
(A) Alkaline phosphatase is raised
(B) Serum phosphate is low
(C) Serum calcium is low or normal
(D) All of the above

Answer: (D)

1110. Alkaline phosphatase is elevated in all, EXCEPT:
(A) Rickets
(B) Osteomalacia
(C) Hypoparathyroidism
(D) Hypophosphatemia

Answer: (C)

1111. Increased alkaline phosphate is seen in:
(A) Multiple myeloma
(B) Primary hyperparathyroidism
(C) Chronic renal failure
(D) Osteoporosis

Answer: (B, C)

1112. During a routine check up, a 67-year-old man is found to have a level of serum alkaline phosphatase three times the upper limit of normal. Serum calcium and phosphorus concentrations and liver function test results are normal. He is asymptomatic. The most likely diagnosis is:
(A) Metastatic bone disease
(B) Primary hyperparathyroidism
(C) Paget’s disease of bone
(D) Osteomalacia

Answer: (C)

1113. A patient Shweta with raised serum alkaline phosphatase and raised parathormone level along with low calcium and low phosphate level is likely to have:
(A) Primary hyperparathyroidism
(B) Paget’s disease
(C) Osteoporosis
(D) Vitamin D deficiency

Answer: (D)

1114. A 65 year old female presented with # of T12 vertebra with increased alkaline phosphatase and paratharmone level along with low calcium and low phosphate level, diagnosis is
(A) Osteoporosis
(B) Paget’s disease
(C) Primary hyperparathyroidism
(D) Vitamin D deficiency

Answer: (D)

1115. The characteristic finding in osteomalacia is
(A) ↓ P
(B) ↓ Ca
(C) ↓ Ca& ↑ P
(D) ↓ Ca& ↓ P

Answer: (D)

1116. All are true regarding Osteomalacia
(A) ↑ Serum Ca++
(B) ↑ Serum Phosphate
(C) Normal Serum calcium
(D) Osteosclerotic lesions
(E) Pseudofracture seen

Answer: (C, E)

1117. Milkman’s fracture is?
(A) Pseudofracture in adults
(B) Fracture of Clavicle in children
(C) Fracture humerus
(D) Fracture first Metacarpal

Answer: (A)

1118. A young patient presents with enlargement of costochondral junction and with the white line of Fraenkel at the metaphysis. The diagnosis is
(A) Scurvy
(B) Rickets
(C) Hyperparathyroidism
(D) Osteomalacia

Answer: (A)

1119. In scurvy there is deficit in
(A) Collagen
(B) Fibroblasts
(C) Elastic fibres
(D) All of the above

Answer: (A)

1120. Vitamin required for collagen is
(A) Vitamin A
(B) Vitamin C
(C) Vitamin D
(D) Vitamin E

Answer: (B)

1121. Vitamin C deficiency leads to
(A) Defective mineralization
(B) Defective Osteoid formation
(C) Normal collagen and Bone matrix
(D) X-ray shows normal evidence

Answer: (B)

1122. Scurvy affects bone by
(A) Decreasing mineralization of bone
(B) Less formation of osteoid matrix
(C) Loss of calcium from mineralized matrix
(D) Affection of zone of provisional calcification only

Answer: (B)

1123. Ring sign is seen in:
(A) Rickets
(B) Barlow’s disease
(C) Scurvy
(D) Fibrous dysplasia

Answer: (C)

1124. Which is/are X-ray finding of infantile scurvy?
(A) A dense line between metaphysis and epiphyseal cartilage
(B) A clear band of rarefaction on the diaphysial side
(C) Both of these
(D) None of these

Answer: (C)

1125. Metaphyseal fracture is commonly seen in
(A) Osteogenesisimperfecta
(B) Scurvy
(C) Rickets
(D) None

Answer: (B)

1126. Soft tissue calcification occurs in all except:
(A) Hyper parathyroidism
(B) Scleroderma
(C) Hyperthyroidism
(D) Hyper vitaminosis D

Answer: (C)

1127. Osteitisfibrosacystica is due to
(A) Paget’s disease of bone
(B) Polyostotic fibrous dysplasia
(C) Parathyroid adenoma
(D) All of the above

Answer: (C)

1128. Most common cause of primary hyperparathyroidism is:
(A) Solitary adenoma
(B) Chief cell hyperplasia
(C) Multiple adenoma
(D) Werner’s syndrome

Answer: (A)

1129. Which is not a feature of Osteitisfibrosacystica?
(A) Fracture
(B) Tetany
(C) Increased serum calcium
(D) Increased alkaline phosphatase

Answer: (B)

1130. Sub-Periosteal erosions of middle phalanges at the radial aspect in characteristic of
(A) Hypothyroidism
(B) Hyperthyroidism
(C) Hypoparathyroidism
(D) Hyperparathyroidism

Answer: (D)

1131. Subperiosteal erosion is seen in
(A) Scurvy
(B) Hyperparathyroidism
(C) Hypoparathyroidism
(D) Rickets

Answer: (B)

1132. In hyperparathyroidism bone resorption is seen in all except
(A) Jaws
(B) Metacarpals
(C) Ribs
(D) End of long bones

Answer: (A)

1133. Soft tissue calcification with hypercalcaemia is abserved in:
(A) Hyperparathyroidism
(B) Alkaptonuria
(C) Gout
(D) Cushing’s disease

Answer: (A)

1134. Tufting of the terminal phalanges is seen in:
(A) Hypoparathyroidism
(B) Hyperparathyroidism
(C) Hyperthyroidism
(D) Hypothyroidism

Answer: (B)

1135. Resorption of the terminal phalanx is not seen in:
(A) Hyperparathyroidism
(B) Reiter’s syndrome
(C) Scleroderma
(D) Psoriasis

Answer: (A)

1136. Erosion of ungula tufts in terminal phalanges of hand may be due to:
(A) Rheumatoid arthritis
(B) Gout
(C) Hyperparathyroidism
(D) Psoriasis

Answer: (D)

1137. Brown Tumor is seen in
(A) Hypothyroidism
(B) Hyperthyroidism
(C) Hypoparathyroidism
(D) Hyperparathyroidism

Answer: (D)

1138. Sclerotic lesion on the bone is seen in all except
(A) Osteitisfibrosa
(B) Osteopetrosis
(C) Melorheostosis
(D) Caffey’s disease

Answer: (A)

1139. Osteitisfibrosacystica is seen in:
(A) Hyperparathyroidism
(B) Hypoparathyroidism
(C) Hypothyroidism
(D) Hyperthyroidism

Answer: (A)

1140. Absence of lamina dura in the alveolus occurs in
(A) Rickets
(B) Osteomalacia
(C) Deficiency of vitamin C
(D) Hyperparathyroidism

Answer: (D)

1141. Sclerosis of bone is seen in all except:
(A) Secondaries from prostate
(B) Fluorosis
(C) Hyperparathyroidism
(D) Osteopetrosis

Answer: (C)

1142. Brown tumors are seen in:
(A) Hyperparathyroidism
(B) Pigmented villonodularsynovitis
(C) Osteomalacia
(D) Neurofibromatosis

Answer: (A)

1143. A 50 year old man presented with multiple pathological fractures. His serum calcium was 11.5 mg/dl and phosphate was 2.5 mg/dl. Alkaline phosphatase was 940 I.U/dl. The most probable diagnosis is
(A) Osteoporosis
(B) Osteomalacia
(C) Multiple Myeloma
(D) Hyperparathyroidism

Answer: (D)

1144. Treatment of choice for Paget’s disease of the bone is
(A) Vitamin D
(B) Immobilization of the limb
(C) Surgical treatment
(D) Calcitonin

Answer: (D)

1145. Paget’s disease also known as:
(A) Osteitisdeformans
(B) Osteitisfibrosacystica
(C) Osteochonoritis
(D) Osteomalacia
(E) Marble bone disease

Answer: (A)

1146. Paget’s disease of bone commonly affects:
(A) Skull
(B) Vertebra
(C) Pelvis
(D) Femur
(E) Humerus

Answer: (A, B, C, D)

1147. Paget’s disease of one commonly affects:
(A) Skull
(B) Vertebra
(C) Pelvis
(D) Phalanges
(E) Humerus

Answer: (A, B, C)

1148. Pain in pagets disease is relieved best by
(A) Simple analgesics
(B) Narcotic analgesics
(C) Radiation
(D) Calcitonin

Answer: (D)

1149. The complications of Paget’s disease is:
(A) Osteogenic sarcoma
(B) Deafness
(C) Heart failure
(D) All of the above

Answer: (D)

1150. Deafness in case of Paget’s disease is due to
(A) Thickened cranium
(B) Narrowing of foramina of skull
(C) Brain compression
(D) Otosclerosis

Answer: (B)

1151. Drug therapy of Paget’s disease (OsteitisDeformans) include all except:
(A) Alendronate
(B) Etidronate
(C) Calcitonin
(D) Plicamycin

Answer: (D)

1152. Following are features of Paget’s disease except
(A) Deformity of bones
(B) Secondary osteosarcoma
(C) Lowered serum alkaline phosphatase
(D) Surgery

Answer: (C)

1153. The histopathologic feature of Paget’s disease includes
(A) Simultaneous osteoblastic activity at places
(B) Osteoclasticresorption
(C) Replacement of bone marrow by fibro vascular tissue
(D) All of the above

Answer: (D)

1154. Which of the following is a primary defect in Paget’s disease?
(A) Osteoblast
(B) Osteoclast
(C) Osteocyte
(D) Fibroblast

Answer: (B)

1155. All of the following statements regarding Paget’s disease are correct except
(A) Females are affected more than males
(B) If frequently leads to Osteogenic sarcoma
(C) Serum alkaline phosphates level is ↑ed
(D) All called as Osteitisdeformans

Answer: (A)

1156. Calcitonin is used in the treatment of
(A) Post menopausal osteoporosis
(B) Malignant hyper calcemia
(C) Paget’s disease
(D) All of the above

Answer: (C)

1157. A 67 year old man on biochemical analysis found to have three fold rise of level of serum alkaline phosphatase that upper limit of norm value during a routine checkup but serum calcium and phosphorous concentration and liver function test results and normal. He is asymptomatic. The probable cause is
(A) Multiple myeloma
(B) Paget’s disease of bone
(C) Primary hyperparathyroidism
(D) Osteomalacia

Answer: (B)

1158. Calcitonin is used in
(A) Paget’s disease
(B) Hyperparathyroidism
(C) Osteosclerosis
(D) Vitamin D intoxication

Answer: (A)

1159. A patient presents with bone pains and on investigating calcium and phosphorus levels in serum were found to be normal except with elevation of serum alkaline phosphatase. The probable diagnosis is
(A) Osteomalacia
(B) Paget’s disease
(C) Osteoporosis
(D) Hyperparathyroidism

Answer: (B)

1160. Which is not true about Paget’s disease of bone?
(A) Often there is no symptom
(B) Alkaline phosphates is low
(C) Thickening of bone due to widening of cortex
(D) All of these

Answer: (B)

1161. Which of the following is not true regarding Paget’s disease of bone?
(A) Serum alkaline phosphatase is low
(B) In frequently leads to Osteogenic sarcoma
(C) Males are affected more than females
(D) Involved area shows rapid formation and resorption of bone

Answer: (A)

1162. Paget’s disease
(A) Is painless
(B) Begins as an osteoblastic
(C) Is frequently complicated by osteogenic sarcoma
(D) Does not affect bones singly (e.g. Only the clavicle)

Answer: (C)

1163. 60 years old male with bony abnormally at upper tibia associated with sensori neural hearing loss. On laboratory examination serum alkaline phosphatase levels are (440 mU/1) elevated and serum Ca++ and PO4− cotton wool spots in x-ray skull. Diagnosis is:
(A) Fibrous dysplesia
(B) Paget disease
(C) Osteosclerotic metastasis
(D) Osteoporosis

Answer: (B)

1164. True about paget’s disease:
(A) Common in young girl 10-16yr of age
(B) ↑ALP
(C) Associated with hypercalcemia& hypophosphatemia
(D) More common in female
(E) Associated with chondrosarcoma

Answer: (B, E)

1165. Which of the following is seen in osteoporosis:
(A) Low Ca, high PO4, high alkaline phosphatase
(B) Low Ca, low PO4, low alkaline phosphatase
(C) Normal Ca, normal PO4, normal alkaline phosphatase
(D) Low Ca, low PO4, normal alkaline phosphatase

Answer: (C)

1166. True about osteoporosis:
(A) ↓with age
(B) Due to diet deficiency in Ca
(C) Commonly presented with #
(D) Cod-fish vertebra seen

Answer: (C, D)

1167. Osteoporosis is seen in all the following except
(A) Thyrotoxicosis
(B) Rheumatoid arthritis
(C) Hypoparathyroidism
(D) Steroid therapy

Answer: (C)

1168. All are causes of Osteoporosis, except:
(A) Thyrotoxicosis
(B) Hypothyroidism
(C) Chronic heparin therapy
(D) Old age

Answer: (B)

1169. Osteoporosis may be seen in all except:
(A) Hyperparathyroidism
(B) Hypoparathyroidism
(C) Thyrotoxicosis
(D) Heparin administration

Answer: (B)

1170. Osteoporosis is caused by all except:
(A) Sarcoidosis
(B) Old age
(C) Hypoparathyroidism
(D) Steroid therapy

Answer: (C)

1171. Osteoporosis is seen in:
(A) Thyrotoxicosis
(B) Cushing’s disease
(C) Menopause
(D) All of the above

Answer: (D)

1172. All of the following conditions may be responsible for osteoporosis except
(A) Steroid therapy
(B) Prolonged weightlessness in spaceship
(C) Hyperparathyroidism
(D) Hypoparathyroidism

Answer: (D)

1173. Osteoporosis is seen in following except
(A) Old age
(B) Prolonged Heparin administration
(C) Hyperthyroidism
(D) Steroid intake

Answer: (C)

1174. The most common manifestation of osteoporosis is:
(A) Compression fracture of the spine
(B) Asymptomatic, detected incidentally by low serum Calcium
(C) Bowing of legs
(D) Loss of weight

Answer: (A)

1175. The most common site of fracture of the following bone in senile osteoporosis is
(A) Vertebra
(B) Neck of femur
(C) Radius
(D) Shaft of femur

Answer: (A)

1176. Osteoporosis is characterized most commonly by
(A) Fracture vertebra
(B) Backache
(C) Bowing of legs
(D) Abdominal pain

Answer: (A)

1177. All of the following fractures are associated with osteoporosis except:
(A) Colles’ fracture
(B) Fracture of clavicle
(C) Vertebral fracture
(D) Fracture neck of femur

Answer: (B)

1178. Not a complication of menopause:
(A) Fracture spine
(B) Colles fracture
(C) Fracture neck of femur
(D) Supra condylar fracture humerus

Answer: (D)

1179. Risk factors for osteoporosis
(A) Late menopause
(B) COPD
(C) Obesity
(D) Smoking
(E) OCP

Answer: (D, B)

1180. Bone Density is best studied by:
(A) CT scan
(B) DEXA scan
(C) MRI scan
(D) Bone scan

Answer: (B)

1181. Drug of choice for senile osteoporosis is
(A) Estrogens
(B) Androgens
(C) Calcitonin
(D) Ethidronate

Answer: (A)

1182. Which of the following is useful in the treatment of Osteoporosis?
(A) Potassium
(B) Sodium chloride
(C) Calcium
(D) Phosphate

Answer: (C)

1183. A 65-year-old female patient presents with
(A) Calcium + Oestrogen + Progesterone
(B) Oestrogen + Progesterone
(C) Calcium + Ostrogen
(D) Calcium

Answer: (A)

1184. Calcium content of bone is increased in
(A) Prolonged immobilization
(B) Glucocorticoid’s administration
(C) Hyperparathyroidism
(D) Estrogen supplementation in post menopausal women

Answer: (D)

1185. Treatment of postmenopausal osteoporosis:
(A) Tamoxifen
(B) Progesterone
(C) Estrogen
(D) Alendronate
(E) Calcitonin

Answer: (C, D, E)

1186. Rx of postmenopausal osteoporosis:
(A) Calcitonin
(B) Alendronate
(C) Progesterone
(D) Tamoxifience
(E) Androgen

Answer: (A, B)

1187. Bone density is decrease in which of the following
(A) Osteoporosis
(B) AVN of bone
(C) Osteopetrosis
(D) Fracture and collapse of cancellous bone
(E) SLE

Answer: (A)

1188. Denosumab- a monoclonal antibody against RANKL receptor is used in treatment of
(A) Rheumatoid arthritis
(B) Osteoporosis
(C) Osteoarthritis
(D) SLE

Answer: (B)

1189. All of the following agents decreased bone resorption in osteoporosis, except:
(A) A lendronate
(B) Etidronate
(C) Strontium
(D) Terparatide

Answer: (D)

1190. Osteoporosis is characterized by:
(A) Increased serum alkaline phosphatase
(B) Decreased bone density
(C) Wasting of muscles
(D) Looser’s zone seen
(E) Decreased serum Ca2+

Answer: (B)

1191. Traumatic fracture showing avascular necrosis:
(A) Femoral neck
(B) Surgical neck of humerus
(C) Body of talus
(D) Cuboid

Answer: (A)

1192. Avascular necrosis seen in
(A) # surgical neck of humerus
(B) # neck of scapula
(C) # neck of talus
(D) # neck of femur
(E) # neck f scaphoid/radius

Answer: (C, D)

1193. Posttraumatic avascular necrosis occur in fracture of
(A) Neck femur
(B) Surgical neck humerus
(C) Neck talus
(D) Waist scaphoid
(E) Neck radius

Answer: (A, C)

1194. Most common site of avascular necrosis amongst the following is:
(A) Medial candyle of femur #
(B) Talus #
(C) Olecranon #
(D) Head of the radius #

Answer: (B)

1195. A vascular necrosis can be a possible sequelae of fracture of all of the following bones, except:
(A) Femur neck
(B) Scaphoid
(C) Talus
(D) Calanuem

Answer: (D)

1196. Avascular necrosis of head of the femur is most common in
(A) Subcapital #
(B) Basal #
(C) Fracture intertochantric
(D) Trans cervical fracture

Answer: (A)

1197. The most common site of fracture neck of femur that causes avascular necrosis is
(A) Subcapital
(B) Intertrochanteric
(C) Transcervical
(D) Basal

Answer: (A)

1198. Avascular necrosis of head of femur can occur in
(A) Sickle cell anaemia
(B) Caisson’s disease
(C) Intracapsular fracture neck
(D) Trochanteric fracture

Answer: (A, B, C)

1199. Osteonecrosis of femoral head seen in:
(A) Sickle cell disease
(B) Gaucer’s disease
(C) Polycythemia
(D) Hyperparathyroidism
(E) Fracture neck of femur

Answer: (A, B, C, E)

1200. Avascular necrosis of femur seen in:
(A) Steroid therapy
(B) Gaucher’s disease
(C) Fracture
(D) Pancreatitis

Answer: (ALL)

1201. Pathological changes in Caison’s disease is due to:
(A) N¬2
(B) O2
(C) CO2
(D) CO

Answer: (A)

1202. Avascular necrosis is seen in:
(A) Sickle cell disease
(B) Thalassemia
(C) Polychthemia
(D) Hyperparathyroidism

Answer: (A, C, B)

1203. Aseptic necrosis can occur in:
(A) Perthes disease
(B) Slipped capital femoral epiphysis
(C) Steroid administration
(D) Caisson’s disease
(E) Gaucher’s disease

Answer: (ALL)

1204. A 50-year-old man sustained posterior dislocation of lest hip in an accident. Dislocation was reduced after 3 days. He started complaining of pain in left hip after 6 months. X-ray of the pelvis were normal. The most relevant investigation at this stage will be-
(A) CRP Levels in blood
(B) Ultrasonography of hip
(C) Arthrography of hip
(D) MRI of hip

Answer: (D)

1205. A patient is using oral steroids for a period of 5 years and patient complaints of pain in the both hip regions. Which one of the following is a diagnostic modality of conformation of diagnosis?
(A) Plain X ray
(B) CT scan
(C) MRI
(D) Isotope Bone scan

Answer: (C)

1206. A woman of 45, a known cause of pemphigus vulgaris on a reglar treatment with controlled primary disease presented with pain in the right hip and knee. Examination revealed no limb length discrepancy but the patient has tenderness in the Scarpa’s triangle and limitation of adduction and internal rotation of the right hip joint as compared to the other side. The most probable diagnosis is
(A) Stress fracture of neck of femur
(B) Avascular necrosis of femoral head
(C) Perthe’s disease
(D) Transient synovitis of hip

Answer: (B)

1207. A thirty one year old male with Nephrotic syndrome complains of pain in right hip joint of 2 months duration. The movements at the h ip are free but painful terminally. The most likely diagnosis is
(A) Tuberculosis of hip
(B) Avascular necrosis of femoral head
(C) Chondrolysis of hip
(D) Pathological fracture of femoral neck

Answer: (B)

1208. A patient of nephrotic syndrome taking steroids for 6 years presented with a limp gait and of limitation of abduction and internal rotation. He most probably has:
(A) Renal osteodystrophy
(B) Avascular necrosis of hip
(C) Septic arthritis
(D) Osteomylitis of hip jt

Answer: (B)

1209. A 45 year old was given steroids after renal transplant. After 2 years he had difficulty in walking and pain in both hips. Which one of the following most likely cause?
(A) Primary Osteoarthritis.
(B) Avascular necrosis.
(C) Tuberculosis.
(D) Aluminum toxicity.

Answer: (B)

1210. Which joint is commonly involved in Osteochondritis Dissecans
(A) Ankle joint
(B) Knee joint
(C) Wrist joint
(D) Elbow joint

Answer: (B)

1211. Most common site of osteochondritis dessicans
(A) Lateral part of the medial femoral condyle.
(B) Medial part of the medial femoral condyle.
(C) Lateral part of the lateral femoral condyle.
(D) Medial part of the lateral femoral condyle.

Answer: (A)

1212. Kinebock’s disease is due to avascular necrosis of:
(A) Femoral neck
(B) Medial cuneiform bone
(C) Lunate bone
(D) Scaphoid bone

Answer: (C)

1213. What is Kineboch’s disease
(A) Osteochondritis of lunate
(B) Osteochondritis of neck of femur
(C) Osteochondritis of navicular
(D) Osteochondritis of tibial condyle

Answer: (A)

1214. Kinebock’s disease is due to avascular necrosis of:
(A) Femoral neck
(B) Medial cuneiform bone
(C) Lunate bone
(D) Scaphoid bone
(E) Medial condyle of tibia

Answer: (C)

1215. Which of the following is true about manifestation of Kienbocks disease:
(A) Constant tenderness is present
(B) Seen in old age
(C) Restriction of joint movement is late sequale
(D) Tenderness in lunate bone
(E) Ulna is shortened

Answer: (C, D, E)

1216. Osteocondritis is Osgood Schlatter disease affect which bone?
(A) Capitulum’s
(B) Matatarsal
(C) Navicular
(D) Tibial tuberosity

Answer: (D)

1217. Osgood Shalttller disease
(A) Involve the knee joint
(B) Pelvis
(C) Wrist joint.
(D) Cervical spine

Answer: (A)

1218. Osgood-Shclatter disease is
(A) A traction injury of femoral epiphysis
(B) A traction injury of posterior epiphysis of oscalcis
(C) A traction injury of vertebral epiphysis
(D) A traction injury of the tibial tubercle of the tibial epiphysis
(E) None of the above is true

Answer: (D)

1219. Osgood Schlatters disease affects:
(A) Proximal (upper) tibia
(B) Lower tibia
(C) Distal femur
(D) Proximal (upper) femur
(E) Patella

Answer: (A)

1220. Osteochondiritis is not seen in – disease
(A) Pelligrini – Stieda
(B) Panner’s
(C) Clave’s
(D) Kohler’s

Answer: (A)

1221. Islene’s disease is osteochondritis of
(A) 2nd Metacarpal
(B) 5th Metacarpal
(C) 2nd Metatarsal
(D) 5th Metatarsal

Answer: (D)

1222. Frieberg’s Osteochondritis is
(A) 2nd Metatarsal head
(B) 5th metatarsal head
(C) 2nd Metatarsal base
(D) 5th Metatarsal base

Answer: (A)

1223. Proximal interphalangeal, distal interphalangeal&Ist carpometacarpal joint involvement and sparing of wrist is a feature of
(A) Rheumatoid arthritis
(B) Pseudogout
(C) Psoraticarthropathy
(D) Osteoarthritis

Answer: (D)

1224. Least common site to be involved in osteoarthritis amongst the following is:
(A) Hip joint
(B) Knee joint
(C) Carpometacarpal joint of thumb
(D) Metacarpophalangeal joint

Answer: (D)

1225. In patient with osteoarthritis of knee joint, atrophy occur most commonly in which muscle
(A) Qudriceps only
(B) Hamstrings only
(C) Bot (a) and (b)
(D) Gastrocnemius

Answer: (A)

1226. A 35 year old male patient develops involvement of proximal and distal interphalangeal joints and Istcanpometacarpal joints with sparing of wrist and metacarophalangeal joint. The Diagnosis is
(A) Osteoarthritis
(B) Psoriatic arthropathy
(C) Rheumatoid arthritis
(D) Pseudogout

Answer: (A)

1227. Osteo arthritis following is not a predisposing factor
(A) Diabetes mellitus
(B) Defective joint position
(C) Weight bearing joints
(D) Incongruity of articular surfaces
(E) Old age

Answer: (A)

1228. In Fracture acetabulum, late complication is
(A) Osteoarthritis
(B) Tardy sciatic nerve palsy
(C) Recurrent Dislocation
(D) None of the above

Answer: (A)

1229. True about osteoarthritis except:
(A) Commonly found in adult before 50 yrs.
(B) Heberden’sodules are found
(C) Single joint involvement
(D) Lower limb deformity is seen
(E) Ankylosis occurs

Answer: (A)

1230. Osteoarthritis all are true except:
(A) Occurs after 50 years of age
(B) Single joint may be affected
(C) Heberden’s node present
(D) Lower limb deformity seen
(E) Ankylosis occurs

Answer: (NONE)

1231. Treatment of Osteo arthritis include all except:
(A) Graded muscle exercises
(B) Replacement of articular surface
(C) Correction of deformities
(D) Increase the weight bearing by the affected joint.
(E) Rest to the joint in acute phase

Answer: (D)

1232. Severe disability in primary osteoarthritis of hip is best managed by:
(A) Arthrodesis
(B) Arthroplasty
(C) Mc Murray’s osteotomy
(D) Intra-articular hydrocortisone and physiotherapy

Answer: (B)

1233. The most common site of primary osteroarthrosis is
(A) Hip joint
(B) Knee joint
(C) Ankle joint
(D) Shoulder joint

Answer: (B)

1234. Osteoarthrosis does not affect
(A) Knee joint
(B) Hip joint
(C) Interphalangeal joint
(D) Metacarpophalangeal joint
(E) Should joint

Answer: (D)

1235. Osteoarthritis commonly involves:
(A) Proximal IP joint
(B) Distal IP joint
(C) Ist carpometacarpal joint
(D) Wrist joint
(E) Distal radio-ulnar joint

Answer: (A, B, C)

1236. Involvement of PIP jt, DIP jt and the carpo-mcpjt of base of thumb with sparing the wrist is seen in:
(A) Rheumatoid arthritis
(B) Osteoarthritis
(C) Psoriatic arthritis
(D) Pseudogout

Answer: (B)

1237. Heberden’sArthropathy affects:
(A) Lumbar spine.
(B) Symmetrically large joints.
(C) Sacroiliac joints.
(D) Distal interphalangeal joints.

Answer: (D)

1238. 40 years patient having arthritis of PIP and DIP along with carpometacarpal joint of thumb and sparing of wrist and metacarpophalangeal joint, most likely diagnosis is
(A) Rheumatoid arthritis
(B) Osteoarthritis
(C) Psoriatic arthritis
(D) Pseudogout

Answer: (B)

1239. Earliest radiological sing of the Osteoarthritis is
(A) Narrowing of joint space
(B) Osteophyte formation
(C) Cystic lesion on cancellous bone
(D) Sclerosis in subchondral bone

Answer: (A)

1240. Which is characteristic F/O osteo arthritis on X-ray?
(A) Spurring
(B) Subchondral sclerosis
(C) Dimunition of cartilage space
(D) All of these

Answer: (D)

1241. Osteoarthritis is associated with all of the following except:
(A) Decrease joint space
(B) Subchondral sclerosis
(C) Osteophyte formation
(D) Ca++ deposition in joint space

Answer: (D)

1242. Osteoarthritis of Knee involves –
(A) Hamstring
(B) Quadriceps
(C) Both (a) and (b)
(D) Gastrocneimius

Answer: (B)

1243. The cause of rheumatoid arthritis is
(A) Familial
(B) Immunological
(C) Infective
(D) Traumatic

Answer: (B)

1244. The following is involved in Rheumatoid arthritis:
(A) Synovial fluid
(B) Synovial membrane
(C) Cartilage
(D) Subchondral bone

Answer: (B)

1245. Which of the following is inflammatory arthritis
(A) Rheumatoid arthritis
(B) Osteoarthritis
(C) Osteochondritis
(D) All of the above

Answer: (A)

1246. In rheumatoid arthritis, pathology starts in the
(A) Articular cartilage
(B) Capsule
(C) Synovium
(D) Muscles

Answer: (C)

1247. Inflammatory arthritis is/are all except
(A) Osteoarthritis
(B) Rheumatoid arthritis
(C) Psoriatic arthritis
(D) Gout arthritis
(E) Lymes arthritis

Answer: (A)

1248. All of the following are seen in inflammatory polyarthritis, except
(A) New bone formation
(B) Erythema
(C) Increased ER
(D) Morning stiffness more than one hour

Answer: (A)

1249. Distal interphalangeal joint is not involved in:
(A) Rheumatoid arthritis
(B) Psoriatic arthritis
(C) Multicentrichisteocytosis
(D) Neuropathic arthropalsty

Answer: (A)

1250. Which joint is spared in Rheumatoid arthritis
(A) MP joints of hand
(B) DIP joints of finger
(C) PIP joints of finger
(D) Atlanto-axial joint

Answer: (B)

1251. The most common arthritis that affects the wrist is
(A) Osteoarthritis
(B) Tuberculous arthritis
(C) Rheumatoid arthritis
(D) Gout

Answer: (C)

1252. Swan neck deformity is a feature of:
(A) Syphilitic arthritis
(B) Gouty arthritis
(C) Rheumatoid arthritis
(D) Osteo arthritis
(E) TB or Psoriatic arthritis

Answer: (C)

1253. Swan-neck deformity is
(A) Flexion of Metacarpophalangeal joint and extension at interphalangeal joint
(B) Extension at Proximal interphalangeal joint and flexion at Distalinterphalangeal joint
(C) Flexion at proximal interphalangeal joint and extension at distal interphalangeal joint
(D) Extension at Metacarpophalangeal joint and flexion interphalangeal joint

Answer: (B)

1254. Hyperextension of PIP joints and hyperflexion of DIP joint is known as:
(A) Trigger finger
(B) Boutoiner’sdeformtiy
(C) Swan neck deformity
(D) Mallet finger

Answer: (B)

1255. “Wind-sweep deformity” is seen in
(A) Ankylosing spondylitis
(B) Scurvy
(C) Rheumatoid arthritis
(D) Rickets

Answer: (C, D)

1256. A burn patient develop claw hand. Joint affected will be:
(A) Flexion at proximal Interphalangeal joint
(B) Flexion at Distal Interphalangeal joint
(C) Thumb Abduction
(D) Flexion at Metacarpophalangeal joint
(E) Extension at Metacarpophalangeal joint

Answer: (A, B, E)

1257. Butonniere deformity occur due to:
(A) Flexion of Proximal interphalangeal joint
(B) Flexion at Distal interphalangeal joint
(C) Extension at Ditalinterphalangeal joint
(D) Extension at Metarcarpophalangeal joint
(E) Flexion at Metarcarpophalangeal joint

Answer: (A, C)

1258. What is pathonomic feature of rheumatoid arthritis?
(A) Rheumatoid factor
(B) Rheumatoid nodule
(C) Morning stiffness
(D) Ulnar drift of fingers

Answer: (B)

1259. RA factor is used mainly
(A) Screening patients for Rheumatoid arthritis
(B) Predicting multisystem disease
(C) Predicting severity of disease
(D) Monitoring treatment

Answer: (B, C)

1260. Rheumatoid factor in rheumatoid arthritis is important because:
(A) RA factor is associated with bad prognosis
(B) Absent RA factor rules out the diagnosis of Rheumatoid arthritis
(C) It is very common in childhood Rheumatoid arthritis
(D) It correlates with disease activity

Answer: (A)

1261. Type of anemia seen in Rheumatoid arthritis is:
(A) Microcytic hypochromic anaemia
(B) Macrocytic hyperchromicanaemia
(C) Normocytic hypochromic anaemia
(D) Normocytic normochromic anaemia

Answer: (D)

1262. Joint not involved in Rheumatoid arthritis according to 1987 modified ARA criteria?
(A) Knee
(B) Ankle
(C) Tarsometatarsal
(D) Metatarsophalangeal

Answer: (C)

1263. All the following are true about Rheumatoid arthritis except:
(A) Positive for Anti-IgG antibody
(B) Juxta-articular osteoporosis
(C) Morning stiffness
(D) C Reactive protein indicates better prognosis

Answer: (D)

1264. Para-articular erosion are most commonly seen in:
(A) Osteoarthritis
(B) Rheumatoid arthritis
(C) Gout
(D) Acute suppurative arthritis

Answer: (B)

1265. Pain in small joints in an elderly lady is most likely due to:
(A) Rheumatic arthritis
(B) Rheumatoid arthritis
(C) Psoriatic arthritis
(D) Reiter’s disease

Answer: (B)

1266. All are features of Rheumatoid arthritis, EXCEPT:
(A) Osteosclerosis of joint
(B) Soft tissue swelling
(C) Narrowing of joint space
(D) Periarticular osteoporosis

Answer: (A)

1267. True regarding felty’s syndrome is all, EXCEPT:
(A) Splennomegaly
(B) Rheumatoid arthritis
(C) Neutropenia
(D) Nephropathy

Answer: (D)

1268. Which of the following is true regarding Rheumatoid arthritis:
(A) Typically involves small and large joints symmetrically but spares the cervical spine
(B) Causes pleural effusion with low sugar
(C) Pulmonary nodules are absent
(D) Enthesopathy prominent

Answer: (B)

1269. All of the following are well known features of Rheumatoid arthritis except
(A) Bilateral hip arthritis
(B) Erosion of distal interphalangeal joints
(C) Pleural effusion
(D) Hypocomoplementemia

Answer: (B)

1270. Which is true regarding rheumatoid arthritis
(A) Small and large joins are affected mostly
(B) Younger females are affected more commonly
(C) Diagnosed only if Rheumatoid factor is positive
(D) Life expectancy is unchanged
(E) Hepatosplenomegaly is common

Answer: (A)

1271. Extra articular C/F Rheumatoid Arthritis:
(A) Splenomegaly
(B) Fever
(C) Subcutaneous nodules
(D) Pleural effusion

Answer: (ALL)

1272. In R.A. (Rheumatoid Arthritis) true is:
(A) M.C. in young
(B) R. F. is diagnostic
(C) Small & big joints involved
(D) Felty syndrome associated

Answer: (C, D)

1273. True about Rheumatoid arthritis:
(A) Involves peripheral joints more commonly than axial joints
(B) Does not decrease life expectancy
(C) A/wvasculitis
(D) Adrenal hyperplasia

Answer: (A, C)

1274. A woman presented with right shoulder pin and Rheumatoid factor test came as negative, but the pain responded to the Prednisolone therapy. The diagnosis include
(A) Osteopetrosis
(B) Seronegative Rheumatoid arthritis
(C) Polymylagia
(D) Polymylagiarheumtica

Answer: (B)

1275. Still’s disease is
(A) Post traumatic bone formation in the lateral ligament of the knee
(B) Spastic diplegia
(C) Rheumatoid arthritis is child hood
(D) Rheumatoid arthritis in the elderly.

Answer: (C)

1276. Treatment of Rheumatoid arthritis include all except
(A) Give rest to the joints
(B) Correction of deformities
(C) Synovectomy
(D) Exercises
(E) Immuno suppressive drugs

Answer: (D)

1277. The following are rheumatoid disease modifying drugs except:
(A) Chloroquine
(B) Gold
(C) Penicillamine
(D) BAL

Answer: (D)

1278. Gold salts can be used is
(A) Ankylosing spondylitis
(B) Rheumatoid arthritis
(C) Osteoarthritis
(D) Behcet’s syndrome

Answer: (B)

1279. Indication of systemic steroids in rheumatoid arthritis is:
(A) Mononeuritis multiplex
(B) Carpul tunnel syndrome
(C) Presence of deformities
(D) Articular cartilage involvement

Answer: (A)

1280. Multiple loose bodies are seen in:
(A) Osteochondritisdessicans
(B) Synovial chondrocalcinosis
(C) Osteoarthritis
(D) Rheumatoid arthritis
(E) Osteogenesisimperfecta

Answer: (A, B, C)

1281. The following is the commonest cause of loose body in a joint
(A) Osteoarthritis
(B) Osteochondral fracture
(C) Synovial chondromatosis
(D) Ostochondritisdissecans

Answer: (D)

1282. One of the following is to be considered as differential diagnosis for foreign body in plain X-ray of knee joint:
(A) Fabella
(B) Calcified bursa
(C) Patella
(D) Chondromatosis

Answer: (D)

1283. True about osteochondromatosis:
(A) Malignant
(B) Manifest at adult
(C) Seen at 4-6 years after disease
(D) Common in women

Answer: (B, C, D)

1284. Disease where distal Interphalangeal joint is characteristically involved
(A) Psoriatic arthritis
(B) Rheumatoid
(C) SLE
(D) Gout

Answer: (A)

1285. Terminal interphalangeal joints of hands are commonly involved in:
(A) Psoriatic arthropathy
(B) Rheumatoid arthritis
(C) Still diseases
(D) Ankylosing spondylitis

Answer: (A)

1286. A 35-year old male develops involvements of PIP, DIP and metacarpopahalangeal joints with sparing of wrist and carpometacarpal joints. The probable diagnosis is:
(A) Psoriatic arthopathy
(B) Osteo arthritis
(C) Rheumatoid arthritis
(D) Pseudo gout

Answer: (A)

1287. In Psoriatic arthropathy, treatment of choice is
(A) Methotrexate
(B) PUVA therapy
(C) Corticosteroids
(D) Indomethacin

Answer: (A)

1288. All are true regarding psoriatic arthritis except:
(A) Arthritis mutilans
(B) DIP involvement
(C) Ankylosis of small joints
(D) Sacroilitis
(E) Lengthening of digit k/a telescoping

Answer: (E)

1289. In Reiter’s disease, untrue is
(A) Conjunctivitis
(B) Ulcer on palm & soles
(C) Interstitial lung disease
(D) After sexual contact

Answer: (C)

1290. Which one of the following is not associated with HLA B 27?
(A) Ankylosing spondylitis
(B) Reiter’s syndrome
(C) Sjogren’s syndrome
(D) Psoriatic arthritis

Answer: (C)

1291. Ankylosing spondylitis in associated with:
(A) HLA-B27
(B) HLA-B-8
(C) HLA-DW4/DR4
(D) HLA-DR3

Answer: (A)

1292. All are seronegative ( spondyloepiphyseal) arthritis with ocular manifestations, except-
(A) Ankylospondilitis
(B) Ritter’s disease
(C) Rheumatoid arthritis
(D) Psoriatic arthritis

Answer: (C)

1293. All the following disease are associated with HLA B-27 & Uveitis, except:
(A) Bechcet’s syndrome
(B) Psoriasis
(C) Ankylosing Spondylitis
(D) Reiter’s syndrome

Answer: (A)

1294. HLA B27 is associated with:
(A) Rheumatoid arthritis
(B) Ankylosing spondylitis
(C) Rheumatic arthritis
(D) Gouty arthritis

Answer: (B)

1295. Positivity of HLA B 27 in Ankylosing spondylitis
(A) 10%
(B) 96%
(C) 78%
(D) 100%

Answer: (B)

1296. Bamboo spine is seen in:
(A) Tuberculosis
(B) Rheumatoid arthritis
(C) Ochronosis
(D) Ankylosingspondylosis

Answer: (D)

1297. The early X-ray changes of Ankylosing spondylitis would be
(A) Disc space narrowing
(B) Anterior osteophyte formation
(C) Sacroiliac joint erosion
(D) Facetal joint ankylosis

Answer: (C)

1298. Ankylosing spondylitis the joint least commonly involved is
(A) Elbow
(B) Sacroiliac
(C) Ankle
(D) Spinal

Answer: (A)

1299. A young male presents with joint pains and backache. X-ray of spine shows evidence of sacroilitis. The most likely diagnosis is
(A) Rheumatoid arthritis
(B) Ankylosing spondylitis
(C) Polyarticular juvenile arthritis
(D) Psoriatic arthropahty

Answer: (B)

1300. One of the following is true regarding Ankylosing spondylitis
(A) Common is females
(B) Sacrolitis is common
(C) Anti DNA and antinuclear antibodies are present
(D) Symmetrical peripheral arthritis

Answer: (B)

1301. Which is not involved in Ankylosing spondylitis
(A) Knee & Ankle
(B) Sacroilac jt.
(C) Wrist & elbow
(D) Spine

Answer: (C)

1302. Which is true about Ankylosing spondylitis?
(A) Sacroliac joints unusually involved
(B) Peripheral joint is rarely involved
(C) Common manifestation if females
(D) Iritis is seen more frequently

Answer: (D, B)

1303. One of the following is not associated with Ankylosing spondylitis
(A) Pain more in the early morning period
(B) Pain is relieved in lying down position
(C) Morning stiffness more than 3 months
(D) May be associated with uveitis

Answer: (B)

1304. Which of the following agent has/have proven to be a value in treatment of Ankylosing spondylitis?
(A) Phenylbutazone
(B) Intramuscular gold
(C) Naproxen
(D) Penicillamine

Answer: (A)

1305. The treatment of choice for Ankylosing spondylitis is
(A) Phenylbutazone
(B) Radiotherapy
(C) Steroid
(D) All of the above

Answer: (A)

1306. Which joint is commonly affected in Ankylosing spondylitis?
(A) Ankle
(B) Knee
(C) Shoulder
(D) All of the above

Answer: (C)

1307. Enthesopathy is commonly found in:
(A) Rheumatoid arthritis
(B) Rheumatic fever
(C) Ankylosing spondylitis
(D) Osteoarthritis

Answer: (C)

1308. All are true regarding Ankylosing spondylitis except
(A) sacroilitis
(B) HLA-B27
(C) Iridocyclitis
(D) More common in females
(E) Spinal fractures

Answer: (D)

1309. Earliest investigation for diagnosis of Ankylosing spondylitis:
(A) MRI STIR sequence
(B) Bone scan
(C) CT scan
(D) X-ray
(E) USG

Answer: (A)

1310. A 65 yrs old man had H/o of back pain sine 3 months. ESR is raised. He also has marked stiffness on examination & mile restriction of chest movements. On X-ray, syndesmophytes are present in vertebrae. Diagnosis is:
(A) Ankylosing spondylitis
(B) Degenerative osteoarthritis of spine
(C) Ankylosinghyperosteosis
(D) Lumbar canal stenosis

Answer: (A)

1311. Bleeding into joint cavities is not common in
(A) Christmas disease
(B) Hemophilia
(C) ITP
(D) None of above

Answer: (C)

1312. All are features of haemophilic knee joint, EXCEPT:
(A) Juxta articular osteosclerosis
(B) Sub chondral cyst formation
(C) Widening of intercondylarnoth
(D) Squarring of patella

Answer: (A)

1313. Most common site for pseudotumour like growth inhaemophilicarthropathy is
(A) Quadriceps femoris
(B) Hamstring muscle
(C) Gastrocnemius
(D) Ilioopsoas

Answer: (A)

1314. Arthroscopy is contraindicated in
(A) Chronic joint disease
(B) Loose bodies
(C) Haemophilia
(D) Meniscal tear

Answer: (C)

1315. True about treatment of haemarthosis
(A) Aspiration
(B) POP
(C) Traction
(D) Compression bandage.

Answer: (A, B, D)

1316. Neuropathic joint may arise in
(A) Syringomyelia
(B) Tabesdorsalis
(C) Leprosy
(D) All of the above

Answer: (D)

1317. Following are the recognized causes of Charcot’s (Neuro-pathic) joint, except for:
(A) Peripheral neuritis
(B) Syringomyelia
(C) Tabesdorsalis
(D) Hysterical joint

Answer: (D)

1318. Most common charcot’s joints involved in diabetes mellitus are those of:
(A) Shoulder
(B) Ankle
(C) Knee
(D) Foot

Answer: (D)

1319. Clutton’s joints are:
(A) Syphilitic joints
(B) End stage Tuberculous joints
(C) Associated with trauma
(D) Usually painful

Answer: (A)

1320. The most common cause of Neuropathic joints is:
(A) Leprosy
(B) Diabetes
(C) Rheumatoid arthritis
(D) Syphilis

Answer: (B)

1321. Neuropathic joint of foot and ankle is most commonly due to
(A) CTEV
(B) Hansen’s disease
(C) Polio
(D) Mycetoma

Answer: (B)

1322. Charcot’s joints have all of the following characteristics except
(A) Copious effusion in the joint
(B) Painful limitation of joint movements
(C) Hypermobility of joint
(D) Osteophyte formation

Answer: (B)

1323. A 60-year old man with diabetes mellitus presents with painless, swollen right ankle joint. Radiographs of the ankle show destroyed joint with large number of loose bodies. The most probable diagnosis is:
(A) Charcot’s joint
(B) Clutton’s joint
(C) Osteoarthritis
(D) Rheumatoid arthritis

Answer: (A)

1324. In a patient suffering from tabesdorsallscharcot’s joint occurs most commonly at
(A) Elbow
(B) Tarsometatarsal
(C) Wrist
(D) Knee

Answer: (D)

1325. Dissociative sensory loss occurs in
(A) Tuberculosis of spine
(B) Disc prolapse
(C) Tabesdorsalis
(D) Syringomyelia

Answer: (D)

1326. The joint commonly involved in syphilitic arthritis is
(A) Hip
(B) Shoulder
(C) Wrist
(D) Knee

Answer: (D)

1327. Tertiary syphilitic arthritis most frequently involve
(A) Spine
(B) Hip
(C) Ankle
(D) Knee

Answer: (D)

1328. Joint least affected by Neuropathy
(A) Shoulder
(B) Hip
(C) Wrist
(D) Elbow

Answer: (A)

1329. Neuropathic joints of ankle and foot are most commonly caused by:
(A) Polio
(B) Club foot
(C) Mycetoma
(D) Hansen’s disease

Answer: (D)

1330. Painless effusion in joints in Congenital Syphilis are called
(A) Clutton’s joint
(B) Charcot’s joint
(C) Barton’s joint
(D) Chronic osteomyelitis

Answer: (A)

1331. False about Charcot’s joint in diabetes mellitus is
(A) Limitation of movements with bracing
(B) Arthrodesis
(C) Total ankle replacement
(D) Arthrocentesis

Answer: (C)

1332. Erosion of bone is seen with all of the following except
(A) Gout
(B) SLE
(C) Psoriasis
(D) Rheumatoid arthritis

Answer: (B)

1333. Which is NOT characterized by bony lesion
(A) Gout
(B) SLE
(C) Psoriasis
(D) Rheumatoid arthritis

Answer: (B)

1334. Erosive arthritis is not noted in
(A) Amyloidosis
(B) Hyperparathyroidism
(C) Psoriasis
(D) Sickle-cell disorder

Answer: (B)

1335. Deforming Polyarthritis is associated with all of the following except
(A) Rheumatoid arthritis
(B) Psoriatic arthritis
(C) Behcet’s syndrome
(D) Ankylosing spondylitis

Answer: (C)

1336. Synovial fluid of low viscosity seen in:
(A) Gout
(B) Septic arthritis
(C) TB
(D) Osteoarthritis
(E) Rheumatoid arthritis

Answer: (A, B, C, E)

1337. False statement about gout:
(A) Can lead to chronic arthritis
(B) Negatively bifringent crystals are present
(C) More common in premenopausal females
(D) Fbeuxostat is used in treatment
(E) Serum uric acid level may be normal during attack

Answer: (C)

1338. Which of the following is not affected in gout?
(A) Muscle
(B) Skin
(C) Cartilage
(D) Tendon
(E) Bursa

Answer: (A)

1339. Tophi of gout is fund in:
(A) Articular cartilage
(B) Synovium
(C) Skin
(D) Muscle
(E) Joint capsule

Answer: (A, B, C, E)

1340. Clinical features of Gout are all except
(A) Chronic patient with renal complication allopurinol is drug of choice
(B) +vebirefringent crystal in the Synovial fluid
(C) Hyperuricemia
(D) Mostly affects smaller joint

Answer: (B)

1341. A 35-year-old businessmen presents suddenly with severe pain, swelling and redness in left big toe in early morning. Most likely diagnosis is
(A) Rheumatoid arthritis
(B) Gouty arthritis
(C) Pseudogout
(D) Septic arthritis

Answer: (B)

1342. All of the followings drugs are used in an attack of Gout except
(A) Aspirin
(B) Idomethacin
(C) Colchicines
(D) Phenlbutazone

Answer: (A)

1343. Sudden attack of acute pain in great toe is due to
(A) Rheumatoid arthritis
(B) Gout
(C) Rheumatic fever
(D) Psoriatic arthritis

Answer: (B)

1344. “Gouty arthritis” usually involves first:
(A) Ankle
(B) Great toe
(C) Thumb
(D) Shoulder joint

Answer: (B)

1345. MC joint involved in Gout:
(A) Knee
(B) Hip
(C) MP joint of the big toe
(D) MP joint of thumb.

Answer: (C)

1346. In a patient of gouty arthritis best investigation to detect it
(A) Serum uric acid
(B) Uric acid in urine
(C) Urate crystal in synovial fluid
(D) Serum calcium level

Answer: (C)

1347. Biochemical marker in early gouty arthritis is:
(A) S. uric acid
(B) Increased urinary uric crystal
(C) Synovial fluid urate crystal
(D) Alkaline phosphate

Answer: (C)

1348. What is not rue about gout:
(A) Abrupt increase in serum urate levels is more common a cause for acute gout than an abrupt fall in urate levels
(B) Patient may be asymptomatic with high serum uric acid for years
(C) Development of arthritis correlates with level of serum uric acid
(D) Uric acid crystals are best seen by polarizing light microscope

Answer: (A)

1349. Specific test for gout is:
(A) Raised serum uric acid level
(B) Raised uric acid in synovial fluid of joint
(C) Raised urea level
(D) Raised urease enzyme level

Answer: (B)

1350. All are true about gouty arthritis except
(A) Arthritis is manifested after long attack of hyperuricemia
(B) There is good correlation between level of uric acid and severity of gouty arthritis
(C) Synovial analysis is diagnostic
(D) Allopurinol is treatment of choice in acute gout

Answer: (B)

1351. Periosteal reaction is not common in
(A) Syphilis
(B) Gout
(C) Osteomyelitis
(D) Tuberculousdactylitis

Answer: (B)

1352. In a gouty arthritis, the characteristic X-ray findings includes
(A) Osteoporosis
(B) Erosion of joint
(C) Soft tissue calcification
(D) Narrowing of joint space

Answer: (B)

1353. A person has severe pain & swelling in great toe. True statements are:
(A) Allopurinol used in acute control of gout
(B) Colchicine acts slowly
(C) Colchicine cause gastrointestinal disturbances
(D) High serum uric acid level may be not present
(E) Joint Fluid aspiration is done for investigation

Answer: (C, D, E)

1354. Characteristic crystals in pseudogout are:
(A) Calcium pyrophosphate
(B) Sodium monourate
(C) Potassium monourate
(D) Sodium pyrophosphate

Answer: (A)

1355. How to differentiate gout with pseudogout:
(A) Large joint involvement
(B) Birefringent (Particles) crystals
(C) Serum uric acid normal
(D) Associated with hyperparathyroidism
(E) Pain is very intense

Answer: (B, E)

1356. Calcification of menisci is seen in:
(A) Hyperparathyroidism
(B) Pseudogout
(C) Renal Osteodystrophy
(D) Acromegaly

Answer: (B)

1357. Soft tissue calcification around the knee is seen in
(A) Scurvy
(B) Scleroderma
(C) Hyperparathyroidism
(D) Pseudogout

Answer: (D)

1358. The most commonly involved joint in pseudo gout
(A) Knee
(B) Great toe
(C) Hip
(D) Elbow

Answer: (A)

1359. Subluxation of atlanto-occipital joint is seen in all except
(A) Gout
(B) Parpharyngeal abscess
(C) Rheumatoid arthritis
(D) Ankylosing spondylitis

Answer: (A)

1360. Chondrocalcinosis is seen in:
(A) Ochronosis
(B) Hypoparathyroidism
(C) Rickets
(D) Hypervitaminosis D

Answer: (A)

1361. The earliest manifestation of Alkaptonuria is
(A) Ankylosis of lumbodorsal spine
(B) Ochronotic arthritis
(C) Prostatic calculi
(D) Pigmentation of tympanic membrane
(E) All of the above

Answer: (B)

1362. A lady presents with right knee swelling. Aspiration was done in which CPPD crystals were obtained. Next best investigation is:
(A) ANA
(B) RF
(C) CPK
(D) TSH

Answer: (D)

1363. In a patient with gouty arthritis, synovial fluid aspiration will show:
(A) Monosodium Urate crystals
(B) Calcium Pyrophosphate crystals
(C) Mononuclear Leucocytosis
(D) Polymorphonuclear Leukocytosis

Answer: (A)

1364. X-ray of a young man shows heterotopic calcification around bilateral knee joints. Next investigation would be
(A) Serum phosphate
(B) Serum calcium
(C) Serum PTH
(D) Serum Alkaline phosphatase

Answer: (A)

1365. All of the following are associated with CV junction anomalies except:
(A) Rheumatoid Arthritis
(B) AnkyloisngSpondylosis
(C) Odonotoiddysgenesis
(D) Basilar degeneration

Answer: (B)

1366. All of the following statements about synovial fluid are true, except:
(A) Secreted primarily by type A synovial cells
(B) Follows Non-Newtonian Fluid Kinetics
(C) Contains Hyaluronic acid
(D) Viscosity is variable

Answer: (A)

1367. Which of the following statements about changes in articular crtilage with aging is not true:
(A) Total proteoglycan content is decreased
(B) Synthesis of proteoglycans is decreased
(C) Enzymtic degradation of proteoglycans is increased
(D) Total water content of cartilage is decrease

Answer: (C)

1368. Heterotropic calcification is seen in all except
(A) Ankylosing spondylitis
(B) Gouty arthritis
(C) Forrerstier’s disease
(D) Traumatic paraplegia
(E) After revision of surgery

Answer: (NONE)

1369. Common vertebral level of spondylolisthesis is
(A) L4-5
(B) L3-4
(C) L1-2
(D) T12-L1
(E) L5-S1

Answer: (E)

1370. In spondylolisthesis, there is fracture of vertebra in
(A) Spinous process
(B) Neural arch pars inter articularis
(C) Tansverse process
(D) Body

Answer: (B)

1371. True about spondylolisthesis is/are
(A) Congenital defect of posterior arch
(B) Slipping of L over S1
(C) Progressive slipping
(D) Abnormal congenital development

Answer: (A, B, C)

1372. Cervical spondylosis is more common at
(A) C1-C2
(B) C2-C3
(C) C6-C7
(D) C4-C5

Answer: (C)

1373. Cervical spondylosis
(A) Most frequently results from an incidence of acute trauma
(B) Causes compression of nerve roots to produce an upper motor neuron lesion in the lower limbs
(C) Produces pain and Parasthesia over the lateral aspect of the forearm and thumb when affecting the 6th cervical nerve
(D) Most frequently affects the upper cervical vertebrae

Answer: (C)

1374. In cervical spondylosis which part of vertebral body involved
(A) Inferior articular facet
(B) Pars interarticularis
(C) Superior articular facet
(D) All of the above

Answer: (D)

1375. Osteophytes developing at the joint at Luscka characteristically compresses spinal nerves at
(A) Intervertebral foramen
(B) Anterior part of body
(C) Posterior part of body
(D) Paradural areas

Answer: (A)

1376. Gradual painful limitation of shoulder movements in an elderly suggest that the most probable diagnosis is
(A) Arthritis
(B) Osteoarthritis
(C) Periarthritis
(D) Myositis Ossificans
(E) Fracture – dislocation

Answer: (C)

1377. Which of the following movements are restricted in Frozen shoulder?
(A) Abduction & Internal rotation
(B) Adduction & external rotation
(C) All range of movements
(D) Only abduction

Answer: (C)

1378. Painful are syndrome is seen in all except:
(A) Complete tear of supraspinatus
(B) #grater tuberosity
(C) Subacromial bursitis
(D) Suprasinatus tendinitis

Answer: (A)

1379. Which movements at shoulder gets restricted when supraspinatous torn?
(A) Flexion
(B) Adduction
(C) Abduction
(D) Rotation only

Answer: (C)

1380. ‘Tennis elbow’, is characterized by:
(A) Tenderness over the medial epicondyle
(B) Tendinitis of common extensor origin
(C) Tendinitis of common flexor origin
(D) Painful flexion and extension

Answer: (B)

1381. Tennis elbow is
(A) Olecranon bursitis
(B) Pain over the medial epicondyle
(C) Pain over the lateral epicondyle
(D) Myositis ossificans

Answer: (C)

1382. Pain & tenderness over the lateral condyle of humerus with a painful dorsi flexion of the wrist is indicative of:
(A) Golfer’s Elbow
(B) Tennis Elbow
(C) Pitcher’s Elbow
(D) Cricket Elbow

Answer: (B)

1383. A 40-year-old man was repairing his wooden shed on Sunday morning. By afternoon, he felt that the hammer was becoming heavier and heavier. He felt pain in lateral side of elbow and also found that squeezing water out of sponge hurt his elbow. Which of the muscles are most likely involved.
(A) Biceps brachii and supinator
(B) Flexor digitorumsuperficialis
(C) Extensor carpi radialislongus and brevis
(D) Triceps brachii and anconeous

Answer: (C)

1384. DeQuervian’s disease classically affects the:
(A) Flexor pollicislongus and brevis
(B) Extensor carpi radialis and extensor pollicislogus
(C) Abductor pollicislongus and brevis
(D) Extensor pollicisbrevis and abductor pollicislongus

Answer: (D)

1385. In de Quervian’s disease the following tendons are involved
(A) Abductor pollicislongus + Extensor pollicisbravis
(B) Abductor pollcisbrevis + Extensor pollicislongus
(C) Adductor pollicisbrevis + Extensor policislongus
(D) Extensor pollicislongus + Flexor pollicislongus

Answer: (A)

1386. Finkelstein’s test is associated with
(A) Dequervians disease
(B) Dupuytren’s Contracture
(C) Carpal tunnel syndrome
(D) Any of the above

Answer: (A)

1387. Which sing is positive in de Quervain’s disease?
(A) Phalen’s sign
(B) Froment’s sign
(C) Cozen’s sign
(D) Finkelstein’s sign

Answer: (D)

1388. About De Quervian’s disease, which of the following is correct:
(A) Pain after straining at thumb base
(B) Involve External pollicislongus
(C) Tense mass may visible
(D) Steroid is used to relieve symptoms
(E) Involve Abductor pollicislongus

Answer: (A, C, D)

1389. Dupuytren’s contracture is:
(A) Thickening of palmar fascia
(B) Base of little finger involved first
(C) Seen in cirrhotics
(D) Seen in epileptics on hydantoin
(E) All of the above

Answer: (E)

1390. Dupuytrens contracture is fibrosis of
(A) Palmar fascia
(B) Forearm muscles
(C) Sortorius fascia
(D) None of the above

Answer: (A)

1391. The following structure is involved in Duputren’s contracture
(A) Thickening of the palmar fascia
(B) Thickening of the dorsal fascia
(C) Contracture of the flexor tendons
(D) Post burns contracture

Answer: (A)

1392. True about Duputren’s contracture
(A) Slight preponderance in males
(B) Slight preponderance in females
(C) Much more common in males
(D) Much more common in females

Answer: (C)

1393. Dupuytren’s Contracture of the hand commonly starts in
(A) Thumbs
(B) Index finger
(C) Middle finger
(D) Ring finger
(E) Little finger only

Answer: (D)

1394. All are true of dupuytren’s contracture except
(A) Usually 4th finger is involved
(B) Bilateral disease is are
(C) Surgical release is useful
(D) May be associated with Pyroine disease

Answer: (B)

1395. The best treatment for Dupuytren’s contracture is
(A) Fasciotomy
(B) Fasciectomy
(C) Incision and release
(D) Fasciectomy + Skin transplantation

Answer: (D)

1396. Dupuytren’s contracture occur in:
(A) Diabetes Mellitus
(B) Alcohol
(C) Epilepsy
(D) Rheumatoid Arthritis
(E) Chronic Pulmonary disease

Answer: (A, B, C, E)

1397. True about Dupuytren’s contracture:
(A) A/W peyronie’s disease
(B) First affect index finger
(C) Nodule formation & thickening of palmar fascia
(D) Amputation may be required

Answer: (A, C, D)

1398. A 65 yr old alcoholic suffering from diabetes has flexion deformity of the right little finger over the metacarpophalangeal joint of around 15 degrees. The ideal management would be:
(A) Observation
(B) Percutaneous fasciotomy
(C) Partial (selective) fasciectomy
(D) Complete fasciectomy

Answer: (A)

1399. Cause of trigger finger is
(A) Thickening of the fibrous tendon sheath
(B) Following local trauma
(C) Unaccustomed activity
(D) All of the above

Answer: (D)

1400. In trigger finger the level of tendon sheath constriction is found at the level of:
(A) Middle phalanx
(B) Proximal intephalangeal joint
(C) Proximal phalanx
(D) Metacarpophalangeal joint

Answer: (D)

1401. Trigger finger is
(A) A feature of carpal tunnel syndrome
(B) Injury to fingers while operating gun
(C) Stenosis tenovaginitis of flexor tendon or affected finger
(D) Any of the above

Answer: (C)

1402. Trigger finger occurs in
(A) Rheumatoid arthritis
(B) Trauma
(C) Osteosarcoma
(D) Osteoarthritis

Answer: (A)

1403. Trigger finger is due to
(A) Tenovaginitis
(B) Synovitis
(C) Bursitis
(D) Fibrositis

Answer: (A)

1404. Trigger finger is caused by
(A) Rheumatoid arthritis
(B) Tenosynovitis
(C) Colles’ fracture
(D) Osteoarthritis

Answer: (B)

1405. The level of the constricting nodule in case of a trigger finger is at
(A) Neck of the corresponding metacarpal bone
(B) Metacarpophalangeal joint
(C) Proximal interphalangeal joint
(D) Distal interphalangeal joint

Answer: (B)

1406. Pulley involved in trigger finger
(A) A1
(B) A2
(C) A3
(D) A4

Answer: (A)

1407. Kanaval’s sign is positive in
(A) Tenosynovitis
(B) Carpel tunnel syndrome
(C) Trigger finger
(D) Dupuytren’s contracture

Answer: (A)

1408. True regarding mallet finger is
(A) Avulsion of tendon at the base of the middle phalanx
(B) Avulsion of extensor tendon at the base of the distal phalanx
(C) Fracture of distal phalanx
(D) Fracture of the proximal phalanx

Answer: (B)

1409. Mallet finger is due to avulsion of extensor tendon of:
(A) Proximal phalanx
(B) Middle phalanx
(C) Distal phalanx
(D) Metacarpals
(E) Any other phalanx

Answer: (C)

1410. A 30 year old man involved in a fisticuff, injured his middle finger and noticed slight flexion of DIP joint. X-rays were normal. The most appropriate management at this stage is-f
(A) Ignore
(B) Splint the finger in hyperextension
(C) Surgical repair of the flexor tendon
(D) Buddy strapping

Answer: (B)

1411. True about ganglion:
(A) Common in volar aspect
(B) Seen adjacent to tendon sheath
(C) Communicates with joins cavity and tendon sheath
(D) It is Unilocular

Answer: (B, C, D)

1412. Compound palmar ganglion is
(A) Tuberculosis affection of ulnar bursa
(B) Pyogenic affection of ulnar bursa
(C) Non specific affection of ulnar bursa
(D) Ulnar bursitis due to compound injury

Answer: (A)

1413. House maids knee is bursitis of:
(A) Prepatellar bursa
(B) Infrapatellar bursa
(C) Olecranon
(D) Ischial bursa

Answer: (A)

1414. Usual site of Tubercular bursitis:
(A) Prepatelar
(B) Subdeltoid
(C) Subpatellar
(D) Trochanteric
(E) None

Answer: (D)

1415. Site of TB bursitis:
(A) Prepatellar
(B) Subacromial
(C) Subdeltoid
(D) Subpatellar
(E) Trochanteric

Answer: (C, E)

1416. Which of the following cysts is medially situated
(A) Housemild’s knee
(B) Clergyman’s knee
(C) Bursa anserine
(D) Semimembranosus bursitis
(E) Morrant Baker’s cyst

Answer: (C)

1417. Hallux valgus means
(A) Outward deviation of great toe
(B) Inward deviation of great toe
(C) Outward deviation of fifth toe
(D) Inward deviation of fifth toe

Answer: (A)

1418. Hallux valgus is associated with all except
(A) An exostosis on the medial side of the head of the first metatarsal
(B) A bunion
(C) Osteoarthritis of the metatarsophalangeal joint
(D) Over-riding or under-riding of the second toe by the third

Answer: (A)

1419. In Hallux valgus surgery, the patients who are likely to be most satisfied are;
(A) Those with pain
(B) Those with hammertoe
(C) Those with metatarsus prims varus
(D) Young age

Answer: (A)

1420. The primary pathology in Athletic Pubalgia is:
(A) Abdominal muscle strain
(B) Rectus femoris strain
(C) Gluteus medius strain
(D) Hamstring strain

Answer: (A)

1421. Thoracic Outlet Syndrome is best Diagnosed By:
(A) CT scan
(B) MRI
(C) Digital subtraction angiography
(D) Clinical examination

Answer: (D)

1422. True about Osteomyelitis is A/E
(A) Staph. Aureus is most common
(B) Epiphysis most commonly involved region
(C) In sickle cell anemia – salmonella is causative organism
(D) Sequestrum is a piece of dead bone
(E) Involucrum is dense sclerotic bone overlying a seuestrum

Answer: (B)

1423. Acute Osteomyelitis is most commonly caused by
(A) Staphylococcus aureus
(B) Actinomyces bovis
(C) Nocardia asteroids
(D) Borrelia Vincentii

Answer: (A)

1424. The most common organism causing osteomyelitis in drug abusers is
(A) E. coli
(B) Pseudomonas
(C) Klebsiella
(D) Staph. Aureus

Answer: (B)

1425. Commonest cause of hematogenous osteomyelitis
(A) Streptococcus
(B) Staph aureus
(C) Salmonella
(D) H. influenza

Answer: (B)

1426. True about HIV, Osteomyelitis is all EXEPCT:
(A) Necrosis absent
(B) Often bilateral
(C) Periosteal new bone formation
(D) Most common cause is staph. Aureus

Answer: (D)

1427. Acute Osteomyelitis of long bones commonly affects the
(A) Epiphysis
(B) Diaphysis
(C) Metaphysis
(D) Articular surface

Answer: (C)

1428. When osteomyelitis disseminates by Hematogenous way the most affected part of bone is
(A) Metaphyses
(B) Epiphyses
(C) Diaphyses
(D) Any of the above

Answer: (A)

1429. The most common source of bone and joint infection is
(A) Direct spread
(B) Percutaneous
(C) Lymphatic
(D) Haematogenous

Answer: (D)

1430. Chronic osteomyelitis is diagnosed mainly by
(A) Sequestrum
(B) Bone fracture
(C) Deformity
(D) Brodie’s abscess

Answer: (A)

1431. About Sequestrum not true is
(A) Infection nidus
(B) Lighter than live bone
(C) Dead piece of bone
(D) Heavier than live bone & trabeculated

Answer: (D)

1432. Involucrum means
(A) Fragment of dead bone
(B) Hole formed in the bone during the formation of a draining sinus
(C) Osteomyelitis of spine
(D) Periosteal new bone formation around necrotic sequestrum

Answer: (D)

1433. Involucrum is found
(A) Underneath the sequestrum
(B) Around the sequestrum
(C) At metaphysis
(D) Beneath the periosteum

Answer: (B)

1434. Non-healing sinus is a common clinical feature is chronic osteomyelitis. The most common frequent cause for this presentation is
(A) Resistant organisms
(B) Retained foreign body
(C) Presence of sequestrum
(D) Intraosseous cavities

Answer: (C)

1435. A dead piece of bone is known as
(A) Involucrum
(B) Sequestrum
(C) Cloacae
(D) All of these

Answer: (B)

1436. All are associated with chronic osteomyelitis except
(A) Amyloidosis
(B) Sequestrum
(C) Metastatic abnormality
(D) Myositis ossificans

Answer: (D)

1437. Which of the following terms in inappropriate to the condition of Osteomyelitis?
(A) Cloacae
(B) Involucrum
(C) Sequestrum
(D) Myelocoele

Answer: (D)

1438. When does the lesion of Osteomyelitis appear on X-ray
(A) 2 hour
(B) 24 hours
(C) 1 week
(D) 2 week

Answer: (D)

1439. Complications of acute osteomyelitis:
(A) Malignancy
(B) Fracture of the affected bone
(C) Sepsis
(D) Chronicity

Answer: (B, D)

1440. True regarding acute osteomyelitis in a child:
(A) Diagnosis by X-ray is 8-10 days after onset
(B) There is diffuse tenderness at the site
(C) Antibiotic therapy should be at least for 4 weeks
(D) Salmonella is the most common cause

Answer: (A, B, C)

1441. Which is false regarding acute osteomyelitis?
(A) Staphylococcus is usual organism
(B) Rest and elevation relieves pain
(C) Parenteral antibiotics are given
(D) Surgery is the only treatment

Answer: (D)

1442. Acute Hematogenous osteomyelitis is treated with all except:
(A) Antibiotics
(B) Splinting
(C) Analgesics
(D) Surgery

Answer: (D)

1443. The deal treatment for acute Osteomyelitis of long bones is
(A) Antibiotics only
(B) Drilling of bone
(C) Decompression
(D) Antibiotics and delayed decompression

Answer: (A)

1444. Instillation treatment in Osteomyelitis is
(A) Continuous suction + continuous drainage
(B) Intermittent suction + continuous drainage
(C) Continuous suction + intermittent suction
(D) Intermittent suction + intermittent drainage

Answer: (A)

1445. Earliest site of bone involvement in heatogenous osteomyelitis:
(A) Metaphysis
(B) Diaphysis
(C) Epiphysis
(D) Point of entry of the nutrient artery

Answer: (A)

1446. Sclerosis of a long bone may suggest
(A) Osteoid osteoma
(B) Sclerosing Osteomyelitis
(C) Both are correct
(D) None of the above

Answer: (C)

1447. Metaphyseal lesion in children include
(A) Fracture
(B) Osteomyelitis
(C) Dislocation
(D) Ewing’s tumour
(E) Osteosarcoma

Answer: (B, E)

1448. Sabre tibia seen in
(A) Tuberculous Osteomyelitis
(B) Syphilitic osteitis
(C) Rickets
(D) Paget’s disease

Answer: (B)

1449. Brodies abscess usually involves
(A) Long bones
(B) Short bones
(C) Pelvic bones
(D) Flat bones

Answer: (A)

1450. What is Brodie’s abscess
(A) Long standing localized pyogenic abscess in the bone
(B) Cold abscess
(C) Subperiosteal abscess
(D) Soft tissue abscess

Answer: (A)

1451. Which of the following is NOT TRUE regarding tubercular osteomyelitis?
(A) It is a secondary TB
(B) Periosteal reaction is seen
(C) Sequestration is uncommon
(D) Inflammation is minimum

Answer: (B)

1452. Tom smith’s arthritis is due to:
(A) Pyogenic infection in infancy
(B) TB
(C) RA
(D) OA

Answer: (A)

1453. Tom Smith arthritis manifests as;
(A) Increase hip mobility and unstability
(B) Hip stiffness
(C) A + D
(D) shortening of limb

Answer: (C)

1454. Chondrolysis occurs commonly in
(A) T.B. arthritis
(B) Syphilitic arthritis
(C) Chondrosrcoma only
(D) Septic arthritis of infancy

Answer: (D)

1455. Acute supurative arthritis is associated with all except:
(A) May be caused by penetrating wound
(B) May be caused by a compound fracture involving a joint
(C) May be due to blood borne infection
(D) Causes the joint to be held in the position of ease
(E) Tends to end with the formation of a fibrous ankylosis

Answer: (E)

1456. Bony ankylosis result from
(A) Pyogenic arthritis
(B) Tuberculosis arthritis
(C) Osteoarthritis
(D) Rheumatic arthritis

Answer: (A)

1457. Most common cause of bony ankylosis is
(A) Rheumatoid arthritis
(B) Pyogenic arthritis
(C) Traumatic arthritis
(D) Osteoarthritis

Answer: (B)

1458. Most common cause of nongonococcal septic arthritis is
(A) Staph-aureus
(B) H. influenza
(C) Pseudomonas
(D) Streptococcus

Answer: (A)

1459. Septic arthritis in a 2 year old child is often caused by
(A) Hemophilous influezae
(B) Staphylococcus aureus
(C) Gonococci
(D) Pneumococci

Answer: (B)

1460. In actinomycosis of the spine, the abscess usually erodes:
(A) Intervertebral disc
(B) Into the pleural cavity
(C) Into the reteropritoneal space
(D) Towards the skin

Answer: (D)

1461. Most common site of Actinomycosis amongst the following is:
(A) Tibia
(B) Rib
(C) Mandible
(D) Femur

Answer: (C)

1462. Madura foot is
(A) Tuberculous infection of the bone
(B) Madura mycotic infection of the bone
(C) Parasitic infection of the bone
(D) Water borne disease of the foot

Answer: (B)

1463. A patient with swelling foot, pus discharge, multiple sinuses. KOH smear shares filamentous structures. Diagnosis is
(A) Osteomyelitis
(B) Madulra Mycosis
(C) Anthrax
(D) None

Answer: (B)

1464. A man aged 40 yrs presents with swelling over foot with multiple sinuses. What is most probable diagnosis:
(A) TB
(B) Maduromycosis
(C) Actinomyecetoma
(D) Osteomyelitis
(E) Tetanus unilateral

Answer: (B)

1465. Tuberculosis osteomyelitis is most likely:
(A) Paucibacillary and haematoganous
(B) Multibacillary and haematoganous
(C) Multibacillary and lymphatogenous
(D) Paucibacillary and lympatogenous

Answer: (A)

1466. Commonest site of skeletal tuberculosis is
(A) Tibia
(B) Radius
(C) Humerus
(D) Vertebrae

Answer: (D)

1467. The commonest infective lesion of the spine in India as:
(A) Pyogenic infection
(B) Fungal
(C) T.B.
(D) Typhoid

Answer: (C)

1468. Tuberculosis of the spine is known as:
(A) Pott’s disease
(B) Scheuermann’s disease
(C) Perthes disease
(D) Frieberg’s disease

Answer: (A)

1469. Tuberculosis spine; most common site is:
(A) Sacral
(B) Cervical
(C) Dorsolumbar
(D) Lumbosacral

Answer: (C)

1470. Commonest site for tuberculous spondylitis is:
(A) T12/L1
(B) C6-7
(C) L4-5
(D) S1-2

Answer: (A)

1471. Pott’s spine is commonest in spine
(A) Cervical
(B) Thoracic
(C) Lumbar
(D) Sacral

Answer: (B)

1472. The most common type of spinal Tuberculosis is
(A) Anterior
(B) Posterior
(C) Central
(D) Paradiscal

Answer: (D)

1473. Tuberculosis of the spine starts in:
(A) Vertebral body
(B) Nucleous pulposus
(C) Annulus fibrosis
(D) Paravertebral fasica

Answer: (A)

1474. Tuberculosis of the spine commonly affects all of the following parts of the vertebra except:
(A) Body
(B) Lamina
(C) Spinous process
(D) Pedicle

Answer: (C)

1475. Tuberculosis of the spine is thought to originate from
(A) By extension from the para vertebral structures
(B) In the cancellous vertebral body
(C) In the ligamentous structures
(D) In the nucleus pulposus

Answer: (B)

1476. Early features of spinal tuberculosis in child includes
(A) Pain on sudden movement
(B) Gradual deformity
(C) Night cries
(D) Sudden deformity

Answer: (A)

1477. Earliest feature of spinal tuberculosis is
(A) Gibbus
(B) Muscle spasm
(C) Pain
(D) Psoas abscess

Answer: (C)

1478. In TB spine the first symptom is
(A) Decreased sensation
(B) Decreased motor power
(C) Pain
(D) Increased deep tendon reflexes.

Answer: (C)

1479. First symptom in tuberculous cord compression is:
(A) Sensory change
(B) Decrease tendon reflex
(C) Spasticity
(D) Lower limb weakness

Answer: (C)

1480. Which of the following causes para vertebral abscess?
(A) Brucella
(B) Tuberculosis
(C) Kala-azar
(D) Typhoid

Answer: (B)

1481. Cold abscess in chest wall is most common due to
(A) T. B spine
(B) T. B rib
(C) T. B pelvis
(D) T. B pleura

Answer: (A)

1482. Paraplegia due to Tuberculosis of spine most commonly occurs at
(A) Cervical spine
(B) Upper thoracic spine
(C) Lower thoracic spine
(D) Lumbar spine

Answer: (B)

1483. Paraplegia is common in Tuberculosis of dorsal spine, because
(A) Incidence of Tuberculosis is commoner in dorsal spine
(B) Natural tendency for kyphosis
(C) Canal is narrow
(D) Spinal cord ends at L1
(E) All of the above

Answer: (E)

1484. In tuberculosis of spine, which one of the following is not a cause for Paraplegia?
(A) Stretching of spinal cord in gibbus deformity
(B) Spinal artery compression
(C) Compression by granulation tissue
(D) Oedema of spinal cord

Answer: (B)

1485. The most common cause of Paraplegia of early onset of Tuberculosis of spine is
(A) Spinal artery thrombosis
(B) Sudden collapse of vertebra
(C) Granulation tissue pressing on cord
(D) Cold abscess pressing on the cord

Answer: (D)

1486. The most common cause of Kyphosis in a male is
(A) Congenital
(B) Tuberculosis
(C) Trauma
(D) Secondaries

Answer: (B)

1487. The most common sequelae of tuberculous spondylitis in an adolescent is:
(A) Fibrous Ankylosis
(B) Bony-Ankylosis
(C) Pathological dislocation
(D) Chronic osteomyelitis

Answer: (B)

1488. In Bony ankylosis, there is
(A) Painless, No movement
(B) Painful complete movement
(C) Painless complete movement
(D) Painful incomplete movement

Answer: (A)

1489. Clinical features of TB spine are:
(A) Night sweat
(B) Reduced appetite
(C) Loss of lordosis
(D) Back pain

Answer: (A, B, C, D)

1490. Clinical features of TB spine are A/E
(A) Loss of lordosis
(B) Night sweat
(C) Loss of appetite
(D) Fever (evening rise)
(E) Back Pain
(F) None

Answer: (F)

1491. Clinical features of T.B. Spine are A/E:
(A) Loss of lordosis
(B) Night sweats
(C) Weight gain
(D) Evening rise of temperature
(E) ↓ed appetite

Answer: (C)

1492. True about TB spine is all except:
(A) Early paraplegia is good prognosis
(B) Insidious onset paraplegia is good prognosis
(C) Dorsolumbart spine is commonest site
(D) It is commonest site for TB of bone in the body

Answer: (B)

1493. Poor prognostic factors in pott’s paraplegia:
(A) Acute onset of paraplegia
(B) Sudden progression of paraplegia
(C) Motor paralysis alone
(D) Long standing paraplegia
(E) Paraplegia in children

Answer: (B, D)

1494. Indicator (s) of poor prognosis in the pott’s spine is/are:
(A) Healed vertebral lesion
(B) Grade IV Pott’s spine
(C) Kyphotic angle >600
(D) Short duration
(E) Acute onset

Answer: (B, C, E)

1495. The 1st sign of TB is
(A) Narrowing of intervertebral space
(B) Rarefaction of vertebral bodies
(C) Destruction of laminae
(D) Fusion of spinous processes

Answer: (A)

1496. Earliest features of TB vertebra
(A) Decreased joint space
(B) Soft tissue swelling
(C) Decreased movements
(D) Pain

Answer: (A)

1497. Earliest radiological sign of Spinal tuberculosis is
(A) Wedging of vertebra
(B) Syndesmophyte formation
(C) Formation of paravertebral abscess
(D) Decreased joint space

Answer: (D)

1498. A 35 yr old lady has chronic backache. On X ray she had a D12 collapse but Intervertebral disc space is maintained. All are possible cause except:
(A) Multiple myeloma
(B) Osteoporosis
(C) Metastasis
(D) Tuberculosis

Answer: (D)

1499. Compression of single vertebral with narrow joint space is characteristic of
(A) Caries spine
(B) Fracture spine
(C) Prolapsed intervertebral disc
(D) Scondaries spine

Answer: (A)

1500. A 46-year-old, known alcoholic, presented with pain in the dorsal spine. On examination there is tenderness at te dorso-lumbar junction. Radiograph shows destruction of the 12th dorsal vertebra with loss of disc space between D12-L1 vertebrae. The most probable diagnosis is
(A) Metastatic spine disease
(B) Pott’s spine
(C) Missed trauma
(D) Multiple myeloma

Answer: (B)

1501. X-ray showing decreased Intervertebral space and presence of Para vertebral shadow. What could be the diagnosis?
(A) Tuberculosis of spine
(B) Ankylosing spondylitis
(C) Eosinophilic granuloma
(D) Multiple myeloma

Answer: (A)

1502. Treatment of Pott’s paraplegia is
(A) Lateral decompression
(B) Posterior decompression
(C) Excision of vertebra
(D) Antituberculous drugs

Answer: (A, D)

1503. Tuberculosis of spine is best treated by
(A) Anterior fusion
(B) Posterior fusion
(C) Antero-lateral decompression
(D) Aspiration of Para spinal abscess and instillation of streptomycin

Answer: (C)

1504. The ideal surgical treatment for Pott’s paraplegia is
(A) Laminectomy and decompression
(B) Anterior decompression and bone grafting
(C) Anterolateral decompression
(D) Costotransversectomy

Answer: (B)

1505. Surgical treatment in Pott’s spine is indicated if there is
(A) Progressive loss of function in spite of medical treatment
(B) No improvement in motor power in spite of 3 months of treatment
(C) There is no improvement in fever in 3 months of treatment
(D) Patient who is an adult or middle aged

Answer: (B)

1506. Indicator 9s) of poor prognosis in the pott’s spine is/are:
(A) Healed vertebral lesion
(B) Grade IV pott’s spine
(C) Kyphotic angle >60°
(D) Short duration
(E) Acute onset

Answer: (A, B, C)

1507. All of the following movements of the hip are painful in a patient with psoas abscess, except
(A) Extension
(B) Adduction to abduction
(C) Abduction to adduction
(D) Fixed flexion to further full flexion.

Answer: (D)

1508. A psoas abscess present on the right side, the correct statement is
(A) Opposite hip flexion relieves pain
(B) Same side hip flexion relieves pain
(C) Same side extension relieves pain
(D) Kyphosis may occur

Answer: (B)

1509. Spina Ventosa results from
(A) Sarcoidosis
(B) Tuberculosis
(C) Histiocytosis X syndrome
(D) Both A + B but not C

Answer: (D)

1510. Caries sica is seen in:
(A) Hip
(B) Shoulder
(C) Knee
(D) None of the above

Answer: (B)

1511. Short long bones of hand and foot are commonly infected by the following organism
(A) Pyogenic
(B) Tuberculous
(C) Fungal
(D) All of the above

Answer: (B)

1512. The most common site of skeletal tuberculosis is
(A) Hip + Spine
(B) Knee + Hip joints
(C) Knee joint
(D) Cervical spine.

Answer: (A)

1513. The most common cause of Monoarthritis in children is
(A) Septic arthritis
(B) Tuberculous arthritis
(C) Osteoarthritis
(D) Rheumatoid arthritis
(E) Any of the above

Answer: (B)

1514. Tuberculous arthritis in advanced cases lead to:
(A) Bony ankylosis
(B) Fibrous ankylosis
(C) Loose joints
(D) Charcots joints

Answer: (B)

1515. All are features of joint Tuberculosis except
(A) Synovium is involved
(B) Synvial fluid has < 20% blood sugar
(C) Kissing arthritis – subchondral bone is involved
(D) Pain is a common feature
(E) Lymphocyte / Moncyte ratio is decreased

Answer: (B)

1516. Triple deformity of knee is classically seen in
(A) Fracture patella
(B) Tuberculosis
(C) Rheumatic arthritis
(D) Rheumatoid arthritis

Answer: (B)

1517. The most appropriate for a 20 year old man suffering from old tuberculosis arthritis of knee with triple deformity is
(A) Plaster immobilization
(B) Joint clearance and traction
(C) Total knee replacement
(D) Charnleys’ Arthrodesis

Answer: (D)

1518. Treatment of triple deformity is
(A) ATT
(B) ATT + Immobilization
(C) ATT + Immobilization + Debridement
(D) None

Answer: (B)

1519. A fiver year old child is suffering from painful restriction of all movements of hip joint, the most likely cause in our country is
(A) Congenital coax vara
(B) Tuberculosis arthritis
(C) Perthe’s disease
(D) Sequelae of septic arthritis of infancy
(E) Psoas abscess

Answer: (B)

1520. Wandering acetabulum is seen in
(A) Fracture acetabulum
(B) Dislocation of femur
(C) CDH
(D) Tuberculosis of hip

Answer: (D)

1521. Malignant bone tumors are all except
(A) Osteosarcoma
(B) Gaint cell tumor (Osteoclastoma)
(C) Aneurysmal bone cyst
(D) Chondrosarcoma
(E) Ewing’s stumor

Answer: (B, C)

1522. Fibrous bony lesions is/are:
(A) Desmoplastic fibroma
(B) Benign fibrous histocytoma
(C) Fibrous dysplasia
(D) Osteosarcoma
(E) Admantinoma

Answer: (A, C, B)

1523. Benign bone tumor is/are:
(A) Osteoid osteoma
(B) Chondroblastoma
(C) Endothelial hemangioma
(D) Osteoclastoma
(E) Osteochondroma

Answer: (A, B, D, E)

1524. Bone forming tumors are:
(A) Osteosarcoma
(B) Osteoid osteoma
(C) Giant cell tumour
(D) Osteoblastoma
(E) Chondrosarcoma

Answer: (A, B, D)

1525. Benign bone tumors are:
(A) Osteoid osteoma
(B) Osteochondroma
(C) Chondroblastoma
(D) Chondromyxooid fibroma
(E) Multiple myeloma

Answer: (A, B, C, D)

1526. All of the following tumor are benign tumour except:
(A) Chondroma
(B) Chordoma
(C) Osteochondroma
(D) Enchondroma

Answer: (B)

1527. According to a newer hypothesis Ewings sarcoma arises from:
(A) Epiphysis
(B) Diaphysis
(C) Medullary cavity
(D) Cortex

Answer: (C)

1528. Classification system of bone tumors is:
(A) Enneking
(B) Manchester
(C) Edmonton
(D) TNM

Answer: (A)

1529. According to Enneking system not true regarding an active benign tumor is
(A) Intracapsular
(B) Margin of reactive bone
(C) Thick rim of reactive bone
(D) Extended curettage is treatment

Answer: (C)

1530. Non neoplastic lesion simulating bone tumor are all except?
(A) Fibrous dysplasia
(B) Bone island
(C) Bone infract
(D) Hurler syndrome

Answer: (D)

1531. Bone tumor arising from diaphysis:
(A) Chondroblastoma
(B) Fibrous dysplasia
(C) Giant cell tumor
(D) Ewing sarcoma
(E) Aneurysmal bone cyst

Answer: (B, D)

1532. Chondroblastoma is a tumour of:
(A) Epiphysis
(B) Metaphysis
(C) Diaphysis
(D) Flat bone

Answer: (A)

1533. Bone tumors arising from diaphysis:
(A) Chondrosarcoma
(B) Weing’s tumor
(C) Osteoclastoma
(D) Chondroblastom
(E) Osteoid osteoma

Answer: (B)

1534. Which of the following occurs in epiphysis:
(A) Osteoclastoma
(B) Chondroblastoma
(C) Osteochodroma
(D) Ewing’s sarcoma
(E) Chondrosarcoma

Answer: (A, B)

1535. The following tumors are seen in metaphysis:
(A) Osteomyelitis
(B) Osteosarcoma
(C) Chondrosarcoma
(D) Osteoclastoma
(E) Wring’s sarcoma

Answer: (A, B, C )

1536. Most common site of osteogenic sarcoma is:
(A) Femur, upper end
(B) Femur, lower end
(C) Tibia, upper
(D) Tibia, lower end

Answer: (B)

1537. Solitary bone cyst is most common in the
(A) Upper end of humerus
(B) Lower end of humerus
(C) Upper end of fibula
(D) Lower end of femur

Answer: (A)

1538. Chordoma can occur over all the following sites, except:
(A) Rib
(B) Clivus
(C) Sacrum
(D) Vertebral body

Answer: (A)

1539. True about bone tumour is:
(A) Multiple myeloma-more than 55 years age and above
(B) Osteogenic sarcoma-fourth decade
(C) Chondrosarcoma – first decade
(D) Osteoclastoma – fifth decade

Answer: (A)

1540. Years old boy, LEAST common cause of proximal lytic lesion of head to femur is:
(A) Plasmacytoma
(B) Metastasis
(C) Histiocytosis
(D) Bone tumour

Answer: (A)

1541. Which of the following is the most common mutation in Ewing’s sarcoma:
(A) Translocation X: 18
(B) Translocation 11 : 22
(C) Activative mutation of G5alpha surface protein
(D) Missense mutation in EXT1

Answer: (B)

1542. A 11 year old boy presented with the complaints of pain in the right arm near the shoulder. X-ray examination revealed an expansile lytic lesion in the upper third of humerus. The most likely diagnosis is
(A) Giant cell tumour
(B) Unicameral bone cyst
(C) Osteochondroma
(D) Parostealostesarcoma

Answer: (B)

1543. Ramu, an 8-year old boy presented with pain in the arm. On x-ray his upper end of humerus demonstrates an expansile lesion in the metaphysis with breech of the overlying cortex. Most likely diagnosis is:
(A) Aneurysmal bone cyst
(B) Unicameral Bone cyst
(C) Chondroblastoma
(D) Osteoclastoma

Answer: (A)

1544. True about aneurysmal bone cyst:
(A) Endothelial lining
(B) Blood filed spaces
(C) Found in upper end of tibia
(D) Pulsatile seen in older age
(E) Seen in older age

Answer: (B, C, D)

1545. Simple bone cyst is seen most commonly in:
(A) Tibia
(B) Radius
(C) Femur
(D) Humerus

Answer: (D)

1546. Babloo a 10 year old boy presents with # of humerus. X ray reveals a lytic lesion at the upper end. Likely condition is
(A) Unicameral bone cyst
(B) Osteosarcoma
(C) Osteoclastoma
(D) Aneurysmal bone cyst

Answer: (A)

1547. A 8 year child present with fracture humerus after trivial injury X-ray shows lytic lesion diagnosis is
(A) Osteoclastoma
(B) Osteomyelitis
(C) Unicameral bone cyst
(D) Aneurysmal bone cyst

Answer: (C)

1548. A classical expansive lytic lesion in the transverse process of a vertebra is seen in:
(A) Osteosarcoma
(B) Osteomyelitis
(C) Unicameral bone cyst
(D) Aneurysmal bone cyst

Answer: (B)

1549. A classical expansile lytic lesion in the transverse process of a vertebra is seen in:
(A) Osteosarcoma
(B) Aneurysmal bone cyst
(C) Osteoblastoma
(D) Osteocalsoma
(E) Metastasis

Answer: (B)

1550. Which of the following conditions is least likely to present as an eccentric osteolytic lesion
(A) Aneurysmal bone cyst
(B) Giant cell tumor
(C) Fibrous cortical defect
(D) Simple bone cyst

Answer: (D)

1551. Differential diagnosis of simple bone cyst are:
(A) Giant cell tumour
(B) Non ossifying fibroma
(C) Enchondroma
(D) Fibrous dysplasia
(E) Eosinophilic granuloma

Answer: (B, C, D, E)

1552. True about simple bone cyst:
(A) Seen in young adult
(B) Present as well demarcated radiolucent lesions
(C) Shaggy fibrous layering
(D) Pathological fracture seen
(E) Commonest site is diaphysis

Answer: (B, C, D)

1553. Histology of unicameral bone cyst
(A) Blood filled cavities
(B) Endothelial lining
(C) Giant cells
(D) Bone formation

Answer: (C, D)

1554. Histological features of unicameral bone cysts are
(A) Blood filled cystic spaces
(B) Endothelial cell lining
(C) Fibrous tissue with cystic spaces
(D) Pseudocyst
(E) Single cavity with connective tissue lining

Answer: (E)

1555. Secondary aneurysmal bone cyst arise in
(A) Osteoclastoma
(B) Chondroblastoma
(C) Fibrous dysplasia
(D) None

Answer: (A, B, C)

1556. An 8 year old boy presents with a gradually progressing swelling and pain since 6 months over the upper tibia. On X-ray, there is a lytic lesion with selerotic margins in the upper tibial metaphysis. The diagnosis is
(A) Osteogenic sarcoma
(B) Osteoclastoma
(C) Brodie’s abscess
(D) Ewing’s sarcoma

Answer: (C)

1557. Epiphyseal tumor is-
(A) Osteoclasoma
(B) Chondromyxoid fibroma
(C) Osteosarcoma
(D) Ewing sarcoma

Answer: (A)

1558. In a young boy, x-ray of upper end of tibia shows a lytic lesion. The least likely diagnosis is
(A) Giant cell tumour
(B) Osteosarcoma
(C) Solitary bone cyst
(D) Tuberculosis

Answer: (A)

1559. When size of osteoclastoma exceeds the size of metaphysis:
(A) Tumor will be covered by cortex.
(B) Tumor will be covered by fibrous capsules
(C) Covered by thin layered of bone
(D) It is limited to metaphysis
(E) Covered by periosteum

Answer: (A, B, C)

1560. The differential diagnosis of lesion, histologically resembling giant cell tumour in the small bones of the hands or feet, includes all of the following except:
(A) Aneursymal bone cyst
(B) Fibrosarcoma
(C) Osteosarcoma
(D) Hyperparathyroidism

Answer: (B)

1561. Variant of Giant tumor is?
(A) Ossifying fibroma
(B) Non ossifying fibroma
(C) Osteosarcoma
(D) Chondroblastoma

Answer: (B, C, D)

1562. Treatment of histologically confirmed giant cell tumour:
(A) Excision, bone grafting and Chemical Cautery
(B) Excision
(C) Radiotherapy
(D) Chemical excision

Answer: (ALL)

1563. Osteoclastoma is treated with:
(A) Total bone replacement
(B) Excision
(C) Curettage
(D) Arthrodesis
(E) Chemotherapy

Answer: (A, B, C, D)

1564. Soap bubble appearance at lower end of radius, the treatment of choice is:
(A) Local excision
(B) Excision and bone grafting
(C) Amputation
(D) Radiotherapy

Answer: (B)

1565. Osteogenic sarcoma arise from:
(A) Epiphysis
(B) Metaphysis
(C) Growth plate
(D) Epiphyseal cortex
(E) Diaphysis

Answer: (B)

1566. What is the most common site of osteosarcoma:
(A) Lower end of femur
(B) Upper end of humerus
(C) Lower end of tibia
(D) Upper end of femur

Answer: (A)

1567. Which of the following bone tumour present secondaries in lung with pneumothorax
(A) Osteosarcoma
(B) Ewing’s sarcoma
(C) Osteoclastoma
(D) Chondroblastoma

Answer: (A)

1568. A pt presents with pheumothorax. Examination shows a swelling over knee. Chest x-ray shows lung nodules, give your most probable diagnosis
(A) Osteosarcoma
(B) Ewing sarcoma
(C) Multiple myeloma
(D) Osteoclastoma

Answer: (A)

1569. Radiological inv. shows sun ray appearance; dx is
(A) Osteosarcoma
(B) GCT
(C) Osteomyelitis
(D) Ewing’s sarcoma

Answer: (A)

1570. True regarding osteosarcoma is
(A) Occurs because of proliferation of osteoclasts
(B) Sunray appearance in x-ray indicates new bone formation
(C) Affects mainly males in 5th or 6th decade
(D) Lymphatic metastasis is most common

Answer: (B)

1571. Radiological features of osteosarcoma:
(A) New bone formation
(B) Sun ray appearance
(C) Codman’s triangle
(D) Soap bubble appearance
(E) Onion pool appearance

Answer: (A, B, C)

1572. 7 year old child presents with a lesion in upper tibia. X-ray shows radiolucent area with Codman’s triangle and Sunray appearance. Diagnosis is
(A) Ewing’s sarcoma
(B) Osteosarcoma
(C) Osteoid osteoma
(D) Chondrosarcoma

Answer: (B)

1573. A15 year old boy presented with mass in lower femur, which on X-ray was found to have Codman’s triangle & sunray appearance. The diagnosis is
(A) Ewing’s Tumour
(B) Osteosarcoma
(C) Osteoclastoma
(D) Osteomyelitis

Answer: (B)

1574. Sunburst appearance found in:
(A) Osteosarcoma
(B) Ewing’s sarcoma
(C) Osteoclastoma
(D) Osteoid osteoma
(E) Chondrosarcoma

Answer: (A)

1575. All of the following investigations are needed for the diagnosis if osteosarcoma, except –
(A) MRI of femur
(B) Bone marrow biopsy.
(C) Bone scan
(D) CT chest.

Answer: (B)

1576. Which of the following malignant tumors is radioresistant?
(A) Ewing’s sarcoma
(B) Retinoblastoma
(C) Osteosarcoma
(D) Neuroblastoma

Answer: (C)

1577. Management plan for osteogenic sarcoma of the lower end of femur must include:
(A) Radiotherapy , amputation, chemotherapy
(B) Surgery alone
(C) Chemotherapy + Limb Salvage Surgery + Chemotherapy
(D) Chemotherapy + radiotherapy

Answer: (C)

1578. True about osteosarcoma:
(A) Involves epiphysis of long bones
(B) Most commonly involve knee & distal femur
(C) Spread to lung through hematogenous route
(D) Exclusively found in adolescent & early adult life
(E) X-ray sunray appearance

Answer: (B, C, E)

1579. All are the predisposing factors of osteogenic sarcoma except:
(A) Paget’s disease of bone
(B) Radiation
(C) Viral infection
(D) Bone infarction

Answer: (C)

1580. True about parosteal osteosarcoma:
(A) Same prognosis as medullary type
(B) Never go to medulla
(C) May involve medulla
(D) Lies on cortical surface
(E) Radiotherapy is treatment of choice

Answer: (C, D)

1581. Features of parosteal osteosarcoma include:
(A) It invades the medullary cavity
(B) Easily diagnosed by x-ray
(C) Prognosis is same as in other forms of osteosarcoma
(D) En bloc resection/amputation is the treatment of choice
(E) Seen most commonly in younger age groups

Answer: (A, B, D)

1582. Which of the following is a pulsatile tumor:
(A) Osteosarcoma
(B) Chondrosarcoma
(C) Osteoclastoma
(D) Ewing’s sarcoma

Answer: (A)

1583. ‘T-10 Protocol’ for treatment of osteosarcoma includes all of the following, Except:
(A) High Dose Methotrexate
(B) Bleomycin, Cyclophosphamide, Doxorubicin (BCD)
(C) Vincristine
(D) Etoposide

Answer: (D)

1584. A 2 yr old boy with Ewing’s sarcoma is having radiotherapy & chemotherapy. Which of the following indicates poor prognosis:
(A) Bet 2 microglobulin
(B) Fever
(C) Thrombocytosis
(D) Young age

Answer: (B)

1585. mic-2 mutation is associated with
(A) Osteosarcoma
(B) Ewing sarcoma
(C) Alveolar soft tissue sarcoma
(D) Dematofibrosarcomprotruberance

Answer: (B)

1586. 8 year old child has fever with pain and swelling in mid-thigh. Lamellated appearance and Codman’s triangle is seen on X-ray. Histopathology shows small round cell tumor positive for MIC-2. Diagnosis is?
(A) Osteosarcoma
(B) Ewing’s sarcoma
(C) Chondroblastoma
(D) Multiple myeloma

Answer: (B)

1587. True about Ewing’s sarcoma is all except:
(A) 5% cases revels t(11-22)
(B) Arise from medullary cavity of tubular bone
(C) Arise from diaphysis
(D) N-myc chromosome

Answer: (B, C)

1588. A child 10 years of age presents with a mass on his left thigh. The mass seems to be rising from diaphysis of femur, and involving the soft tissue of thigh. The child is having fever also. Give your most probable diagnosis
(A) Osteosarcoma
(B) Ewing’s sarcoma
(C) Chondro sarcoma
(D) Malignant fibrous histiocytoma

Answer: (B)

1589. A 12 year old girl complains of pain persisting in his leg for several weeks with a low grade fever. A radiograph reveals a mass in the diaphyseal region of the left femur with overlying cortical erosion and soft tissue extension. A biopsy of the lesion shows numerous small round cells, rich in PAS positive diastase sensitive granules. The most likely histological diagnosis is
(A) Osteogenic sarcoma
(B) Osteoblastoma
(C) Ewing’s sarcoma
(D) Chondroblastoma

Answer: (C)

1590. A 15-year –old boy is injured while playing cricket. X-rays of the lef rule out of a possible fracture. The radiologist reports the boy has an evidence of aggressive bone tumor with both bone destruction and soft tissue mass. The bone biopsy reveals a bone cancer with neural differentiation. Which of the following is the most likely diagnosis?
(A) Chondrablastoma
(B) Ewing’s sarcoma
(C) Neuroblastoma
(D) Osteosarcoma

Answer: (B)

1591. A 7 year old boy with h/o trauma 2 months back now presents with fever & acute pain over thigh. On x-ray femoral shaft shows lesions with multiple laminated periosteal reaction next line of management is
(A) CRP measurement
(B) Core biopsy
(C) Tc99 MDP scan
(D) MRI

Answer: (B)

1592. Most common site of origin of adamantinoma is
(A) Mandible near molar tooth
(B) Middle alveolar margins
(C) Hard palate
(D) Mandible near symphisismenti

Answer: (A)

1593. Most common lesion of the mandible is:
(A) Ameloblastoma
(B) Squamous cell ca
(C) Osteosarcoma
(D) Osteoclstoma

Answer: (A)

1594. Most common site of admantinoma of the long bones is:
(A) Femur
(B) Ulna
(C) Tibia
(D) Fibula

Answer: (C)

1595. True about Ameloblastoma:
(A) Cystic lesion
(B) Rapidly growing
(C) Malignant disease
(D) MC site is Tibia
(E) Presented in children

Answer: (A, E)

1596. A 33-year-old man presented with a slowly progressive swelling in the middle 1/3rd of his right tibia. X-rays examination revealed multiple sharply demarcated radiolucent lesions separated by areas of dense and sclerotic bone. Microscopic examination of a biopsy specimen revealed island of epithelial cells in a fibrous stroma. Which of the following is the most probable diagnosis?
(A) Adamantinoma
(B) Osteofibrous dysplasia
(C) Osteosarcoma
(D) Fibrous cortical defect

Answer: (A)

1597. A 19 year young patient with sclerotic lesion at diaphysis. The diagnosis is:
(A) Osteoid osteoma
(B) Ewing’s sarcoma
(C) Osteoclastoma
(D) Metastasis

Answer: (A)

1598. Babu a 19 yrs old ale has a small circumscribed sclerotic swelling over diaphysis of femur; likely diagnosis is
(A) Osteoclastoma
(B) Osteosarcoma
(C) Ewing’s sarcoma
(D) Osteoid osteoma

Answer: (D)

1599. True statement (s) regarding Osteiodosteoma is/are:
(A) Malignant
(B) MC bone involvement-tibia
(C) Local excision or curettage cure
(D) Bone pain relieved by aspirin
(E) X-ray shows density surrounding radiolucent lesion

Answer: (B, C, D, E)

1600. True about non-ossifying fibroma of bone:
(A) Present until 3rd& 4th decade
(B) Eccentric
(C) Prominent sclerotic margin
(D) Histologically giant cell with areolar
(E) Metaphyseal lesion

Answer: (B, C, D, E)

1601. True about non-ossifying fibroma:
(A) Prominent at 2nd and 4th decade
(B) Prominent sclerotic margins
(C) Fibrous tissue with areolar tissue
(D) Centrally located
(E) Risk of malignancy

Answer: (B, C)

1602. Characteristic radiological feature of fibrous dysplasia is:
(A) Cortical Thickening
(B) Cortical calcification
(C) Ground glass appearance
(D) Bone enlargement

Answer: (C)

1603. Which of the following is not a benign bone tumor
(A) Osteoid osteoma
(B) Chondroma
(C) Enchondroma
(D) Chordoma

Answer: (D)

1604. Chordoma commonly involves:
(A) Dorsal spine
(B) Clivus
(C) Lumbar spine
(D) Sacrum
(E) Cervical spine

Answer: (B, D)

1605. Most common tumour in hand
(A) Exostosis
(B) Giant cell tumour
(C) Enchondroma
(D) Synovial sarcoma

Answer: (C)

1606. All are true about enchondroma except
(A) Usually asymptomatic
(B) Ollier’s disease may be associated
(C) Occurs only in short bones
(D) Radiolucent lesion with flecks of calcification
(E) Treatment is not always necessary

Answer: (C)

1607. True about Hereditary Osteohondromatosis:
(A) Occur mainly in Neonates
(B) Occur in first decade
(C) Does not until 4-6yr
(D) Occur after Puberty
(E) No age predication

Answer: (B)

1608. Most common benign tumor of the bone is
(A) Giant cell tumor
(B) Simple bone cyst
(C) Osteochondroma
(D) Enchondroma

Answer: (C)

1609. All of the following are the causes of sudden increase in pain in osteochondroma, except:
(A) Sarcomatous change
(B) Fracture
(C) Bursitis
(D) Degenerative changes

Answer: (D)

1610. All the statements are true about exotosis, except:
(A) It occurs at the growing end of bone
(B) Growth continues after skeletal maturity
(C) It is covered by cartilaginous cap
(D) Malignant transformation may occur

Answer: (B)

1611. Which of the following statements is true about osteochondromatosis:
(A) Usually affects long bones, but can also occur in skull and pelvis
(B) Usual site is metaphyseal region
(C) Also known as multiple exostoses, diaphysealaclasis
(D) It doesn’t interfee with general body stature
(E) Autosomal dominant in inheritance

Answer: (A, B, C, E)

1612. Which one of the following bone tumors typically affects the epithysis of a long bone?
(A) Osteosarcoma
(B) Ewing’ sarcoma
(C) Chondroblastoma
(D) Chondromyxoid fibroma

Answer: (C)

1613. Chondroblastoma most commonly occurs in:
(A) Metaphysis
(B) Diaphysis
(C) Epiphysis
(D) Medullary cavity

Answer: (C)

1614. Dense calcification is found in
(A) Osteosarcoma
(B) Chondroblastoma
(C) Synovial sarcoma
(D) Osteoblastoma

Answer: (B)

1615. A 15 year-old boy presented with painful swelling over the left shoulder. Radiograph of the shoulder showed an osteolytic area with stippled calcification over the proximal humeral epiphysis. Biopsy of the lesion revealed an immature fibrous matrix with scattered giant cells. Which of the following is the most likely diagnosis?
(A) Gaint Cell Tumor
(B) Chondroblastoma
(C) Osteosarcoma
(D) Chondromyxoid fibroma

Answer: (B)

1616. A 45 years male presented with an expansile lesion in the centre of femoral metaphysis. The lesion shows Endosteal scalloping & punctuate calcification. Most likely diagnosis is:
(A) Osteosarcoma
(B) Chondrosarcoma
(C) Simple bone cyst
(D) Fibrous Dysplasia

Answer: (B)

1617. True about hemangioma of bone
(A) Mostly symptomatic
(B) Peak incidence in 5th decade
(C) Constitute 10-20% of total bone tumor
(D) Overgrowth of bone occurs
(E) Hematogenous spread

Answer: (B, C, D)

1618. Which of the following statements is true regarding hemangioma of the bone:
(A) Occurs commonly in skull bones
(B) Requires observation as I is permalignant
(C) Hamartomatous in origin
(D) Forms 10-20% of the bone tumors
(E) Local gigantism occurs when it occurs in an extremity

Answer: (A, C, E)

1619. All are true for multiple myeloma except
(A) Hypercalcemia
(B) Hyperuricemia
(C) Increased S. Alkaline phosphatase
(D) Monoclonal M Band
(E) Bone marrow plasma cells <5%

Answer: (D, E)

1620. Lytic beveled lesions are seen in skull X-ray, most likely cause is-
(A) Multiple myeloma
(B) Eosinophilic granuloma
(C) Metastasis
(D) Osteosarcoma

Answer: (A)

1621. A patient with pain in back. Lab investigation shows elevated ESR. X-ray skull shows multiple punched out lytic lesions. Most imp. investigation to be done is:
(A) Serum acid phosphatase
(B) CT head with contrast
(C) Whole body scan
(D) Serum electrophoresis

Answer: (D)

1622. A 60 yrs old male has bone pain, vertebral collapse, fracture pelvis, the probable diagnosis is
(A) Multiple myeloma
(B) Scecondaries
(C) TB
(D) Hemangioma f bone

Answer: (A)

1623. Hypercalcemia& hyperglycemia is associated with:
(A) Multiple & Myeloma
(B) Ewing’s sarcoma
(C) Osteosarcoma
(D) Chondrosarcoma

Answer: (A)

1624. Three most common cancers metastasizing to bone
(A) Kidney
(B) Thyroid
(C) Breast
(D) Prostate
(E) Lung

Answer: (C, D, E)

1625. Which of the following usually produces osteoblasticsecondaries
(A) Carcinoma lung
(B) Carcinoma breast
(C) Carcinoma urinary bladder
(D) Carcinoma prostate

Answer: (D)

1626. A patient developed paraplegia. On routine examination and X-ray it was found that there are osteoblastic lesion in his spine. Most probable diagnosis is
(A) Carcinoma thyroid
(B) Ca. Prostate
(C) Breast Ca.
(D) Pancreatic Ca.

Answer: (B)

1627. Expansile lytic osseous metastases are characteristics of primary malignancy of:
(A) Kidney
(B) Bronchus
(C) Breast
(D) Prostate

Answer: (A)

1628. Metastases least common in:
(A) Skull
(B) Pelvis
(C) Vertebrae
(D) Proximal part of long bones of the upper limb
(E) Small bones of the hand

Answer: (E)

1629. Metastatic tumor least common in
(A) Pelvis
(B) Ribs
(C) Small bone of lower limb
(D) Vertebra
(E) Skull

Answer: (C)

1630. Bone metastases is common in which of the following:
(A) Nephroblastoma
(B) Neuroblastoma
(C) RCC
(D) Clear cell sarcoma

Answer: (B, C)

1631. Most common cause of bone malignancy:
(A) Secondaries
(B) Osteosarcoma
(C) Ewing’s sarcoma
(D) Osteoclastoma
(E) Chondrosarcoma

Answer: (A)

1632. Osteosclerotic bone metastasis is found most commonly in which carcinoma:
(A) Kidney
(B) Thyroid
(C) Lung
(D) Prostate

Answer: (D)

1633. True about Bone metastasis:
(A) 5% bone metastasis are symptomatic
(B) Higher serum levels of alkaline phosphatase
(C) Most common secondary in females is breast
(D) Prostate produce osteosclerotic lesion
(E) Commonly involves hand & feet bones

Answer: (B, C, D)

1634. Bone metastases commonly seen in A/E
(A) Skull
(B) Pelvis
(C) Vertebrae
(D) Small bones of hand
(E) Proximal part of long bones of upper limb

Answer: (D)

1635. Which soft tissue sarcoma commonly gives to bone secondary:
(A) Fibrosarcoma
(B) Liposarcoma
(C) Osteosarcoma
(D) Neurofibroma
(E) Synovial sarcoma

Answer: (C, E)

1636. Most common soft tissue tumour in a child:
(A) Rhabdomyosarcoma
(B) Histiocytoma
(C) Fibrosarcoma
(D) Liposarcoma

Answer: (A)

1637. Regional lymph nodes are involved in which of these:
(A) Ewing sarcoma
(B) Osteosarcoma
(C) Adamantinoma
(D) Synovial cell sarcoma

Answer: (D)

1638. All of the following statements about synovial cell sarcoma, are true,, Except
(A) Originate from synovial lining
(B) Occur more often at extra articular sites
(C) Usually seen in patients less than 50 years of age
(D) Knee and foot are common sites involved

Answer: (A)

1639. A 30 years lady presents with pain and tenderness in index finger just under the nail. She was unable to wash her hands with cold water. Patient does not reveal any history of trauma or injury. What could be a probable finding?
(A) Sausage digits
(B) Ridging of nail, discolouration and pin – head tenderness
(C) Stiffness of whole hand
(D) Hypersensitivity of finger

Answer: (B)

Latest Govt Job & Exam Updates:

View Full List ...

© Copyright Entrance India - Engineering and Medical Entrance Exams in India | Website Maintained by Firewall Firm - IT Monteur